You are on page 1of 165

Early pregnancy

20 random questions for Best of 5 Test


Options for Questions 1-1
Balanced translocations are detected in 0.3- Unbalanced translocations are detected in
A 0.5% of couples presenting with recurrent B 0.3-0,5% of couples presenting with
first trimester miscarriage recurrent first trimester miscarriage
Female partners are more likely to be The risk of miscarriage resulting from
C carriers of balanced translocations than male D chromosomal abnormalities in the embryo
partners decreases with increasing maternal age
As the number of miscarriages increases, the
E
risk of euploid pregnancy loss decreases

Explanation
With respect to chromosomal factors A(Correct answ er: C)
Question 1
associated with miscarriage
Genetic / chromosomal factors
 3-5% of couples presenting with recurrent miscarriage carry a chromosomal anomaly, most
commonly a balanced translocation.
 Incidence of balanced translocation in general population ~0.4%.
 Female is twice as likely as male to be carrier of translocation.
 Mosaicisms also common, those involving X chromosome being commonest.
 In couples with recurrent miscarriage, chromosomal abnormalities of the embryo account for
30–57% of further miscarriages.
 The risk of miscarriage resulting from chromosomal abnormalities of the embryo increases with
increasing maternal age.
 As the number of miscarriages increases, the risk of euploid pregnancy loss increases.
 Prompt referral to a clinical geneticist for genetic counseling if abnormal karyotype detected -
options include prenatal diagnosis, pre-implantation genetic diagnosis and familial
chromosome studies?

Options for Questions 2-2


A 80 – 90% B 50 – 60%
C 15 – 20% D 3 – 8%
E 1 – 2%

Explanation
A 30 year old woman is 5 weeks pregnant in
Question 2 her second pregnancy. She suffered from A(Correct answ er: B)

hyperemesis gravidarum in her first pregnancy

1
and wants to know what the recurrence rate is.
HYPEREMESIS GRAVIDARUM
 Vomiting severe enough to require hospital admission - associated with dehydration + weight
loss of at least 3kg.
 Affects 3-10 women /1000 pregnancies
 Presents in first trimester and is unusual after 16 weeks gestation.
 Peak incidence 8-12 weeks
 Aetiology not fully understood but related to HCG and TSH levels +/- psychological factors.
There is, however, no direct relationship between the severity of the disorder and HCG or TSH
levels
 An association exists with hyperthyroidism, pyridoxine deficiency, and psychological factors
 50% recurrence rate
 Diagnosis of exclusion - UTI / Gastroenteritis / pancreatitis / peptic ulceration, hepatitis,
diabetic ketoacidosis, acute appendicitis should be considered
 Associated with metabolic alkalosis - hypochloraemic alkalosis with hypokalaemia and
potassium loss in urine
 Urine is acidic despite systemic alkalosis - when alkalosis is associated with volume depletion,
bicarbonate is not excreted
 Excretion of bicarbonate only occurs with restoration of extracellular fluid volume

Options for Questions 3-3


A Biotin deficiency B Vitamin K deficiency
C Hypernatraemia D Thiamine deficiency
E Hypokalaemia

Explanation
Wernicke’s encephalopathy in women with
Question A(Correct answ er: D)
severe hyperemesis gravidarum is typically
3
secondary to
Maternal risks
 Dehydration
 Electrolyte imbalance
 Mal-nutrition
 Abnormal liver enzymes/jaundice
 Mallory-Weiss tears
 Oesophageal rupture
 Pneumothorax
 Acute tubular necrosis
 Venous thrombo-embolism
 Wernicke’s encephalopathy - due to thiamine deficiency: diplopia, ataxia, confusion and
abnormal ocular movements

2
Options for Questions 4-4
A 1.0 – 1.5 per 1000 pregnancies B 1.0 – 1.5 per 1000 live births
C 0.1 – 0.8 per 1000 pregnancies D 0.1 – 0.8 per 1000 live births
E 2.0 – 3.5 per 1000 pregnancies

Explanation
Question The incidence of gestational trophoblastic A(Correct answ er: B)
4 disease in the UK is
GESTATIONAL TROPHOBLASTIC DISEASE
 Refers to a range of trophoblast disorders that may be benign or malignant. If there is evidence
of persistent trophoblastic tissue ( based on elevated HCG) then the condition is described as
gestational trophoblastic neoplasia.
 UK incidence ~1-1.5:1000 live births with higher incidence in women from Asia
 Incidence of choriocarcinoma ~ 1:50,000 live births (UK).
 Recurrence risk of molar pregnancy = 1:80; 1:6.5 after two previous molar pregnancies.
 If recurrence occurs, 68-80% will be of the same histological type

Options for Questions 5-5


A 1 in 10 B 1 in 20
C 1 in 50 D 1 in 80
E 1 in 300

Explanation
In a woman with one previous molar
Question pregnancy, the risk of having another molar A(Correct answ er: D)
5 pregnancy is

GESTATIONAL TROPHOBLASTIC DISEASE


 Refers to a range of trophoblast disorders that may be benign or malignant. If there is evidence
of persistent trophoblastic tissue ( based on elevated HCG) then the condition is described as
gestational trophoblastic neoplasia.
 UK incidence ~1-1.5:1000 live births with higher incidence in women from Asia
 Incidence of choriocarcinoma ~ 1:50,000 live births (UK).
 Recurrence risk of molar pregnancy = 1:80; 1:6.5 after two previous molar pregnancies.
 If recurrence occurs, 68-80% will be of the same histological type

Options for Questions 6-6


A < 16 year olds B 17 – 21 year olds
C 25 – 29 year olds D 30 – 35 year olds

3
E 45 – 49 year olds

Explanation
Which age group has the lowest risk of
Question A(Correct answ er: C)
gestational trophoblastic disease?
6

RISK FACTORS FOR TROPHOBLASTIC DISEASE


 Maternal age: lowest risk in 25-29 age group, risk increases with increasing (relative risk 26.3
in 45-49 age group) and decreasing (relative risk 6.0 in <15) maternal age
 Previous molar pregnancy
 Racial origin (but see above)

Options for Questions 7-7


The woman has blood group A and her The woman has blood group A and her
A B
partner has blood group A partner has blood group O
The woman has blood group B and her The woman has blood group O and her
C D
partner has blood group B partner has blood group A
The woman has blood group AB and her
E
partner has blood group B

Explanation
The risk of malignant change in a woman with
Question A(Correct answ er: B)
gestational trophoblastic disease is highest if
7

Risk factors for malignant change / need for chemotherapy


 Maternal age: 9 fold increase in risk in >40 years compared to 20-24 age group
 Initial method of evacuation: lowest risk after vacuum aspiration
 ABO blood group: highest risk in woman with blood group A and partner with blood group O;
lowest risk in woman with blood group A and partner with blood group A
 Complete or partial mole: malignant potential higher with complete mole: about 16% require
chemotherapy compared to 0.5%% after partial mole
 Post-evacuation contraception: COCP may be used even before HCG levels have returned to
normal

Options for Questions 8-8


A The fetal heart B The gestation sac
C The yolk sac D The exocoelomic cavity
E The embryo

Explanation

4
During trans-vaginal scanning in the first
Question A(Correct answ er: B)
trimester, the first pregnancy structure that can
8
be identified is
FIRST TRIMESTER ULTRASOUND
• The gestation sac is the first pregnancy structure that can be detected - usually visualised at
31 days or 4+3 weeks from the LMP using trans-vaginal scanning and measures 2-3mm in
diameter

• Detectable by trans-abdominal scanning at ~5+3 weeks gestation

• Typically centrally located within the fundus

• Grows by 1mm per day in diameter at this stage and becomes elliptical in shape when
diameter exceeds 10mm

Options for Questions 9-9


A The maximum sac diameter B The mean sac diameter
The sum of the sac diameters measured in 3
C The maximum sac length D
planes (A + B + C)
E The diameter of the yolk sac

Explanation
During first trimester trans-vaginal scanning,
Question A(Correct answ er: B)
gestation age should be determined by
9

The diameters of the gestation sac should be measured in 3 planes from the inner edges of
the trophpblast and volume calculated from the volume of an ellipsoid = A x B x C x 0.523

• Gestation age should be estimated using mean sac diameter (A x B x C x 1/3) or sac volume.
Once the embryo is identifiable, crown-rump length should be used

• At 6 weeks gestation, mean sac diameter (5th - 95th centiles) = 16 (6 - 26)mm

Options for Questions 10-10


A A+B+C B A x B x C x 0.523
C A x B x C x 3.14 D (A x B x C) squared
E (A + B + C) x 3.14

Explanation
Question A 24 year old woman is undergoing trans- A(Correct answ er: B)
10 vaginal scanning in early pregnancy. The

5
gestational sac diameters have been measured
in 3 planes (A, B, C mm). The sac volume is
calculated from the formula
• The diameters of the gestation sac should be measured in 3 planes from the inner edges of
the trophpblast and volume calculated from the volume of an ellipsoid = A x B x C x 0.523

• Gestation age should be estimated using mean sac diameter (A x B x C x 1/3) or sac volume.
Once the embryo is identifiable, crown-rump length should be used

• At 6 weeks gestation, mean sac diameter (5th - 95th centiles) = 16 (6 - 26)mm

Options for Questions 11-11


A 4+3 – 4+5 weeks B 4+5 – 5+2 weeks
C 5+2 – 6+0 weeks D 6+1 – 6+4 weeks
E 6+3 – 6+6 weeks

Explanation
During trans-vaginal scanning in the first
Question A(Correct answ er: C)
trimester, the embryonic pole is typically
11
visualised by

Trans-vaginal scanning: Landmarks

• At 5+1 - 5+5 weeks, the yolk sac should be detectable in the chorionic cavity and should be
detectable in all viable pregnancies with a mean sac diameter > 12mm

• 5+2 - 6+0 weeks, the embryonic pole detectable at 2-4mm with cardiac pulsation. Embryo
usually detectable with mean sac diameter >18mm

• 6+1 - 6+6 weeks, the embryo is kidney shaped. Crown-rump length 4-10mm

• 7+0 - 7+6 weeks, the crown-rump length is 11 - 16mm

• At 9-10 weeks, the crown-rump length is 23-32mm and the embryonic heart rate peaks at
170 - 180 bpm

Options for Questions 12-12


Diabetes mellitus is more common in Thyroid dysfunction is more common in
A women with recurrent miscarriage comparedB women with recurrent miscarriage compared
to the general population to the general population
Insulin resistance is more common in Polycystic ovaries are more common in
C D
women with recurrent miscarriage compared women with recurrent miscarriage compared

6
to matched fertile controls to the general population
Raised serum leutenising hormone level is
E predictive of miscarriage in women with
recurrent miscarriage

Explanation
With respect to endocrine factors associated
Question A(Correct answ er: C)
with miscarriage
12

 The prevalence of diabetes mellitus and thyroid dysfunction in women who suffer recurrent
miscarriage is similar to that reported in the general population.
 PCOS is associated with an increased risk of miscarriage and this has been attributed to
insulin resistance.
 The prevalence of insulin resistance is increased in women with recurrent miscarriage
compared with matched fertile controls.
 An elevated free androgen index appears to be a prognostic factor for a subsequent
miscarriage in women with recurrent miscarriage
 However, PCO, elevated serum LH and elevated serum testosterone levels do not predict an
increased risk of miscarriage in ovulatory women with a history of recurrent miscarriage who
conceive spontaneously.

Options for Questions 13-13


Thrombophilia screen and TORCH infection
A TORCH infection screen B
screen
TORCH infection screen and pelvic Thrombophilia screen and pelvic ultrasound
C D
ultrasound scan scan
TORCH infection screen, thrombophilia
E
screen and pelvic ultrasound scan

Explanation
A healthy 23 year old woman attends the
emergency gynaecology clinic following a
positive pregnancy test 2 days earlier. She has
abdominal pain and vaginal bleeding. Half an
Question A(Correct answ er: D)
hour later, she suffers a spontaneous
13
miscarriage. Post-mortem confirms a 15-16
weeks normal fetus. The following
investigations are recommended

 All women with recurrent first-trimester miscarriage and all women with one or more second-
trimester miscarriages should have a pelvic ultrasound to assess uterine anatomy. Suspected

7
uterine anomalies may require further investigations to confirm the diagnosis, using
hysteroscopy, laparoscopy or 3-D pelvic ultrasound
Screening for thrombophilias
 All women with recurrent first-trimester miscarriage and all women with one or more second-
trimester miscarriage should be screened before pregnancy for anti-phospholipid antibodies.

Options for Questions 14-14


A Low molecular weight heparin B Reassure and no additional treatment
Low molecular weight heparin + low dose
C D Unfractionated heparin + low dose aspirin
aspirin
E Vaginal progesterone plus low dose aspirin

Explanation
A 25 year old woman with recurrent first
trimester miscarriage was found to have
antiphospholipid antibodies at the time of the
Question third miscarriage but no other abnormalities. A A(Correct answ er: B)
14 repeat test for antiphospholipid antibodies 12
weeks later was negative. She is referred to the
antenatal clinic at 6 weeks gestation. Which
one is the most appropriate treatment?
Unexplained miscarriage
 Women with unexplained recurrent miscarriage have an excellent prognosis for future
pregnancy outcome without pharmacological intervention if offered supportive care alone in the
setting of a dedicated early pregnancy assessment unit.
 Pre-implantation genetic screening with in vitro fertilisation treatment in women with
unexplained recurrent miscarriage does not improve live birth rates.
 Aspirin alone or in combination with heparin does not reduce the risk of miscarriage in women
with unexplained recurrent miscarriage

Options for Questions 15-15


A Dizziness, fainting or syncope B Breast tenderness
C Vaginal bleeding D Shoulder tip pain
E Pain on defecation

Explanation
Which one is considered a common symptom
Question A(Correct answ er: C)
of ectopic pregnancy by NICE?
15

MANAGEMENT OF ECTOPIC PREGNANCY

8
Common symptoms of ectopic pregnancy include:
• abdominal or pelvic pain
• amenorrhoea / missed period
• vaginal bleeding with or without clots
Other reported symptoms include:
• breast tenderness
• gastrointestinal symptoms
• dizziness, fainting or syncope
• shoulder tip pain
• urinary symptoms
• passage of tissue
• rectal pressure or pain on defecation.
Common signs of ectopic pregnancy include:
• pelvic tenderness
• adnexal tenderness
• abdominal tenderness
Other reported signs include:
• cervical motion tenderness
• rebound tenderness or peritoneal signs
• pallor
• abdominal distension
• enlarged uterus
• tachycardia (more than 100 beats per minute) or hypotension (less than 100/ 60
mmHg)
• shock or collapse
• orthostatic hypotension.
Ectopic pregnancy should be excluded even in the absence of risk factors. About a third of
women with an ectopic pregnancy have no known risk factors.

Options for Questions 16-16


A The woman is less than 20 years old B There is an adnexal mass of 45 mm or larger
The woman has a previous ectopic
C Serum beta HCG level is over 2500 IU / L D
pregnancy
E The woman has significant pain

Explanation
A woman attends the emergency
gynaecology clinic with a history of 8 weeks
Question amenorrhoea, abdominal pain and vaginal A(Correct answ er: E)
16 bleeding. A diagnosis of ectopic pregnancy
has been made. Surgery should be offered as
first line treatment if
SURGICAL TREATMENT

9
Offer surgery where treatment with methotrexate is not acceptable to the woman.
Offer surgery as a first-line treatment to women who are unable to return for follow-up after
methotrexate treatment or who have any of the following:
• an ectopic pregnancy and significant pain
• an ectopic pregnancy with an adnexal mass of 35 mm or larger
• an ectopic pregnancy with a fetal heartbeat visible on an ultrasound scan
• an ectopic pregnancy and a serum hCG level of 5000 IU/litre or more.
Offer the choice of either methotrexateor surgical management to women with an ectopic
pregnancy who have a serum hCG level between 1500 IU/litre and 5000 IU/litre, who are able
to return for follow-up and who meet all of the following criteria:
• no significant pain
• an unruptured ectopic pregnancy with an adnexal mass smaller than 35 mm with no
visible heart beat no intrauterine pregnancy (as confirmed on an ultrasound scan).

Options for Questions 17-17


A Quicker recovery B Shorter operating time
A 50-60% chance of a future intra-uterine A 5-10% chance of a future intra-uterine
C D
pregnancy pregnancy
An 80-90% risk of recurrent ectopic
E
pregnancy

Explanation
A 23 year old nulliparous woman presents
with 8 weeks amenorrhoea and right sided
abdominal pain. She has a previous left
salpingectomy for ectopic pregnancy. She is
Question A(Correct answ er: C)
thought to have a right ectopic pregnancy. She
17
should be informed that compared to
salpingectomy, salpingostomy is associated
with

SALPINGECTOMY V SALPINGOTOMY ??
Both tubes present - there does not appear to be a difference in subsequent intra-uterine
pregnancy rate (46 v 44%)??
Recurrent ectopic pregnancy rate appears to be higher after salpingotomy although data are
conflicting??
Risk of persistent trophoblastic tissue higher after salpingotomy - monitor HCG levels; risk of
tubal bleeding in the immediate post-op period.
These risks should be discussed and documented if salpingotomy is being considered or is
requested??
In women with one tube only or contra-lateral tubal disease - tubal conservation associated
with a 54% intra-uterine pregnancy rate but a 20.5% recurrent ectopic pregnancy rate -

10
appropriate pre-op counselling required. This is however, cost effective when the requirement
for IVF is considered.?

Options for Questions 18-18


A 24 hours after treatment B 48 hours after treatment
C 2 and 4 days after treatment D 4 and 7 days after treatment
E 2 and 7 days after treatment

Explanation
A 23 year old woman with a previous
salpingectomy for ectopic pregnancy is found
to have another ectopic pregnancy. She is
Question A(Correct answ er: D)
being counseled about treatment with single
18
dose methotrexate. She will need a repeat
serum beta HCG test

Increase in abdominal pain is reported by 59% of women following methotrexate


administration??
A transient increase in beta-HCG may occur in up to 86% of women between days 1 and 4 of
treatment.
Serum beta-HCG should be measured on days 4 and 7.
A further dose of methotrexate should be considered if HCG levels fall by less than 15%
between days 4 and 7.??
If medical treatment is offered, women should be given clear written information about the
possible need for further treatment and potential complications.
Women should be able to return easily for assessment at any time during follow-up??

Options for Questions 19-19


A viable early intra-uterine pregnancy is a TV ultrasound scan should be repeated
A B
possible diagnosis within 48h
Surgical management should be
C The woman can be managed expectantly D
recommended
Medical treatment with methotrexate is
E
contra-indicated

Explanation
A 34 year old woman presents with 8 weeks of
Question amenorrhoea and a brown vaginal discharge. A(Correct answ er: C)
19 Clinical examination is unremarkable. Urine
pregnancy test is positive and TV scan shows

11
an empty uterus with no adnexal masses.
Serum beta HCG is 1123 IU/L and rises to
1246 IU/L 48h later.

EXPECTANT MANAGEMENT??Pregnancy of unknown location


 Serum HCG levels are below the discriminatory zone (level at which it is assumed that a viable
intra-uterine pregnancy would be visualised on trans-vaginal scan: 1000 - 2000 iu/l)??
 If no pregnancy is detectable on scan, the pregnancy is of unknown location??
 Discriminatory zone dependent on quality of ultrasound equipment, experience of the
sonographer, prior knowledge of the woman’s risks and symptoms and the presence of factors
such as fibroids and multiple pregnancy??
 Women with minimal / no symptoms can be managed expectantly with 48-72h follow-up and
active management if symptomatic, HCG levels rise above discriminatory zone or levels
plateau??
 Intervention may be required in 23-29% of cases??
 Monitor serum HCG until below 20iu/l??
 Provide clear written information on the importance of compliance with follow-up and should
have easy access to the hospital?

Options for Questions 20-20


She should be followed up until HCG levels There is a 5-10% risk that she will require
A B
fall below 50 IU/L additional intervention
She should be advised to avoid pregnancy Intervention should be recommended if
C D
for 3 months after treatment HCG levels rise above 2000 IU/L
TV scan should be repeated every 48 hours
E
during follow-up

Explanation
A 34 year old woman presents with 8 weeks of
amenorrhoea and a brown vaginal discharge.
Clinical examination is unremarkable. Urine
Question pregnancy test is positive and TV scan shows A(Correct answ er: D)
20 an empty uterus with no adnexal masses.
Serum beta HCG is 1123 IU/L and rises to
1246 IU/L 48h later. Following counseling,
she wishes to have expectant management.

EXPECTANT MANAGEMENT??Pregnancy of unknown location


 Serum HCG levels are below the discriminatory zone (level at which it is assumed that a viable
intra-uterine pregnancy would be visualised on trans-vaginal scan: 1000 - 2000 iu/l)??
 If no pregnancy is detectable on scan, the pregnancy is of unknown location??

12
 Discriminatory zone dependent on quality of ultrasound equipment, experience of the
sonographer, prior knowledge of the woman’s risks and symptoms and the presence of factors
such as fibroids and multiple pregnancy??
 Women with minimal / no symptoms can be managed expectantly with 48-72h follow-up and
active management if symptomatic, HCG levels rise above discriminatory zone or levels
plateau??
 Intervention may be required in 23-29% of cases??
 Monitor serum HCG until below 20iu/l??
 Provide clear written information on the importance of compliance with follow-up and should
have easy access to the hospital?
N = 102

MEDICAL DISORDER OF PREGNANCY

Options for Questions 1-1


Low dose combined oral contraceptive Combined oral contraceptive with 50
A B
pill microgram oestrogen
Depo-medroxyprogesterone acetate
C Progestogen-only implant D
every 12 weeks
E Progestogen-only oral contraceptive

Explanation
A 23 year old woman with a history of
epilepsy is treated with phenytoin.
Question A(Correct answ er: B)
Which one is the most appropriate
1
contraceptive option?

 The following anti-epileptics DO NOT induce the P-450 system: Benzodiazepines, gabapentin,
lamotrigine, valproate
 In women of childbearing potential, the possibility of interaction with oral contraceptives should
be discussed and an assessment made as to the risks and benefits of treatment with individual
drugs
 The progestogen-only pill is not recommended as reliable contraception in women taking
enzyme-inducing AEDs
 The progestogen implant is not recommended in women taking enzyme-inducing AEDs
 The use of additional barrier methods should be discussed with women taking enzyme-
inducing AEDs and oral contraception or having depot injections of progestogen
 Women taking phenobarbital, phenytoin, carbamazepine, felbamate, topiramate, or
oxcarbazepine should take an oral contraceptive pill containing at least 50mcg of oestrogen
 Women taking other AEDs can take a normal dose oral contraceptive pill.

13
Options for Questions 2-2
Two tablets of low dose COCP followed
A Cu IUCD B
by two further tablets 12h later
Levonorgestrel 0.75 mg two doses 12h Levonorgestrel 1.5 g followed by 0.75
C D
apart mg 12h later
Levonorgestrel 1.5 g two doses 12h
E
apart

Explanation
A 23 year old woman with a history of
epilepsy requests emergency
Questioncontraception 6 hours after unprotected A(Correct answ er: C)
2 intercourse. Her epilepsy is treated with
sodium valproate. Which one is the most
appropriate treatment option?

If emergency contraception is required in women taking enzyme-inducing AEDs:
Levonorgestrel two tablets (1.5mg) followed 12 hours later by a single tablet (0.75mg),
although this is outside the product license.
 The following anti-epileptics DO NOT induce the P-450 system: Benzodiazepines, gabapentin,
lamotrigine, valproate

Options for Questions 3-3


A Gabapentin B Lamotrigine
C Phenytoin D Sodium valproate
E Carbamazepine

Explanation
Plasma levels of which anti-epileptic drug
Questionare typically reduced when women start A(Correct answ er: B)
3 taking oestrogen-containing
contraceptives?

Discuss with women who are taking lamotrigine that the simultaneous use of any oestrogen-
based contraceptive can result in a significant reduction of lamotrigine levels and lead to loss
of seizure control.
 When a woman starts or stops taking these contraceptives, the dose of lamotrigine may need
to be adjusted

14
Options for Questions 4-4
By the time of the first missed period (4
Renal plasma flow increases by 20-30%
A B weeks from LMP), creatining clearance
by mid-trimester
has increased by 25%
GFR increases mainly in the third Plasma urea rises during the first
C D
trimester trimester
Plasma creatinine is higher in
E
pregnancy compared to non-pregnancy

Explanation
QuestionWith respect to renal physiology during A(Correct answ er: B)
4 pregnancy

RENAL DISEASE AND PREGNANCY ??PHYSIOLOGY ??


 50-80% increase in renal plasma flow by mid-pregnancy then decreases towards non-pregnant
values in third trimester.
 Creatinine clearance increases by 25% at 4 weeks from LMP and by 45% at 9 weeks,
decreases towards non-pregnant values in third trimester.??
 GFR increases in first trimester, peaks ~13 weeks and decreases in third trimester.??
 Fall in plasma osmolality of ~ 8-10 mOsmol/kg??
 Fall in plasma urea and creatinine – ‘normal’ levels in a pregnant woman may therefore
indicate underlying renal disease??
 Dilatation of renal calyces / renal pelvis / ureter above the pelvic brim - more prominent on the
right (secondary to dextrorotation of the uterus and dilatation of the right ovarian venous
plexus), may occur in first trimester but more common in late pregnancy.??
 Increased risk of vesico-ureteric reflux in third trimester.??
 Change in tubular function with increased glucosuria.???

Options for Questions 5-5


A The underlying cause of renal disease B Renal function at the time of conception
C The level of proteinuria D Blood pressure at conception
E Urea concentration at conception

Explanation
In women with chronic renal
Question A(Correct answ er: B)
insufficiency, pregnancy outcome is
5
mainly determined by

EFFECT OF PREGNANCY ON RENAL FUNCTION ??


· Prognosis depends on renal function at the time of conception and the presence of co-
morbidities like hypertension and proteinuria.??

15
· In women with mild renal insufficiency (serum creatinine < 1.5mg/dl), renal function is
usually preserved. The risk of progression to end stage renal failure within 12 months of
delivery is ~ 6%, 20% and 45% for mild, moderate and severe renal impairment respectively??
· Rapid deterioration in renal function may occur in some women - reasons not fully
understood but may be precipitated by UTIs, hypertension and proteinuria during pregnancy??
· With the exception of lupus nephritis (flares increase the risk of renal failure), the
aetiology of the renal insufficiency seems irrelevant with regard to prognosis in pregnancy.??
· Diabetic nephropathy may deteriorate rapidly in up to 45% of patients.??

Options for Questions 6-6


Prednisolone does not cross the
A B Cyclosporine is not terratogenic
placenta
Tacrolimus is associated with neonatal Azathioprine does not cross the
C D
adrenal insufficiency placenta
E Tacrolimus does not cross the placenta

Explanation
QuestionWith respect to immunosuppressive A(Correct answ er: B)
6 drugs in pregnancy

Immunosuppressive drugs in pregnancy ??


· Prednisolone crosses the placenta. Fetal complications include neonatal adrenal
insufficiency and thymic hypoplasia - unlikely to occur if the dose is less than 15 mg/d.??
· Azathioprine is teratogenic in animals but not humans. Crosses the placenta but the
fetal liver cannot convert it to its active form, 6-mercaptopurine. Associated with SGA babies
and dose-related myelosuppression in the fetus.??
· Cyclosporine - not terratogenic but associated with SGA babies.??
· Tacrolimus crosses the placenta and associated with hyperkalemia and renal
insufficiency.??
· Insufficient data are available on the use of mycophenolate mofetil and sirolimus.??

Options for Questions 7-7


If her pregnancy progresses beyond the
Fertility decreases following renal
A B first trimester, the likelihood of a
transplant
successful outcome is 30 – 40%
The risk of congenital abnormalities is The risk of neonatal immune deficiency
C D
not increased is not increased
The risk of perinatal viral infection is not
E
increased

Explanation

QuestionA 35 year old woman has been referred A(Correct answ er: C)

16
7 for pre-conception counseling 12 months
after a renal transplant. She should be
informed that

Pregnancy & renal transplant


Fertility
• Chronic renal disease associated with anovulation and sub-fertility
• Fertility usually restored following renal transplant
• Pregnancy occurs in 5-12% of women of reproductive age following renal transplant.
50% of these pregnancies are unplanned.
• Pregnancy success rate exceeds 90% after the first trimester.
Fetal risks
• Prematurity ~50%
• Growth restriction / SGA
• Perinatal infections especially Hep B and CMV
• Immune deficiency
• Risk of congenital anomalies not increased

Options for Questions 8-8


Every 2 weeks until 24 weeks then
A Every 2 weeks B
weekly
Every 4-6 weeks until 24 weeks then Every 4 weeks until 28 weeks then
C D
every 2 weeks every 2 weeks
E Every 4-6 weeks

Explanation
A 35 year old woman with renal
transplant attends the antenatal clinic at
Question10 weeks gestation. Her pre-pregnancy A(Correct answ er: E)
8 serum creatinine was 102 microM and all
her other blood tests were normal. Her
renal function tests should be preformed
Antenatal care
• In obstetric medical clinic with a nephrologist
• BP monitored at least every 2 weeks
• Monthly FBC
• Plasma levels of immunosuppressive drugs
• Monitor renal function every 4-6 weeks to detect evidence of rejection. Consider
biopsy if rejection is suspected
• U&E
• Ca, phosphate, plasma protein, LFT
• 24h urine protein & creatinine clearance
• Screen for infection

17
• Monthly MSU – treat asymptomatic bacteruria with a 2 week course of antibiotics and
consider prophylactic treatment
• CMV and Herpes testing (cervical culture) twice during pregnancy
• Serial growth scans – increased risk of SGA

Options for Questions 9-9


Affects about 1 in 3000 pregnant
A B Typically gets better after delivery
women
Is mainly caused by Graves disease
Has no effect on the fetus unless the
C which is associated with thyroid D
woman is treated with anti-thyroid drugs
stimulating IgG antibodies
Is not associated with neonatal
E
hypothyroidism

Explanation
Question A(Correct answ er: C)
Hyperthyroidism in pregnancy
9

HYPERTHYROIDISM ??
· Affects ~1:500 pregnancies. 50% of affected women have a positive family history of
thyroid disease??
· 90% due to Graves disease - secondary to autoimmune thyroid stimulating antibodies
(IgG, antibodies) which cross the placenta??
· Symptoms may improve in third trimester??
· Risk of post-partum exacerbation??
· Other causes include toxic multinodular goitre, thyroiditis, thyroid carcinoma, struma
ovarii and trophoblastic disease??
· Pregnancy in women with treated Graves disease may be complicated by fetal /
neonatal thyrotoxicosis due to persistent antibodies - fetal monitoring and neonatal
assessment are indicated.??

Options for Questions 10-10


A Severe hypertension B Cardiac failure
C Placental abruption D Acute renal cortical necrosis
E Difficult intubation

Explanation
Which one is not a recognized
Question A(Correct answ er: D)
complication of poorly controlled
10
maternal hyperthyroidism in pregnancy?
Maternal risks ??

18
· Severe untreated thyroid disease causes anovulation and infertility??
· Retro-sternal extension may cause tracheal obstruction and difficult intubation??
· Cardiac failure, hypertensive disorders??
· Increased risk of placental abruption??
· Thyroid storm (25% maternal mortality) - especially during labour??

Options for Questions 11-11


Circulating levels of TSH receptor The dose of anti-thyroid drug typically
A B
antibodies fall during pregnancy needs to be increased during pregnancy
Breast feeding is contra-indicated in
The dose of anti-thyroid drug typically women with Graves disease but not in
C D
needs to be reduced after delivery women with other forms of
hyperthyroidism
Free T3 and free T4 should not be used
E to monitor treatment with antithyroid
drugs

Explanation
Question A(Correct answ er: A)
In pregnant women with Graves disease
11

Monitor free T3/T4 to avoid maternal hypothyroidism?Reduce dose of anti-thyroid drugs to


lowest effective dose - ensure biochemically and clinically euthyroid (TFT every 1-3 months,
monthly if newly diagnosed)??
· In Graves disease, circulating levels of TSH receptor antibodies fall during pregnancy
and anti-thyroid drugs may not be needed in the third trimester.
· However, it must be ensured that the woman is euthyroid during labour as there is a risk
of thyroid storm.
· There is also a risk of post-partum relapse and follow-up is essential.??
· Monitor fetal growth??
· Risk of post-partum exacerbation - monitor and treat??
· Evaluate neonate for goitre, hyper or hypo-thyroidism - TFT on cord blood and at regular
intervals if breast-feeding??
· Breast-feeding not contra-indicated - very low concentration of PTU in breast-milk??

Options for Questions 12-12


A After 1-2 weeks B After 2-3 weeks
C After 3-4 weeks D After 4-6 weeks
E After 6-10 weeks

Explanation

19
A 35 year old woman with
hypothyroidism has been referred to the
antenatal clinic at 27 weeks gestation.
She is taking thyroxine 50 micrograms
Questiondaily. She feels generally tired and low. A(Correct answ er: D)
12 Her TSH level checked by her GP is 15.2
mU/L, free thyroxine = 2.6 pmol/l (normal
10-26). Following adjustment of her
therapy, when should her thyroid function
tests be repeated?

Management of hypothyroidism

· Thyroxine replacement - treat pre-pregnancy if possible??


· Monitor using TSH, free T3/T4 (every trimester) and 4-6 weeks after any dose
adjustment??
· Dose of thyroxine does not need to be changed during pregnancy provided the woman
is euthyroid before pregnancy
· Thyroxine requirement increases by 30-50% during pregnancy, especially during the
first trimester.
· Dose adjustments should be based on TFTs. After delivery, dose should be reduced to
pre-pregnancy dose.??
· Transient hyperthyroidism may occur 6-12 weeks post-partum??
· Neonatal hypothyroidism is very rare and is a result of trans-placental transfer of
blocking antibodies.

Options for Questions 13-13


A Betamethasone B Phenytoin
C Co-amoxiclav D Paracetamol
E Codein phosphate

Explanation
QuestionWhich drug is known to increase the A(Correct answ er: B)
13 metabolism of thyroxine?

Drugs affecting dosage of levothyroxine


Drugs preventing absorption of levothyroxin
 Calcium salts
 Ferrous sulphate
 Aluminium hydroxide
 Cholestyramine
Drugs increasing clearance of levothyroxin
 Phenytoin
 Carbamazepin

20
 Phenobarbitone
 Rifampicin

Options for Questions 14-14


Is associated with thyroid-stimulating Recurs in up to 25% of women in
A B
IgG antibodies subsequent pregnancies
Should be treated based on thyroid
Typically presents 7-14 days after
C function tests rather than maternal D
delivery
symptoms
Is associated with thyroid microsomal
E antibodies but not associated with
thyroid peroxidase antibodies

Explanation
Question A(Correct answ er: B)
Post-partum thyroiditis
14

POST-PARTUM THYROIDITIS ??
· Onset 1-3 months after delivery, associated with thyroid microsomal antibodies and a
family history of autoimmune thyroid disease.??
· Initial presentation is thyrotoxicosis (40%) or hypothyroidism (40%) with 20% of women
having a biphasic illness with hyper- followed by hypo-thyroidism.??
· The disorder is asymptomatic in the majority of women with spontaneous resolution.??
· Need for treatment should be determined by symptoms rather than biochemical indices.
· Treatment for thyrotoxicosis should be with beta-blockers rather than anti-thyroid
agents.
· Thyroxine replacement for hypothyroidism for 6-8 months??
· 3-4% remain hypothyroid; up to 25% recurrence in subsequent pregnancy and up to
30% risk of long-term hypothyroidism in women with thyroid peroxidase antibodies??

Options for Questions 15-15


A In the second trimester B During labour
C During delivery D In the post-partum period
E In the first trimester

Explanation
A 37 year old woman with Von
Willebrand’s disease attends the
Question A(Correct answ er: D)
antenatal clinic at 8 weeks gestation. Her
15
greatest risk of bleeding is: In the first
trimester
Maternal risks associated with bleeding disorders

21
· Bleeding risks - possible improvements with pregnancy. Greatest risk is post-partum as
VIII and vWF levels fall rapidly.??
· Invasive first trimester procedures (CVS) also associated with increased risk of bleeding
· Haemophilia B carriers with low IX levels may need treatment - discuss risks of
treatment

Options for Questions 16-16


Is caused by a mutation in the alpha
A B Typically presents at 2-3 weeks of life
globin gene
Is characterized by intra-vascular red
C cell sickling precipitated by hypoxia, D Is characterized by hypersplenism
acidosis or dehydration
Is not associated with red cell
E
haemolysis

Explanation
Question A(Correct answ er: C)
Sickle cell disease
16

Sickle cell disease and pregnancy


Pathophysiology
• Mutation in beta-globin gene resulting in glutamic acid being replaced by valine in
position 6 of the beta-globin chain. Commonest in West Africans
?1) HbAS - carrier / sickle cell trait
· No detectable clinical abnormality under normal circumstances
· Detectable by Hb electrophoresis
· Not anaemic
· Sickling only under extreme anoxia / dehydration / acidosis??
2) HbSS - sickle cell disease
· Onset at 3-6 months of age, chronic haemolytic anaemia (HbS has lower O2 affinity
therefore anaemia usually asymptomatic).
· Intra-vascular sickling precipitated by hypoxia, dehydration, acidosis, infection and
hypothermia.
· Painful crises - organ infarction, auto-spleenectomy, pulmonary embolism, reduced
renal capacity to concentrate urine, haematuria??
3) HbSC
· Milder variant of HbSS - normal Hb concentration, risk of massive sickling crises in
pregnancy especially in puerparium??
4) HbS- beta-thalassaemia - similar clinical disease to HbSS.??

Options for Questions 17-17


A Sudden onset of severe anaemia B A fall in Hb concentration by over 10 g/L

22
within 24 hours
A fall in Hb by more than 10% from A fall in Hb by more than 30% from
C D
baseline levels over 24 hours baseline levels
A fall in Hb by more than 15% from
E
baseline levels over 7 days

Explanation
A 26 year old woman with sickle cell
disease presents to the assessment unit
Question A(Correct answ er: D)
at 15 weeks gestation because she is
17
feeling increasingly unwell. Acute
anaemic crisis is defined as

MATERNAL RISKS??
1) Complications of sickle cell disease are more common in pregnancy and crises occur in up
to 35% of women. Acute anaemic crisis (fall in Hb by more than 30% below baseline) in 3%??
2) The risk of pre-eclampsia is increased (14%). Eclampsia in ~1%??
3) Increased risk of venous thrombo-embolism??
4) Increased risk of bone marrow embolism??
5) Increased risk of sepsis - UTI / pyelonephritis, pneumonia and puerperal sepsis

Options for Questions 18-18


A Her stated ethnic background B Family origin questionnaire
A detailed family history of
C Her booking haemoglobin concentration D
haemoglobinopathy
E Routine testing of FBC samples

Explanation
A healthy 25 year old woman attends the
Questionantenatal clinic at 10 weeks gestation. A(Correct answ er: B)
18 Her risk of being a carrier of sickle cell
disease should be assessed using

Pre-conception management
• Pre-conception counselling and carrier testing should be available to all women who
are identified as being at higher risk of haemoglobinopathies, using the Family Origin
Questionnaire from the NHS Antenatal and Newborn Screening Programme
• Should be undertaken by a clinician who is able to discuss effects of pregnancy on
sickle cell disease, effect of sickle cell disease on pregnancy and optimise the woman’s
health.
• The woman’s reproductive intentions and use of contraception should be discussed
at each contact with the sickle care team.
23
Options for Questions 19-19
At 38 weeks by caesarean section if no
A At 39 weeks by caesarean section B
spontaneous labour
C At 38 weeks by planned vaginal birth D At 37 weeks by planned vaginal birth
E At 40 weeks by planned vaginal birth

Explanation
A 23 year old woman with sickle cell
disease attends the antenatal clinic at 34
Questionweeks gestation. Her Hb is 79 g/L with A(Correct answ er: C)
19 normal liver and renal function tests.
Fetal growth scan is normal. Delivery
should be
Intra-partum care

• Offer elective delivery after 38+0 weeks of gestation in an obstetric unit that is able to
manage complications of sickle cell disease. Sickle cell disease is not an indication for
caesarean section
• Cross-match if there are atypical antibodies, otherwise group & save
• MDT care including senior midwife, senior obstetrician, anaesthetist and
haematologist
• In women who have hip replacements (because of avascular necrosis) it is important
to discuss suitable positions for delivery.
• Continuous electronic fetal monitoring as increased risk of fetal distress
• Keep woman warm and well hydrated
• There is an increased frequency of sickle cell crisis and acute chest syndrome during
labour.
• There is an increased risk of painful crisis with prolonged labour usually due to
dehydration.
• Avoid the use of pethidine. Regional analgesia is recommended for caesarean
section.

Options for Questions 20-20


Combined oral contraceptive pills and
Progestogen-only pills can be used but
progestogen-only pills can be used but
A depo-medroxyprogesterone acetate is B
Cu IUCD is not recommended (UKMEC
not recommended (UKMEC 3)
3)
Depo-medroxyprogesterone acetate
Combined oral contraceptive pills,
and progestogen-only pills can be used
C progestogen-only pills and Cu IUCD can D
but combined oral contraceptive pills are
all be used (UKMEC 1 or 2)
not recommended (UKMEC 3)

24
Only progestogen-only contraceptives
E
should be offered

Explanation
A 23 year old woman with sickle cell
disease has an emergency caesarean
section at 36 weeks gestation for non-
Question A(Correct answ er: C)
progressive first stage of labour. Her BMI
20
is 22 kg/m2 and she has no other risk
factors. She wishes to discuss options
for contraception before discharge.
Post-natal care
Maternal
• Observe for evidence of sickle cell crisis
• Maintain SO2 above 94% and adequate hydration based on fluid balance until
discharge.
• TEDS plus LMWH while in hospital and for 7 days post-discharge following vaginal
delivery or for a period of 6 weeks following caesarean section.
• Progestogen-only contraceptives are safe and effective (UKMEC 1)
• Oestrogen-containing contraceptives and copper IUCD should be used as second-
line agents (UKMEC 2)

INFERTILITY AND ENDOCRINOLOGY


Options for Questions 1-1
Treatment can proceed but it will not be
A B Further treatment is not recommended
funded by the NHS
Treatment can proceed and should be Treatment can continue until the woman
C D
funded by the NHS is aged 42 years
Treatment can proceed on a case-by-
E case basis depending on the woman’s
general practitioner

Explanation
A 39 year old woman and her 40 year old
partner are undergoing IVF because of
unexplained sub-fertility. The woman has
undergone one episode of ovarian
Question A(Correct answ er: C)
stimulation and there are 6 embryos.
1
Two embryos are replaced but there is
no pregnancy. The woman attends 6
months later for further treatment at
which point she is aged 40 years.

25
Access to IVF
• A full cycle of IVF treatment comprises 1 episode of ovarian stimulation and the
transfer of any resultant fresh and frozen embryo(s).
• In women aged under 40 years who have not conceived after 2 years of regular
unprotected intercourse or 12 cycles of artificial insemination (where 6 or more are by
intrauterine insemination), offer 3 full cycles of IVF. If the woman reaches the age of 40
during treatment, complete the current full cycle but do not offer further full cycles.

Options for Questions 2-2


A baseline ultrasound scan should be
An ultrasound scan should be
performed 1 week before the start of
A B undertaken on days 7-9 and again on
ovarian stimulation in a GnRH agonist
days 11-14
cycle
An ultrasound scan should be
More intensive monitoring is required in
C undertaken on days 10-12 and again on D
GnRH antagonist controlled cycles
days 16-18
The risk of OHSS is higher in cycles
monitored by ultrasound alone
E
compared to cycles monitored by
ultrasound + serum oestradiol

Explanation
QuestionWith respect to monitoring of ovarian A(Correct answ er: B)
2 response during an IVF cycle

Monitoring of stimulated cycles


• The aim is to reduce the incidence and severity of OHSS, and to optimise the timing
of luteinisation before oocyte retrieval.
• Three ultrasound scans are commonly offered
1. At the start of ovarian stimulation in GnRH agonist-controlled cycle
2. To assess at day seven to nine
3. To determine timing of hCG administration at days 11 to 14.
• The extent of monitoring is reduced in GnRH antagonist controlled cycles.
• There is no significant differences between cycle monitoring using both serum
oestradiol and ultrasound versus ultrasound alone in clinical pregnancy rates and OHSS
rates in normal responders undergoing GnRHa-rFSH during IVF- embryo-transfer.

Options for Questions 3-3


This is usually undertaken using Support can be undertaken using
A B
recombinant LH or GnRH antagonist progesterone or GnRH antagonist
Use of GnRH antagonist reduces the
C Use of HCG reduces the risk of OHSS D
risk of OHSS

26
Support can be undertaken using
E
progesterone or HCG

Explanation
QuestionWith respect to luteal phase support A(Correct answ er: E)
3 during an IVF cycle

Luteal-phase support
• A systematic review has confirmed the effectiveness of routine luteal phase support
after embryo transfer in IVF cycles involving the use of GnRH agonists
• The use of hCG in this situation can aggravate OHSS and progesterone should be
the preparation of choice in high-risk women

Options for Questions 4-4


Embryos should ideally be transferred Bed-rest for 30-45 minutes should be
A when the endometrial thickness is 3-5 B recommended immediately after embryo
mm transfer
Bed-rest for 3-4 hours should be
Embryos are typically transferred after
C recommended immediately after embryo D
2-3 days in culture
transfer
Embryo transfer after 7-10 days in
E
culture may increase pregnancy rates

Explanation
QuestionWith respect to embryo transfer during A(Correct answ er: D)
4 IVF

• Single embryo transfer is increasingly promoted to reduce the rate of multiple births.
• In addition, there is a trend to extend the culture of embryos to day 5 or 6 (blastocyst)
rather than the conventional day 2 or 3 (cleavage) which is thought to improve the
chances of a live full-term singleton birth
• Women should be offered ultrasound-guided embryo transfer because this improves
pregnancy rates
• Replacement of embryos into a uterine cavity with an endometrium of less than 5 mm
thickness is not recommended because unlikely to result in a pregnancy
• Bed rest of more than 20 minutes’ duration following embryo transfer does not
improve the outcome of IVF treatment.

Options for Questions 5-5


In the first full IVF cycle, two top quality In the second full IVF cycle, two top
A B
embryos should be transferred quality embryos should be transferred
C In the third full IVF cycle, three top D Three embryos can be transferred if

27
quality embryos should be transferred ovarian response was poor
In the first and second full IVF cycles,
E only a single top quality embryo should
be transferred

Explanation
A 34 year old woman is undergoing IVF
Questionbecause of male factor infertility. A(Correct answ er: E)
5 Regarding recommendations for embryo
transfer

For women aged 37–39 years


1. In the first and second full IVF cycles use single embryo transfer if there are 1 or more top-
quality embryos. Consider double embryo transfer if there are no top-quality embryos.
2. In the third full IVF cycle transfer no more than 2 embryos.

Options for Questions 6-6


The most likely diagnosis is Intra-uterine insemination is the
A B
hyperandrogenaemia recommended first line treatment
Weight loss is the recommended first The woman’s blood should be sent for
C D
line treatment karyotype
The woman has a WHO Group I
E
ovulation disorder

Explanation
A 34 year old woman and her 35 year old
partner have been referred because of
primary infertility. The woman has
irregular menstrual cycles every 40-120
Questiondays and her BMI is 32 kg/m2. Pelvic A(Correct answ er: C)
6 examination and pelvic ultrasound scan
are normal. Endocrine profile shows a
serum testosterone of 3.2 nmol/L but the
rest of the tests are normal. Her partner’s
semen analysis is normal.

Meets criteria for PCOC


• Oligo-ovulation and/or anovulation
• Hyperandrogenism (clinical and/or biochemical)
• Polycystic ovaries on ultrasound scan (12 or more small antral follicles present)
Options for treatment include
Weight loss & life-style modifications
• Often the first line treatment for obese PCOS patients

28
• Obesity is associated with increased insulin resistance and an exacerbation of PCOS
• There is no significant differences in the number of clinical pregnancies when
comparing lifestyle modification (low calorie diet plus exercise) with clomifene citrate
alone, metformin alone, or clomifene citrate plus metformin
• Women with WHO Group II anovulatory infertility who have a BMI ≥ 30 should be
advised to lose weight. This alone may restore ovulation, improve response to ovulation
induction agents, and have a positive impact on pregnancy outcomes

Options for Questions 7-7


Clomifene citrate is associated with a Treatment with metformin is associated
A higher clinical pregnancy rate compared B with a higher clinical pregnancy rate that
to treatment by lifestyle modification treatment by lifestyle modification
Treatment with metformin + clomifene
Clomifene citrate is the recommended
citrate is associated with a higher
C D first line medical treatment at a dose of
clinical pregnancy rate than treatment
50 mg from days 2-6 of the cycle
by lifestyle modification
Treatment with clomifene citrate is
E associated with a 1% multiple
pregnancy rate

Explanation
A 34 year old woman and her 35 year old
partner have been referred because of
primary infertility. The woman has
irregular menstrual cycles every 40-120
Questiondays and her BMI is 32 kg/m2. Pelvic A(Correct answ er: D)
7 examination and pelvic ultrasound scan
are normal. Endocrine profile shows a
serum testosterone of 3.2 nmol/L but the
rest of the tests are normal. Her partner’s
semen analysis is normal.

PCOS: Options for treatment include


Weight loss & life-style modifications
• Often the first line treatment for obese PCOS patients
• Obesity is associated with increased insulin resistance and an exacerbation of PCOS
• There is no significant differences in the number of clinical pregnancies when
comparing lifestyle modification (low calorie diet plus exercise) with clomifene citrate
alone, metformin alone, or clomifene citrate plus metformin
Medical treatment
• First line ovarian stimulation treatment for women with PCOS
• Medical treatment of anovulatory infertility due to PCOS is usually with clomifene
citrate and/or metformin (unlicensed)

29
• Clomifene is taken as a 50mg single daily dose for 5 days from early in the menstrual
cycle
• If ovulation is not achieved at this dose then in subsequent cycles the dose is
increased up to 150 mg
• If no ovulation occurs at doses of 100–150 mg daily then the term ‘clomifene
resistance’ is used.
• Clomifene is associated with a multiple pregnancy rate of ~10%

Options for Questions 8-8


Letrozole is associated with a higher risk The risk of adverse pregnancy outcome
A of multiple pregnancy compared to B is higher with clomifene compared to
clomifene citrate clomifene + metformin
There is no significant difference in the
The risk of adverse pregnancy outcome
number of multiple pregnancies when
C is lower with metformin compared to D
metformin is compared to metformin +
clomifene
clomifene
The risk of adverse pregnancy outcome
E is higher with letrozole compared to
clomifene citrate

Explanation
With respect to the risk of adverse
Questionpregnancy outcome following medical A(Correct answ er: D)
8 induction of ovulation in women with
PCOS

Adverse pregnancy outcomes


• There are no significant differences in adverse pregnancy outcomes between
metformin and clomifene citrate or metformin plus clomifene citrate
• There are no significant differences in the number of multiple pregnancies when
comparing metformin, metformin plus clomifene citrate, letrozole or rFSH to clomifene
citrate alone
• There is no significant difference in the number of multiple pregnancies when
comparing metformin to metformin plus clomifene citrate

Options for Questions 9-9


A Hyper-prolactinaemia B PCOS
C Premature ovarian failure D Kallmann’s syndrome
E Anorexia nervosa

Explanation

QuestionWhich one is a WHO Group III ovulation A(Correct answ er: C)

30
9 disorder?

WHO Group III Ovulation Disorders


Group III ovulation disorders are caused by ovarian failure.
Around 5% of women with ovulation disorders have a group III ovulation disorder

Options for Questions 10-10


Raised serum prolactin levels are
Hyper-prolactinaemia occurs in about
A B associated with low conception rates in
1% of infertile but ovulatory women
ovulatory women with infertility
Serum prolactin levels should be Serum prolactin levels should be
C measured in all women referred with D measured in women with sub-fertility
sub-fertility and oligomenorrhoea
Serum prolactin levels should be
E
measured on days 3-5 of the cycle

Explanation
QuestionWith respect to the measurement of A(Correct answ er: D)
10 prolactin levels in women with infertility

Prolactin measurement
• The incidence of hyperprolactinaemia in infertile but ovulatory women is 3.8% -
11.5%
• There is no significant association between prolactin, progesterone levels and
cumulative conception rates in ovulatory women
• Estimation of prolactin levels should be reserved for women with symptoms of an
ovulatory disorder, galactorrhoea or a pituitary tumour.

Options for Questions 11-11


The results of tubal patency tests should
Tubal factors account for about 15% of
A B be known before an assessment of
the causes of infertility in women
ovulation is undertaken
The results of tubal patency tests should
Tubal factors account for about 25% of
C be known before semen analysis is D
the causes of infertility in women
undertaken
Women with irregular menstrual periods
E
are at increased risk of tubal disease

Explanation
QuestionWith respect to tubal factors causing A(Correct answ er: A)
11 infertility

31
Assessing tubal damage
• Tubal factors account for 14% of the causes of subfertility in women
• The results of semen analysis and assessment of ovulation should be known before
a test for tubal patency is performed.

Options for Questions 12-12


If the tubes are patent on HSG, there is If the tubes are patent on HSG, there is
A a 99% chance they will be patent at B a 50% chance they will be obstructed at
laparoscopy & dye test laparoscopy & dye test
Using laparoscopy and dye as the gold Using laparoscopy and dye as the gold
C standard, HSG has a sensitivity of 65% D standard, HSG has a specificity of 95%
in detecting tubal obstruction in detecting tubal obstruction
If HSG indicates that the tubes are
obstructed, there is a 95% chance that
E
the tubes will be obstructed at
laparoscopy and dye

Explanation
A 34 year old woman and her 37 year old
Questionpartner are being investigated for primary A(Correct answ er: C)
12 infertility. The woman has a hysteron-
salpingogram (HSG).

HSG Vs Lap & dye


• Among women whose tubes were found to be patent using HSG, 18% were found to
have tubal obstruction or peritubal adhesions using laparoscopy and a further 34% were
found to have endometriosis and/or fibroids
• However, the detection and treatment of pathology missed by HSG did not increase
live birth rates
• Using laparoscopy as gold standard, HSG as a test for tubal obstruction has a
sensitivity of 0.65 and specificity of 0.83
• When HSG suggests the presence of tubal obstruction this will be confirmed by
laparoscopy in only 38% of women. Thus, HSG is a not a reliable indicator of tubal
occlusion
• When HSG suggests that the tubes are patent, this will be confirmed at laparoscopy
in 94% of women, and so HSG is a reliable indicator of tubal patency
• Women who are not known to have comorbidities (such as pelvic inflammatory
disease, previous ectopic pregnancy or endometriosis) should be offered HSG to
screen for tubal occlusion because this is a reliable test for ruling out tubal occlusion, is
less invasive and makes better use of resources than laparoscopy

Options for Questions 13-13

32
Chlamydia antibody titres are more There is a positive correlation between
A useful in diagnosing tubal obstruction B chlamydia antibody levels and the
than hystero-salpingography severity of tubal damage
Chlamydia antibody titres are superior to There is no correlation between
C laparoscopy & dye test in identifying D chlamydia antibody levels and live birth
tubal disease rates
Elevated chlamydia antibody titres are
E of no significance in women below the
age of 25 years

Explanation
QuestionIn women undergoing investigations for A(Correct answ er: B)
13 infertility

Chlamydia antibodies
· The discriminative capacity of chlamydial antibody testing is comparable to that of
HSG in the diagnosis of tubal pathology
· Elevated titres of chlamydial antibodies are significantly associated with tubal
disease
· The titre of chlamydial antibodies has also been reported to be more accurate in
predicting severe tubal pathology than unspecified tuboperitoneal abnormalities
· Chlamydial antibody levels are quantitatively related to severity and extent of
tubal pelvic damage. An elevated chlamydial antibody titre is significantly associated
with poor live birth rates, but not pregnancy rates
· However, the chance of conception with or without tubal surgery is related to the
degree of damage found at laparoscopy, with the chlamydial antibody titre adding no
further diagnostic value

Options for Questions 14-14


Reduction of the woman’s viral load
A Timed unprotected intercourse B followed by regular unprotected
intercourse
Reduction of the woman’s viral load
Intra-uterine insemination using donor
C followed by timed unprotected D
sperm
intercourse
Intra-uterine insemination using
E
partner’s sperm

Explanation
A 23 year old woman has just started a
Questionnew relationship with a 24 year old man. A(Correct answ er: E)
14 The woman has disclosed that she is
HIV positive but the couple is considering

33
starting a family. Which one is the most
appropriate option?

HIV: Female to male transmission


• Use of assisted reproductive techniques (ART), such as IUI or IVF

Options for Questions 15-15


A 0.1-0.5% B 1.0-3.0%
C 5.0-9.5% D 11-12.5%
E 14-16.5%

Explanation
QuestionThe prevalence of chlamydia trachomatis A(Correct answ er: B)
15 in sub-fertile women in the UK is

Screening for Chlamydia trachomatis


• Chlamydia trachomatis is present in 11% of the sexually active population aged ≤ 19
years
• It is a major cause of PID, leading to chronic abdominal pain, ectopic pregnancy and
tubal factor infertility
• The prevalence of C. trachomatis in sub-fertile women in the UK is 1.9%
• Uterine instrumentation may reactivate or introduce upper tract dissemination of
endocervical chlamydial infection, resulting in iatrogenic PID
• Clinical pelvic infection following HSG has been reported in up to 4% of cases and in
10% of patients with tubal disease
• Doxycycline or azithromycin are effective prophylaxis and treatment for chlamydia.
Prophylaxis should be considered in women undergoing HSG

Options for Questions 16-16


A Imipramine B Neostigmine
C Testosterone implant D Per-cutaneous sperm aspiration and IVF
E Sperm retrieval by masturbation and IUI

Explanation
A 34 year old woman and her 43 year old
partner have been referred for fertility
Questiontreatment. The male has a neurological A(Correct answ er: D)
16 disorder resulting in anejaculation. All
other investigations are normal. Which
one is the most appropriate treatment?

Anejaculation

34
• Medical treatment of anejaculation has included the use of alpha-agonistic drugs
such as imipramine, pseudoephedrine or parasympathomimetic and neostigmine.
Treatment with alpha-agonistics has significantly lower success rates than treatment
with parasympathetic drugs in the reversal of anejaculation (19% with alpha-agonists
versus 51% with parasympathomimetics)
• Medical treatment is associated with considerable side-effects such as headache,
nausea and vomiting and is not recommended as first line treatment
• Sperm retrieval using invasive procedures for IVF / ICSI should be used in cases of
ejaculatory failure

Options for Questions 17-17


A Retrograde ejaculation B Anejaculation
C Primary testicular failure D Hypogonadotrophic hypogonadism
E Previous vasectomy

Explanation
A 43 year old man is undergoing
investigation for infertility. Semen
Questionanalysis shows azoospermia. Endocrine A(Correct answ er: C)
17 profile shows elevated FSH levels with
low testosterone levels. Which one is a
possible diagnosis?

Causes of azoospermia
1) Hypothalamic-pituitary failure: (Hypogonadotrophic hypogonadism). Accounts for < 1% of
male factor fertility. It results in a deficiency of luteinizing hormone (LH) and follicle-stimulating
hormone (FSH), which is associated with failure of spermatogenesis and testosterone
secretion.
2) Primary testicular failure (non-obstructive azoospermia): The diagnosis is based on
reduction in testicular size and elevation of serum FSH levels. It is the most common cause of
male infertility due to oligozoospermia. May be due to:
• Cryptorchidism
• Torsion
• Trauma
• Orchitis
• Chromosome disorders (Klinefelter’s syndrome, Y-chromosome microdeletions)
• Systemic disease
• Radiotherapy or chemotherapy
• Idiopathic (66%)
There is no effective treatment. Men undergoing treatments that cause infertility should be
offered the opportunity to cryopreserve semen.

Options for Questions 18-18

35
A Retrograde ejaculation B Congenital absence of the vas deferens
C Primary testicular failure D Klinfelter’s syndrome
E Previous chemotherapy

Explanation
A 23 year old man is undergoing
investigation for infertility. Semen
analysis shows azoospermia. On
Question A(Correct answ er: B)
examination, testicular size is normal.
18
Endocrine profile shows normal FSH and
testosterone levels. Which one is the
most likely diagnosis?

Obstruction of the genital tract (obstructive azoospermia): Prevalence < 2%


Diagnosis is based on normal testis size and normal serum FSH levels.
Causes include
Congenital bilateral absence of vas deferens (associated with cystic fibrosis mutations or renal
tract abnormality).

Options for Questions 19-19


Sperm concentration below 15 million Total spermatozoa count = 10 million
A B
per ml per ejaculate
Percentage of spermatozoa with Percentage of normal spermatozoa =
C D
progressive motility of 20% 10%
E Percentage of live spermatozoa = 26%

Explanation
QuestionAsthenozoospermia is a suitable A(Correct answ er: C)
19 description for

Definition of semen abnormalities


Asthenozoospermia
• Percentage of progressively motile (PR) spermatozoa below the lower reference limit
WHO reference values for semen analysis
 Semen volume: ≥ 1.5 ml
 pH: ≥ 7.2
 Sperm concentration: ≥ 15 million spermatozoa per ml
 Total sperm number: ≥ 39 million spermatozoa per ejaculate
 Total motility (percentage of progressive motility and non-progressive motility): ≥ 40%
motile or ≥ 32% with progressive motility
 Vitality: ≥ 58% live spermatozoa
 Sperm morphology (percentage of normal forms): ≥ 4%

36
Options for Questions 20-20
Typically presents 3-9 days after HCG Typically presents 3-9 days after
A B
administration embryo replacement
Typically presents 12-17 days after Typically presents 12-17 days after
C D
GnRH administration HCG administration
Typically presents 12-17 days after
E
embryo replacement

Explanation
Question A(Correct answ er: D)
Late-onset OHSS
20
Late onset OHSS
· 12-17 days after HCG administration.
· Related to pregnancy and the number of gestation sacs and reflects endogenous HCG
production. More likely to be severe and last longer than early-onset OHSS??
· Risk can be reduced by using progesterone rather than HCG for luteal phase support??
· Differential diagnoses include ovarian cyst torsion / haemorrhage, pelvic infection, intra-
abdominal haemorrhage, ectopic pregnancy and other acute abdomen??
· Women undergoing assisted conception should be provided with verbal and written
information about the risk of OHSS, symptoms and have 24h access to a clinician with the
expertise to diagnose and manage OHSS???

Hypertensive disorders of pregnancy


Options for Questions 1-1
Women with BP persistently over
A Women requiring ventilation B
170/110 mmHg despite treatment
C Women with severe oliguria D Women with HELLP syndrome
Women with abnormal neurological
E
signs

Explanation
With respect to the care of women with
Question A(Correct answ er: A)
pre-eclampsia, which women require
1
level 3 critical care?

Indications for level 1 critical care

37
• Pre-eclampsia with mild or moderate hypertension
• Ongoing conservative antenatal management of severe preterm hypertension
• Step-down treatment after the birth
Indications for level 2 critical care
Severe pre-eclampsia with:
• Eclampsia
• HELLP syndrome
• Haemorrhage
• Hyperkalaemia
• Severe oliguria
• Coagulation support
• Intravenous antihypertensive treatment
• Initial stabilisation of severe hypertension
• Evidence of cardiac failure
• Abnormal neurology
• Step-down from level 3 care
Indications for level 3 critical care
• Severe pre-eclampsia and needing ventilation

Options for Questions 2-2


A Stop MgSO4 and treat with diazepam B Continue MgSO4 and add diazepam
C Continue MgSO4 and add phenytoin D Bolus of 4g MgSO4 over 5 minutes
E Bolus of 2g MgSO4 over 15 minutes

Explanation
A healthy 35 year ole woman is admitted
with severe pre-eclampsia at 38 weeks
gestation. Induction of labour is initiated
Questionand she is started on intravenous A(Correct answ er: D)
2 magnesium sulphate. Her blood pressure
is well controlled and renal function is
normal. At 8cm dilatation, she suffers a
grand mal seizure.

MgSO4 Dose
• Loading dose of 4 g iv over 5 minutes, followed by an infusion of 1 g/hour maintained
for 24 h
• Treat recurrent seizures with a further dose of 2–4 g given over 5 minutes.
• Do not use diazepam, phenytoin as an alternative to magnesium sulphate

Options for Questions 3-3


Significantly better than placebo in Significantly better than placebo in
A B
reducing maternal mortality reducing serious maternal morbidity

38
Significantly better than placebo in Significantly better than placebo in
C D
reducing risk of placental abruption preventing eclampsia
Significantly better than placebo in
E
reducing the risk of stillbirth

Explanation
A 37 year old woman is admitted with
Questionsevere pre-eclampsia at 39 weeks A(Correct answ er: D)
3 gestation. Treatment with magnesium
sulphate has been shown to

Rationale
 In groups of women with mild – moderate or severe pre-eclampsia, magnesium sulphate is
statistically significantly better than none/placebo in preventing eclampsia but does not
significantly alter the risk of maternal death, serious maternal morbidity, pulmonary oedema,
placental abruption, kidney dialysis, stillbirth or neonatal death.

Options for Questions 4-4


Plasma volume expansion is not Plasma volume expansion
A B
indicated recommended with 250 ml colloid
Plasma volume expansion Plasma volume expansion
C D
recommended with 250 ml crystalloid recommended with 500 ml colloid
Plasma volume expansion
E
recommended with 500 ml crystalloid

Explanation
A 34 year old woman has an emergency
caesarean section at 34 weeks gestation
Questionbecause of pre-eclampsia. BP is A(Correct answ er: A)
4 persistently over 170/110 mmHg despite
intravenous labetalol. Treatment with iv
hydralazine is indicated.

Anti-hypertensive therapy
• The route of administration would depend on maternal condition, but where feasible
oral administration should be preferred to intravenous administration because it is likely
to be cost effective.
• Severe hypertension should be treated with either labetalol (oral or iv), hydralazine
(iv) or nifedipine (oral)
• Response to treatment should be monitored to ensure that BP is well controlled and
to identify adverse effects on the woman and fetus

39
• Up to 500 ml crystalloid fluid may be administered before or at the same time as the
first dose of hydralazine in the antenatal period to reduce the risk of hypotension.
Otherwise, volume expansion is not recommended in women with pre-eclampsia.
• Systolic BP should be maintained below 150 mmHg and diastolic BP between 80 and
100 mmHg.

Options for Questions 5-5


A Every hour B Every 2 hours
C Every 30 minutes D Every 15 minutes
E Every 5 minutes

Explanation
A healthy 37 year old woman presents in
spontaneous labour at 39 weeks
Questiongestation. Her BP is 152/92 mmHg and A(Correct answ er: A)
5 there is 2+ proteinuria. Her cervix is 5cm
dilated with intact membranes. Her BP
should be measured

Intra-partum care?
BP monitoring
During labour, measure blood pressure:
• Hourly in women with mild or moderate hypertension
• Continually in women with severe hypertension
• Continue use of antenatal antihypertensive treatment

Options for Questions 6-6


Monitor BP every 4-6 hours and test for
A B Monitor BP and test for proteinuria daily
proteinuria daily
Monitor BP and test for proteinuria at
C D Monitor BP every 15 minutes
least twice a week
Monitor BP and test for proteinuria once
E
a week

Explanation
A low risk 33 year old woman attends the
assessment unit at 34 weeks gestation.
The woman’s BP is 155/104 mmHg and
Question A(Correct answ er: C)
there is no proteinuria. She has no
6
symptoms and the fundal height is on the
50th centile on a customized growth
chart. Which one is the most appropriate

40
management?

Moderate hypertension (150-159 / 100 – 109)


• Out-patient management
• Treat with anti-hypertensive: oral labetalol is first line; methyldopa and nifedipine may
be used after considering side-effects. Keep systolic BP < 150 mmHg and diastolic BP
between 80 – 100 mmHg
• Monitor BP at least twice a week
• Test for proteinuria at every visit
• FBC, renal (including electrolytes) & liver function tests. Do not repeat if no
proteinuria at subsequent visits

Options for Questions 7-7


Systolic 130-140 mmHg; diastolic 80-90 Systolic below 150 mmHg, diastolic 80-
A B
mmHg 100 mmHg
Systolic below 140 mmHg, diastolic 80- Systolic below 140 mmHg, diastolic
C D
100 mmHg below 90 mmHg
Systolic 120-130 mmHg, diastolic below
E
80 mmHg

Explanation
A low risk 33 year old woman attends the
assessment unit at 34 weeks gestation.
The woman’s BP is 155/104 mmHg and
Questionthere is no proteinuria. She has no A(Correct answ er: B)
7 symptoms and the fundal height is on the
50th centile on a customized growth
chart. All blood tests are normal. The
target for BP control should be

Moderate hypertension (150-159 / 100 – 109)


• Out-patient management
• Treat with anti-hypertensive: oral labetalol is first line; methyldopa and nifedipine may
be used after considering side-effects. Keep systolic BP < 150 mmHg and diastolic BP
between 80 – 100 mmHg
• Monitor BP at least twice a week
• Test for proteinuria at every visit
• FBC, renal (including electrolytes) & liver function tests. Do not repeat if no
proteinuria at subsequent visits

Options for Questions 8-8


A Admit until delivery B Admit until BP below 159/109 mmHg

41
C Admit until BP below 140/90 mmHg D Admit until pre-eclampsia is excluded
Treat as out-patient with daily
E
assessment unit visits

Explanation
A low risk 33 year old woman attends the
assessment unit at 30 weeks gestation.
The woman’s BP is 165/104 mmHg and
Question A(Correct answ er: B)
there is no proteinuria. She has no
8
symptoms, feels good fetal movements
and the fundal height is on the 50th
centile on a customized growth chart.

Severe hypertension (≥ 160/110 mmHg)


• In-patient treatment until BP ≤ 159/109 mmHg
• Treat with anti-hypertensive: oral labetalol is first line; methyldopa and nifedipine may
be used after considering side-effects . Keep systolic BP < 150 mmHg and diastolic BP
between 80 – 100 mmHg
• Monitor BP at least every 6 hours
• Test for proteinuria daily
• FBC, renal (including electrolytes) & liver function tests at presentation and then
weekly
• In women receiving outpatient care after severe hypertension has been effectively
controlled in hospital, measure blood pressure and test urine twice weekly and carry out
weekly blood tests.

Options for Questions 9-9


Peak post-natal blood pressure occurs Peak post-natal blood pressure occurs
A B
in the first 24h after birth 3-5 days after birth
Peak post-natal blood pressure occurs Blood pressure falls to pre-pregnancy
C D
5-7 days after birth levels within 24h of birth
Peak post-natal blood pressure occurs
E
7-0 days after birth

Explanation
QuestionWith respect to post-natal care in women A(Correct answ er: B)
9 with gestational hypertension

Post-natal care
BP monitoring
• Peak blood pressure in the postnatal period occurs 3–5 days after birth
• Additional BP monitoring is appropriate if treatment is altered.
• BP should be monitored:

42
1.Daily for the first 2 days after birth
2.At least once between day 3 and day 5 after birth
3.As clinically indicated if antihypertensive treatment is changed after birth.

Options for Questions 10-10


Monitor BP at least once between days
A B Monitor BP daily from days 3-5
3 and 5
C Monitor BP daily from days 3-7 D Monitor BP weekly for the next 2 weeks
E Monitor BP weekly for the next 6 weeks

Explanation
A 40 year old woman has labour induced
at 39 weeks gestation because of mild
Questiongestational hypertension. She has a A(Correct answ er: A)
10 ventouse delivery and her post-natal BP
is 145/95 mmHg. She is discharged 24h
after birth and is not on any medication.

Post-natal care
BP monitoring
• Peak blood pressure in the postnatal period occurs 3–5 days after birth
• Additional BP monitoring is appropriate if treatment is altered.
• BP should be monitored:
1 Daily for the first 2 days after birth
2 At least once between day 3 and day 5 after birth
3 As clinically indicated if antihypertensive treatment is changed after birth.

Options for Questions 11-11


Increase dose to 200 mg three times a
A Stop anti-hypertensive drugs B
day
C Omit next dose of labetalol D Continue current treatment
E Change labetalol to coracten

Explanation
A 34 year old woman has a spontaneous
vaginal delivery following induction of
labour at 39 weeks gestation because of
Questionmoderate gestational hypertension. She A(Correct answ er: D)
11 is on labetalol 200 mg twice a day. On
day 2 post-partum, you are asked to
review her. Her BP has been 130-
135/80-88 mmHg.

43
Antihypertensive therapy
• Methyldopa has a well-recognised association with clinical depression and should be
avoided in the postnatal period
• Continue use of antenatal antihypertensive treatment
• Consider reducing antihypertensive treatment if their blood pressure falls below
140/90 mmHg
• Peak BP is on day 3-5 post-partum so appropriate to continue treatment on day 2
• Reduce antihypertensive treatment if their blood pressure falls below 130/80 mmHg.
• If a woman has taken methyldopa to treat gestational hypertension, stop within 2
days of birth.
• For women not on antihypertensive treatment, start treatment if BP > 149/99 mmHg.

Options for Questions 12-12


Labetalol is associated with low Atenolol is associated with low birth
A B
placental weight weight
There is a recognized association There is a recognized association
C between labetalol and neonatal D between methyldopa and neonatal
hypoglycaemia jaundice
ACE inhibitors can safely be used after
E
the second trimester

Explanation
With respect to the potential side-effects
Question A(Correct answ er: B)
of anti-hypertensive drugs used during
12
pregnancy and the post-natal period

Fetal effects of anti-hypertensive drugs


• ACE inhibitors associated with increased rates of congenital malformations, IUGR,
hypoglycaemia, kidney disease and preterm birth.
• Angiotensin II receptor blockers associated with increased rates of congenital
malformations. Chlorothiazides may carry the risk of congenital abnormality, neonatal
thrombocytopenia, hypoglycaemia and hypovolaemia.
• Women taking these drugs should be informed of the risks and other
antihypertensive treatment should be discussed if they are planning pregnancy.
• Stop antihypertensive treatment in women taking ACE inhibitors or Angiotensin II
receptor blockers if they become pregnant (preferably within 2 working days of
notification of pregnancy and offer alternatives.
• There is no evidence that other common antihypertensives in current use are
teratogenic.
• Methyldopa and labetalol associated with mild neonatal hypotension in the first 24-
48h of life.
• Atenolol associated with low birth weight and low placental weigh

44
Options for Questions 13-13
A After 37 weeks B At 37 weeks
C After a course of corticosteroids D Within the next 24-48 hours
E At 40 weeks

Explanation
A 34 year old woman with a history of
chronic hypertension attends the
antenatal clinic at 34 weeks gestation.
QuestionHer BP is 148/92 mmHg on labetalol 200 A(Correct answ er: A)
13 mg four times a day and there is no
proteinuria. Fetal growth scan and
umbilical artery Dopplers are normal.
Delivery should be planned

Timing of delivery
• If blood pressure < 160/110 mmHg (with or without treatment), delivery should not be
offered before 37 weeks.
• For women with BP < 160/110 mmHg (with or without treatment) after 37 weeks,
timing of delivery, and maternal and fetal indications should be agreed between the
woman and the senior obstetrician.
• Delivery is indicated in women with refractory severe chronic hypertension, after a
course of corticosteroids (if required)

Options for Questions 14-14


Women with a BMI over 30 kg/m2 and
A Primigravidas over the age of 35 years B
inter-pregnancy interval over 10 years
Women whose mother and sister have Women aged over 40 years with a BMI
C D
both had pre-eclampsia over 30 kg/m2
Women aged over 40 years with an
E
inter-pregnancy interval over 10 years

Explanation
Aspirin to reduce the risk of pre-
Question A(Correct answ er: E)
eclampsia is recommended in which
14
women?

Risk factors for pre-eclampsia: Moderate risk


• First pregnancy
• Age ≥40 years
• Inter-pregnancy interval > 10 years
• BMI ≥ 35 kg/m2 at booking

45
• Family history of pre-eclampsia
• Multiple pregnancy
?Recommended intervention:
If ≥ 2 moderate risk factors, Aspirin 75 mg of daily from 12 weeks until delivery.

Options for Questions 15-15


Methylene-tetrahydrofolate reductase
A B Hyper-homocysteinaemia
gene mutation
C Anti-cardiolipin antibodies D Lupus anticoagulant
E Prothrombin gene mutation

Explanation
QuestionWhich thrombophilia is not associated A(Correct answ er: D)
15 with an increased risk of pre-eclampsia?

Thrombophilia and risk of pre-eclampsia


• Pregnant women with hyperhomocysteinaemia are more likely to develop pre-
eclampsia than women with other thrombophilias
• Both anticardiolipin antibodies and prothrombin heterozygosity are statistically
significantly associated with pre-eclampsia
• Methylenetetrahydrofolate reductase gene mutation homozygous is associated with
pre-eclampsia although the risk is lower than for the thrombophilias above
Antithrombin III, protein C or protein S deficiencies, lupus anticoagulants and acquired
activated protein C resistance (APCR) are not associated with pre-eclampsia.

Options for Questions 16-16


A Trophoblast cells B Uterine vascular endothelium
C Heart D Muscle
E Adipose tissue

Explanation
Which tissue or organ has not been
Question A(Correct answ er: B)
shown to express placental growth
16
factor?
Placental Growth Factor (PlGF)
Member of the VEGF family of proteins
Expressed by trophoblast cells and placental villi and at low levels in heart, lung, muscle and
adipose tissue.
The secretion and biological activity of PlGF is highly regulated through secretion as a homo-
(PlGF:PlGF) or hetero-dimer (PlGF:VEGF-A)

46
Options for Questions 17-17
A 1 in 50 B 1 in 20
C 1 in 15 D 1 in 5
E 1 in 2

Explanation
A 25 year old woman attends for follow-
up 2 weeks after delivery. She developed
pre-eclampsia at 26 weeks gestation
Question A(Correct answ er: E)
resulting in emergency caesarean
17
section at 27 weeks because of severe
pre-eclampsia. Her risk of pre-eclampsia
in a future pregnancy is

Advice and follow-up care at transfer to community care


Tell women who had pre-eclampsia that their risk of developing:
• gestational hypertension in a future pregnancy ranges from about 1 in 8 (13%)
pregnancies to about 1 in 2 (53%) pregnancies
• pre-eclampsia in a future pregnancy is up to about 1 in 6 (16%) pregnancies
• pre-eclampsia in a future pregnancy is about 1 in 4 (25%) pregnancies if their pre-
eclampsia was complicated by severe pre-eclampsia, HELLP syndrome or eclampsia
and led to birth before 34 weeks, and about 1 in 2 (55%) pregnancies if it led to birth
before 28 weeks.

Options for Questions 18-18


Stop captopril immediately but continue
A Recommend termination of pregnancy B
nifedipine
Ensure that captopril is discontinued Ensure that captopril is discontinued
C D
before 6 weeks gestation within 48 h
Ensure that she is seen in the obstetric
E
medical clinic before 10 weeks gestation

Explanation
A 37 year old woman phones the
maternity assessment unit for advice.
She has a history of chronic
hypertension and developed pre-
Question A(Correct answ er: D)
eclampsia at 27 weeks in her last
18
pregnancy which ended 12 months ago.
Hypertension is treated with captopril
and nifedipine. She has been using
contraception but has missed her period

47
and her pregnancy test is positive.

Pre-pregnancy advice
Tell women who take angiotensin-converting enzyme (ACE) inhibitors or angiotensin II
receptor blockers (ARBs):
• that there is an increased risk of congenital abnormalities if these drugs are taken
during pregnancy
• to discuss other antihypertensive treatment with the healthcare professional
responsible for managing their hypertension, if they are planning pregnancy. Stop
antihypertensive treatment in women taking ACE inhibitors or ARBs if they become
pregnant (preferably within 2 working days of notification of pregnancy) and offer
alternatives.

Options for Questions 19-19


The target is below 140/90 mmHg in The target is 120/80 mmHg for women
A women with uncomplicated chronic B with chronic hypertension and end-
hypertension organ damage
The target is lower than 140/90 mmHg The target is below 150/90 mmHg for
C for women with chronic hypertension D women with chronic hypertension and
and end-organ damage end-organ damage
The target is 120-140/80-90 mmHg for
E women with uncomplicated chronic
hypertension

Explanation
QuestionWith respect to targets for treatment of A(Correct answ er: C)
19 hypertension in pregnancy

Treatment of hypertension
In pregnant women with uncomplicated chronic hypertension aim to keep blood pressure less
than 150/100 mmHg.
Do not offer pregnant women with uncomplicated chronic hypertension treatment to lower
diastolic blood pressure below 80 mmHg.
Offer pregnant women with target-organ damage secondary to chronic hypertension (for
example, kidney disease) treatment with the aim of keeping blood pressure lower than 140/90
mmHg.
Offer pregnant women with secondary chronic hypertension referral to a specialist in
hypertensive disorders.
Offer women with chronic hypertension antihypertensive treatment dependent on pre-existing
treatment, side-effect profiles and teratogenicity.

Options for Questions 20-20

48
A 0.1 B 0.2
C 0.4 D 1.2
E 1.5

Explanation
Compared to placebo, the relative risk of
Question A(Correct answ er: C)
eclampsia in women with pre-eclampsia
20
treated with magnesium sulphate is
Eclampsia
Eclampsia was reported in six trials (11,444 women). There was more than a halving in the risk
of eclampsia associated with the use of magnesium sulphate (RR 0.41, 95% confidence
interval (CI) 0.29 to 0.58; risk difference (RD) -0.01, 95% CI -0.02 to -0.01; number needed to
treat (NNTB) 100, 95% CI 50 to 100) rather than placebo or no anticonvulsant.
Lelia Duley et al.Magnesium sulphate and other anticonvulsants for women with pre-eclampsia
Lelia Duley et al. Cochrane Intervention Review 2010.

Oncology
Options for Questions 1-1
One smear test showing borderline
A Two inadequate smears B
nuclear changes in endocervical cells
Three smear test showing mild
C D Two consecutive inadequate smears
dyskaryosis
A third test reported as abnormal at any
E
grade

Explanation
QuestionWhich one is an indication for referral for A(Correct answ er: B)
1 colposcopy?

Standards
Women should be referred for colposcopy after:
• Three consecutive inadequate smears
• Three tests reported as borderline nuclear change in squamous cells in a series,
without the woman being returned to routine recall
• One test reported as borderline nuclear change in endocervical cells
• Three tests reported as abnormal at any grade in a 10 year period
• If they have been treated for CIN and have not been returned to routine recall and a
subsequent test is reported as mild dyskaryosis or worse.

49
Ideally, women should be referred for colposcopy after one test reported as mild dyskaryosis,
but it remains acceptable to recommend a repeat test.
At lease 90% of women should be seen within 8 weeks of referral.

Options for Questions 2-2


She should avoid driving for at least 2 She should abstain from vaginal
A B
weeks intercourse for 7 days
Her periods may temporarily become
C D She should avoid swimming for 6 weeks
heavier and more painful
That diathermy loop excision does not
E
increase the risk of pre-term labour

Explanation
A 35 year old woman has colposcopy
Question A(Correct answ er: C)
and diathermy loop excision for CIN 2.
2
She should be advised that

Patient information and communication


Women should be advised
• To avoid using tampons for 4 weeks following treatment
• To abstain from vaginal intercourse for 4 weeks following treatment
• To avoid swimming for 2 weeks following treatment
• That they may drive following loop excision or local treatment, unless advised
otherwise by the colposcopist
• That other normal activities, including light exercise, may continue
• That, although there are no known health grounds for avoiding travel following
treatment, overseas medical attention for complications arising from the treatment may
not be covered by insurance
• That there may be a temporary change in the menstrual pattern following loop
excision - bleeding may be more sustained, heavier and more painful
• That single conisation, cervical diathermy and loop excision are each associated with
a small but significant increase in the incidence of preterm labour and preterm prelabour
rupture of membranes

Options for Questions 3-3


80% of women referred with low grade
95% of cases should have the specimen
A B dyskaryosis should be treated at the first
removed as a single sample
visit
80% of cases should have the specimen Ectocervical lesions should be excised
C D
removed as a single sample to a depth of at least 5 mm
Only in exceptional circumstances
E should women with CIN be treated at
the first visit

50
Explanation
With respect to excision of cervical intra-
Question A(Correct answ er: C)
epithelial neoplasia, the standards
3
recommend that

Excision
• At least 80% of cases should have the specimen removed as a single sample.
Removing the transformation zone in multiple fragments can make histopathological
assessment difficult. Furthermore if microinvasive disease is present, it may be
impossible to allocate a substage or define completeness of excision in fragmented
excisional specimens
• For ectocervical lesions, excisional techniques should remove tissue to a depth of
greater than 7mm
• Treatment at first visit for low-grade dyskaryosis should be used only in exceptional
cases, and only when audit has identified that CIN 2/3 or cGIN is present in ≥90% of the
excised specimens.

Options for Questions 4-4


Every 6 months for 3 years then routine At 6, 12 and 24 months then routine
A B
recall if all negative recall if all negative
At 6 and 12 months then yearly for 5 At 6 and 12 months then yearly for 9
C D
years years
At 6 and 12 months then yearly for 10
E
years

Explanation
A 45 year old woman has diathermy loop
Questionexcision for CIN 1 which was completely A(Correct answ er: B)
4 excised. Plan for follow-up cytology
should be

Duration of follow up
• Women treated for high grade disease (CIN 2, CIN 3, cGIN) require six and 12 month
follow up cytology and annual cytology for the subsequent nine years at least before
returning to screening at the routine interval. If a woman has not attended for all the
specified cytology for her high risk follow up, she should be allowed to return to routine
screening provided her samples are normal at least 10 years after treatment.
• Women treated for low grade disease require six, 12 and 24 month follow up
cytology. If all results are negative, then women may be returned to screening at the
routine interval

Options for Questions 5-5

51
A No further follow-up B Vault smear at 6 moths
Vault smears at 6 and 12 months then Vault smears at 6 and 12 months then
C D
yearly for 9 years yearly for 10 years
E Vault smears at 6 and 18 months

Explanation
A 49 year old woman has a total
abdominal hysterectomy for uterine
Questionfibroids. Histology shows benign A(Correct answ er: B)
5 fibromyoma and no other abnormalities.
The woman had diathermy loop excision
for CIN 3 8 years ago.

Follow-up after Hysterectomy


• Women who have had a hysterectomy with CIN present are potentially at risk of
developing vaginal intraepithelial neoplasia (VaIN) and invasive vaginal disease. For
women on routine recall and with no CIN in their hysterectomy specimen, no further
vaginal vault cytology is required
• Women not on routine recall, and with no CIN in their hysterectomy specimen, should
have vaginal vault cytology at six months following their hysterectomy

Options for Questions 6-6


Vault smears every 6 months for 10
A No follow-up required B
years
Vault smears at 6 and 12 months then Vault smears at 6 and 12 months then
C D
yearly until 65 years old yearly for 9 years
E Vault smears at 6 and 12 months

Explanation
A 47 year old woman with endometriosis
has a total abdominal hysterectomy and
bilateral salpingo-oophrectomy. She had
Question A(Correct answ er: D)
diathermy loop excision 7 years earlier
6
for CIN 3. Histology shows CIN 2 in the
hysterectomy specimen which is
incompletely excised.
Follow-up after Hysterectomy
• Women who undergo hysterectomy and have incompletely excised CIN (or uncertain
excision) should be followed-up as if their cervix remained in situ.
1. CIN 1: vault cytology at six, 12 and 24 months
2. CIN 2/3: vault cytology at six and 12 months, followed by nine
annual vault cytology samples

52
3. Incompletely excised CIN: follow-up continues to 65 years or until 10
years after surgery (whichever is later)

Options for Questions 7-7


A 1 in 48 B 1 in 72
C 1 in 110 D 1 in 220
E 1 in 500

Explanation
The life-time risk of a woman being
Question A(Correct answ er: A)
diagnosed with ovarian cancer in the UK
7
is

Epidemiology
• Fifth commonest cancer in women in the UK after breast, colorectal, lung and uterus.
• ~6,700 new cases diagnosed yearly with ~ 4,300 deaths. Leading cause of
gynaecological cancer deaths
• The reason for the high mortality rate may be because most women are diagnosed
with advanced ovarian cancer
• The lifetime risk of women being diagnosed with ovarian cancer is 1 in 48
• ~90% of ovarian cancer in the UK diagnosed in women aged 45 years and above

Options for Questions 8-8


A Stage IIb B Stage IIc
C Stage IIIa D Stage IIIb
E Stage IIIc

Explanation
A 62 year old woman has TAH + BSO +
omentectomy because of ovarian
malignancy. Histology shows an
Questionendometroid adenocarcinoma involving A(Correct answ er: D)
8 the uterus and fallopian tubes. There are
microscopic deposits on the greater
omentum. Peritoneal cytology is positive.
The stage of the tumour is

Stage III
• Tumour extends beyond the pelvis
• IIIa: microscopic peritoneal metastases beyond the pelvis
• IIIb: macroscopic peritoneal metastases less than 2cm in diameter beyond the pelvis

53
• IIIc: metastases outside the pelvis greater than 2cm in diameter of positive lymph
nodes

Options for Questions 9-9


A Rectal bleeding B Blood in the urine
C Increased urinary frequency or urgency D Post-menopausal bleeding
E Post-coital bleeding

Explanation
Which symptom, if persistent or frequent,
Questionis an indication for measuring CA-125 in A(Correct answ er: C)
9 the community in women aged over 50
years?
Symptoms and signs of ovarian cancer
• The sensitivity of individual symptoms for ovarian cancer is low
• Sensitivity can be improved by combining the symptoms
• 85% of women with ovarian cancer reported at least one symptom during the year
before diagnosis
• CA-125 should be measured in primary care if a woman (especially if 50 years or
over) reports having any of the following symptoms on a persistent or frequent basis
(particularly more than 12 times per month)
• persistent abdominal distension or bloating
• feeling full (early satiety) and/or loss of appetite
• pelvic or abdominal pain
• increased urinary urgency and/or frequency
Consider measuring CA-125 in primary care if a woman reports unexplained weight loss,
fatigue or changes in bowel habit.

Options for Questions 10-10


A Pregnancy B Polycystic ovary syndrome
C Endometriosis D Uterine fibroids
E Pelvic inflammatory disease

Explanation
QuestionWhich one is not associated with raised A(Correct answ er: B)
10 CA-125?

Ovarian cancer tumour markers


CA-125
• Cancer antigen125, glycoprotein

54
• Plays a role as a lubricating barrier against foreign particles and infectious agents on
epithelial surfaces
• 79% of all ovarian cancers are positive for CA-125
• Not currently recommended for ovarian cancer screening in asymptomatic women
• Used to monitor the response to treatment and predicting prognosis after treatment in
women with CA-125 positive ovarian cancer
• Also elevated in endometrial, breast, fallopian tube, lung and GI cancers
• Elevated in benign conditions such as pregnancy, fibroids, endometriosis, pelvic
inflammatory disease

Options for Questions 11-11


A CA-125, AFP and HCG B CA-125, AFP, CEA and HCG
C CA-125, CEA and HCG D CA-125 and HCG
E CA-125

Explanation
The following tumour markers should be
Question A(Correct answ er: A)
measured in a 38 year old woman with
11
suspected ovarian cancer

Tumour marker testing


• CA125 in all women with suspected ovarian cancer
• In addition, measure AFP and beta-hCG in women under the age of 40 with
suspected ovarian cancer to identify women who may not have epithelial ovarian cancer

Options for Questions 12-12


CT scanning is superior to MRI
MRI is recommended for detecting
A B scanning in the assessment of the sub-
disease in the thorax
diaphragmatic region
MRI scanning is superior to CT
CT scanning is more operator-
C scanning in the detection of D
dependent than ultrasound scanning
retroperitoneal lymph nodes
MRI scanning is superior to CT
E scanning in the detection of disease in
the lesser sac

Explanation
QuestionWith respect to imaging for women with A(Correct answ er: B)
12 suspected ovarian cancer

Imaging in suspected ovarian cancer


• Ultrasound has the advantage of being more available, cheaper and safer

55
• Grey-scale ultrasound performs well in identifying simple cystic masses that have a
high negative predictive value. Used as the initial test, it enables adnexal masses to be
triaged into low (not ovarian cancer) and higher risk (suspected ovarian cancer)
categories
• CT is the investigation of choice for detection of disease in the thorax
• In higher risk women, a CT scan has the advantage of enabling a more
comprehensive assessment of the body, and is superior to MRI and ultrasound for
assessment of the sub-diaphragmatic regions, gastro-splenic ligament, lesser sac and
retroperitoneal nodal disease
• CT is less operator-dependent than ultrasound, and more available than MRI.

Options for Questions 13-13


A No further treatment B Adjuvant chemotherapy
C Radiotherapy – external beam D Combined chemo-radiotherapy
E Treatment with tamoxifen

Explanation
A 67 year old woman has TAH + BSO +
omentectomy for suspected ovarian
cancer. Histology shows a well
differentiated serous
Question A(Correct answ er: A)
cystadenocarcinoma confined to the right
13
ovary and not extending to the ovarian
capsule. The left ovary is normal and
peritoneal cytology is negative. Her
subsequent management should include:

Staging of Ovarian Cancer


Stage I
• Tumour confined to the ovaries.
• Stage Ia: tumour confined to one ovary with ovarian capsule intact and no tumour on
the surface of the ovary. Peritoneal cytology is negative.
• Stage Ib: tumour confined to both ovaries with ovarian capsule intact and no tumour
on the surface of the ovary. Peritoneal cytology is negative.
Stage Ic: tumour on the surface of an ovary OR positive peritoneal cytology OR the tumour
ruptures before / during surgery
TREATMENT OPTIONS
Well / moderately well differentiated Stage Ia / Ib
• Total abdominal hysterectomy and bilateral salpingo-oophorectomy with
omentectomy is adequate for patients with well differentiated or moderately well
differentiated stage IA and stage IB disease.
• The under-surface of the diaphragm should be visualized and biopsied
• Pelvic and abdominal peritoneal biopsies and pelvic and para-aortic lymph node
biopsies are required and peritoneal washings should be obtained.

56
• In women who have not completed their family and have grade I tumours, unilateral
salpingo-oophorectomy may be performed. Such women require full surgical staging
including peritoneal washings and biopsies, biopsy of para-aortic nodes and
omentectomy
• In women with apparent stage I disease, chemotherapy can be given in certain
circumstances, such as poorly differentiated tumours and in certain histological sub-
types (for example, clear cell carcinomas)
• Adjuvant chemotherapy should not be offered to women who have had optimal
surgical staging and have low-risk stage I disease (grade 1 or 2, stage Ia or 1b)

Options for Questions 14-14


Intra-peritoneal chemotherapy with
A Systemic chemotherapy with cisplatin B
cisplatin
C Systemic chemotherapy with paclitaxel D Systemic chemotherapy with carboplatin
Intra-peritoneal chemotherapy with
E
carboplatin

Explanation
A 66 year old woman had TAH + BSO +
omentectomy for suspected ovarian
cancer. Histology shows a poorly
differentiated clear cell
Questioncystadenocarcinoma extending from the A(Correct answ er: D)
14 left ovary to involve the muscularis of the
rectum. The right ovary is normal and
peritoneal cytology is negative. All visible
tumour was resected. Her subsequent
management should include:

Stage II
• Tumour extends beyond the ovary or ovaries but is confined to the pelvis.
• IIa: Tumour involves the fallopian tubes or the uterus. Peritoneal cytology is negative
• IIb: Tumour involved other pelvic structures such as the bladder or rectum. Peritoneal
cytology is negative
• IIc: Tumour extends to pelvic organs with positive peritoneal cytology
TREATMENT OPTIONS
Poorly differentiated Stage Ia / Ib or Stage Ic – stage II
• The risk of relapse and death from ovarian cancer is up to 30%. Adjuvant
chemotherapy has been shown to significantly improve survival (5 year survival
improved from 74% to 82% with Carboplatin single agent chemotherapy)
• Women with high-risk stage I disease (grade 3 or stage Ic) should be offered
adjuvant chemotherapy consisting of six cycles of carboplatin

Options for Questions 15-15

57
Block dissection of the retro-peritoneal
Retro-peritoneal lymph node biopsy is
A B lymph nodes should be routinely
not recommended
performed
Peri-operative frozen section may be Urgent microscopy of peritoneal fluid
C used to influence decision on retro- D may be used to influence decision on
peritoneal lymph node dissection retro-peritoneal lymph node dissection
Pelvic lymph node dissection is
E
recommended

Explanation
A 57 year old woman has a 12 cm
complex right ovarian mass. Her risk of
Questionmalignancy index is 327. She is A(Correct answ er: C)
15 undergoing staging laparotomy and the
tumour appears confined to the right
ovary.

• The use of peri-operative frozen section to confirm malignancy is an option to enable


selection of patients who require retroperitoneal lymphadenectomy, thereby reducing
the risks and costs of this strategy
• Retroperitoneal lymph node assessment should be undertaken as part of optimal
surgical staging in women with suspected stage I disease. Block dissection of lymph
nodes should not be routinely performed

Options for Questions 16-16


A CA-125 B CA-125, AFP
C CA-125, AFP, HCG, LDH D CA-125, CEA, AFP, HCG
E CA-125, CEA, AFP, HCG, LDH

Explanation
A 34 year old woman has been referred
to the gynaecology clinic because of
vague abdominal and pelvic symptoms.
QuestionUltrasound scan confirms a 7cm complex A(Correct answ er: C)
16 right ovarian cyst. There is no ascites,
the left ovary, liver, spleen and kidneys
appear normal. Which tumour markers
should be measured?

Investigations
Tumour markers
• CA-125 does not need to be measured in all premenopausal women with an
ultrasound diagnosis of a simple ovarian cyst
58
• If CA-125 is raised and less than 200 units/ml, further investigation may be indicated
including serial monitoring
• If CA-125 is > 200 units/ml, discussion with a gynaecological oncologist is
recommended
• Lactate dehydrogenase (LDH), AFP and HCG should be measured in all women
under age 40 with a complex ovarian mass because of the possibility of germ cell
tumours.

Options for Questions 17-17


A Clear cell adenocarcinoma B Mucinous adenocarcinoma
C Secretory adenocarcinoma D Papillary adenocarcinoma
E Ciliated adenocarcinoma

Explanation
Which histological type of endometrial
Question A(Correct answ er: A)
carcinoma is associated with the worst
17
prognosis?

Cellular Classification of Endometrial Cancer


 Endometrioid adenocarcinoma (75%–80%) including:
 Ciliated adenocarcinoma.
 Secretory adenocarcinoma
 Papillary adenocarcinoma
 Adenocarcinoma with squamous differentiation
 Adenoacanthoma
 Adenosquamous.
 Uterine papillary serous adenocarcinoma (<10%); worse prognosis
 Clear cell adenocarcinoma (4%); worse prognosis
 Mucinous adenocarcinoma (1%).
 Squamous cell carcinoma (<1%)
 Mixed tumours (10%).
 Undifferentiated tumours
10–29% of young women with endometrial cancer have a co-existing ovarian cancer.
Conservative treatment of endometrial cancer to preserve fertility is therefore not
recommended.

Options for Questions 18-18


A Stage Ia B Stage Ib
C Stage Ic D Stage IIa
E Stage IIb

Explanation

59
A 77 year old woman presents with a 6
months history of post-menopausal
bleeding. Endometrial biopsy shows a
well differentiated endometroid
Questionadenocarcinoma. MRI scan shows a 28 x A(Correct answ er: A)
18 33 mm tumour confined to the body of
the uterus and invading the inner 30% of
the myometrium. The ovaries, liver,
kidneys and bladder appear normal. The
stage of the tumour is

Stage 1
 Tumour confined to the body of the uterus
 Ia - No or < 50% myometrial invasion: TAH + BSO. ASTEC trial showed that pelvic
lymphadenectomy is not beneficial.
 Ib - 50% or more myometrial invasion
Stage II
 Tumour invades cervical stroma but does not extend beyond the uterus.
 Note that endocervical glandular invasion only should be considered stage I

Options for Questions 19-19


TAH + BSO versus Radical abdominal TAH + BSO versus TAH + BSO + pelvic
A hysterectomy in women with early B lymphadenectomy in women with early
endometrial cancer endometrial cancer
TAH + BSO versus TAH + BSO + pelvic TAH + BSO versus TAH + BSO +
C lymphadenectomy in women with D adjuvant radiotherapy in women with
advanced endometrial cancer early endometrial cancer
TAH + BSO + adjuvant radiotherapy
versus TAH + BSO + pelvic
E
lymphadenectomy in women with early
endometrial cancer

Explanation
Question A(Correct answ er: B)
The ASTEC trial was a study of
19

Efficacy of systematic pelvic lymphadenectomy in endometrial cancer (MRC ASTEC


trial): a randomised study. ASTEC study group. Lancet 2009 Jan 10;373:125-36

What is already known


 Hysterectomy and bilateral salpingo-oophorectomy (BSO) is the standard surgery for stage I
endometrial cancer.
 Systematic pelvic lymphadenectomy has been used to establish whether there is extra-uterine
disease and as a therapeutic procedure.

60
 However, randomised trials need to be done to assess therapeutic efficacy.
Aim of this study
 To investigated whether pelvic lymphadenectomy could improve survival of women with
endometrial cancer.
Methods
 Multi-centre rendomised trial.
 1408 women with histologically proven endometrial carcinoma thought preoperatively to be
confined to the corpus were randomly allocated to standard surgery (hysterectomy and BSO,
peritoneal washings, and palpation of para-aortic nodes; n=704) or standard surgery plus
lymphadenectomy (n=704).
 The primary outcome measure was overall survival.
 To control for postsurgical treatment, women with early-stage disease at intermediate or high
risk of recurrence were randomised (independent of lymph-node status) into the ASTEC
radiotherapy trial.
 Analysis was by intention to treat.
Results
 After a median follow-up of 37 months, 191 women (88 standard surgery group, 103
lymphadenectomy group) had died, with a hazard ratio (HR) of 1.16 (95% CI 0.87-1.54;
p=0.31) in favour of standard surgery and an absolute difference in 5-year overall survival of
1% (95% CI -4 to 6).
 251 women died or had recurrent disease (107 standard surgery group, 144 lymphadenectomy
group), with an HR of 1.35 (1.06-1.73; p=0.017) in favour of standard surgery and an absolute
difference in 5-year recurrence-free survival of 6% (1-12).
Conclusion
There is no evidence of benefit in terms of overall or recurrence-free survival for pelvic
lymphadenectomy in women with early endometrial cancer.

Options for Questions 20-20


A Stage IIa B Stage II
C Stage IIc D Stage IIIb
E Stage IIIc

Explanation
A 67 year old woman presents with a 6
months history of brown watery vaginal
discharge. Endometrial biopsy shows a
papillary adenocarcinoma. MRI scan
shows a 30 x 48 mm tumour in the body
Question A(Correct answ er: E)
of the uterus and invading the inner 65%
20
of the myometrium. The tumour extends
into the endocervical canal and there is
invasion of the endocervical stroma but
not the parametrium. The ovaries, liver,
kidneys and bladder appear normal.

61
These findings are confirmed at
histology. In addition, three pelvic and
one para-aortic lymph node are positive.
The stage of the tumour is
Stage II
 Tumour invades cervical stroma but does not extend beyond the uterus.
 Note that endocervical glandular invasion only should be considered stage I
Stage III
 Local and / or regional spread of the tumour
 IIIa - invades serosa or adnexae
 (positive peritoneal cytology is reported separately and does not affect the stage)
 IIIb - vaginal and / or parametrial involvement
 IIIc - pelvic or para-aortic nodes

INFECTION IN PREGNANCY

Options for Questions 1-1


A Offer HIV test at every antenatal visit B Offer HIV test at 28 weeks
C Treat her as if she is HIV positive D Discuss HIV screening with her partner
E Undertake ‘anonymous’ HIV testing

Explanation
A healthy 24 year old woman moved to
the UK from West Africa 3 months ago.
Question A(Correct answ er: B)
She attends the antenatal clinic at 14
1
weeks gestation. After counseling, she
declines the HIV test.

Screening for HIV infection in pregnancy


• All pregnant women should be offered HIV testing at booking in every pregnancy
• If HIV testing is declined, this should be documented in the maternity notes and
testing offered again at ~ 28 weeks
• HIV negative women who are judged as being at continued high risk of acquiring HIV
should be offered repeat testing later in pregnancy.
• Routine repeat testing to identify the small number of women who sero-convert
during pregnancy is not recommended.
• The results of HIV tests should be clearly documented in the notes and available to
all clinicians caring for the woman. Women should be reassured that their confidentiality
will be maintained
• The uptake of HIV screening has increased in England between 2005 and 2009 and
is now over 95%

62
Options for Questions 2-2
A Advanced maternal HIV disease B Low CD4 count
C Plasma viral load D Compliance with HAART therapy
E Duration of HIV disease

Explanation
QuestionWhich one is the strongest predictor of A(Correct answ er: C)
2 HIV vertical transmission?

Maternal factors associated with increased risk of vertical transmission include


1.Advanced maternal HIV disease
2.Low antenatal CD4 count
3.High plasma viral load – this being the strongest predictor

Options for Questions 3-3


Offer planned vaginal birth or planned Recommend planned caesarean section
A B
caesarean section at 39 weeks
Recommend planned caesarean section Recommend planned caesarean section
C D
at 38 weeks at 37 weeks
Recommend planned vaginal birth with
E
iv zidovudine during labour

Explanation
A 23 year old woman attends the
antenatal clinic at 35 weeks gestation.
She is known to be HIV positive and is
Question A(Correct answ er: C)
on highly active anti-retroviral therapy.
3
Her CD4 count is normal and her viral
load is 150 copies / ml. She wishes to
discuss mode of delivery.

Mode of delivery & Intra-partum care


• A decision about mode of delivery should be made by 36 weeks of gestation.
• Delivery by elective caesarean section at 38 weeks should be recommended for:
1) Women taking HAART who have a plasma viral load > 50 copies/ml
2) Women taking zidovudine monotherapy as an alternative to HAART
3) Women with HIV and hepatitis C co-infection.
• Otherwise delivery by elective caesarean section should be delayed until 39 weeks to
reduce the risk of neonatal respiratory morbidity.
• A planned vaginal delivery can be offered to women taking HAART who have a
plasma viral load < 50 copies/ml.

63
• Continue HAART throughout labour
• Avoid invasive procedures such as fetal blood sampling and fetal scalp electrode.
Membranes should be left intact for as long as possible. However, amniotomy and
oxytocin may be considered for augmentation of labour
• If instrumental delivery is needed, low-cavity forceps are preferable to ventouse

Options for Questions 4-4


Influenza, varicella zoster and Influenza, hepatitis B and
A B
pneumococcus vaccines pneumococcus vaccines
Rubella, measles and whooping cough Influenza, measles, whopping cough
C D
vaccines and hepatitis B vaccines
Whopping cough, varicella zoster and
E
hepatitis B vaccines

Explanation
A 35 year old woman attends the
antenatal clinic at 8 weeks gestation.
QuestionShe is known to be HIV positive but is A(Correct answ er: B)
4 not on any anti-HIV therapy. Which of
these vaccines should be offered during
pregnancy?

Antenatal screening & immunisation


• Standard screening for syphilis, hepatitis B and rubella at booking
• Screen for hepatitis C, varicella zoster, measles and toxoplasmosis at booking
• Hepatitis B and pneumococcal vaccination is recommended and can be administered
in pregnancy.
• Influenza vaccination depending on season.
• Varicella zoster and measles, mumps and rubella vaccines are contraindicated in
pregnancy.

Options for Questions 5-5


Recommend emergency caesarean Discuss mode of delivery with an HIV
A B
section physician
Recommend tocolytics and Recommend planned vaginal birth with
C D
corticosteroids iv zidovudine cover
Recommend emergency caesarean
E section with iv zidovudine for 4h before
caesarean section

Explanation

QuestionA 34 year old woman presents in A(Correct answ er: B)

64
5 spontaneous labour at 34 weeks
gestation in her first pregnancy. She is
on highly active anti-retroviral therapy
and her latest viral load was 50 copies /
ml (at 32 weeks gestation). Mode of
delivery had not been decided. She is
contracting every 3 minutes and her
cervix is fully effaced, 3 cm dilated with
intact membranes.

Planning for delivery


• A plan of care for anti-retroviral therapy and mode of delivery should be made at 36
weeks following detailed discussion with the mother.
• Only women with plasma viral loads of less than 50 copies/ml should be offered a
planned vaginal delivery.
• The plan should be reviewed when the woman presents in labour, taking into account
the most recent viral load.
• In the absence a plan for delivery, urgent advice should be sought from the HIV
physician
• If there is uncertainty about viral load results, urgent advice should be sought from
the HIV physician.

Options for Questions 6-6


A Offer membrane sweep B Offer caesarean section at 42 weeks
Offer membrane sweep plus induction of Recommend caesarean section as soon
C D
labour at 41-42 weeks as possible
Discuss risks associated with induction
E
of labour in HIV positive women

Explanation
A 34 year old woman attends the
antenatal clinic at 41 weeks gestation.
QuestionShe is known to be HIV positive on A(Correct answ er: C)
6 highly active anti-retroviral therapy. Her
viral load is less than 50 copies / ml and
vaginal delivery is planned.

Prolonged pregnancy
• If vaginal delivery is planned, the woman should be counseled and an individualised
plan made for induction of labour. There is no contraindication to membrane sweep or
to use of prostaglandins.

Options for Questions 7-7

65
Oral erythromycin + corticosteroids +
A Tocolytics + corticosteroids B
tocolytics
Corticosteroids + caesarean section Oral erythromycin + caesarean section
C D
within 48 hours within 4h
E Oral erythromycin + corticosteroids

Explanation
A 34 year old woman presents with pre-
term pre-labour rupture of the
membranes at 28 weeks gestation. She
is known to be HIV positive on highly
Question A(Correct answ er: E)
active anti-retroviral therapy. Her viral
7
load is less than 50 copies / ml and
mode of delivery is still to be decided.
There are no other maternal or fetal
concerns.

Preterm delivery and preterm pre-labour rupture of membranes (PPROM)


• For PPROM after 34 weeks, delivery should be expedited and augmentation may be
considered if the viral load < 50 copies/ml. Broad-spectrum intravenous antibiotics
should be considered.
• Before 34 weeks, oral erythromycin should be prescribed and broad-spectrum
intravenous antibiotics considered. Corticosteroids should be administered for fetal lung
maturity. Given low viral load and risks associated with birth at 28 weeks, expectant
management is appropriate.

Options for Questions 8-8


The woman should be discharged home The woman should be advised not to
A as soon as possible to protect other B breast-feed in order to protect the
women and hospital staff neonate
The woman should avoid contact with
The woman should be asked to get
C D other pregnant women for 7 days after
additional help to look after the baby
symptoms resolve
The woman should wear a surgical
E mask, an apron and gloves when
feeding her baby

Explanation
A 32 year old woman is thought to have
Question A(Correct answ er: C)
developed H1N1 influenza 24 hours after
8
a normal vaginal delivery

H1N1 Infection control measures


66
Community / Primary care
• Avoid contact with others as far as possible. Can return to work when symptom-free.
• Advice on respiratory and hand hygiene
• If post-natal, gets help looking after newborn
• Keep number of staff caring for woman to a minimum.
• Staff should take the following precautions
a) Plastic apron and gloves + surgical mask +/- eye protection if face-to-face contact < 1
meter. Patient should wear surgical mask
b) Women should be seen at home or away from communal areas in a single room
c) Ensure high standard of hygiene and hand washing

Options for Questions 9-9


She can start trying for a pregnancy as She should wait for 7 days after
A B
soon as she wishes stopping treatment
She should wait for 1 month after She should wait for 3 months after
C D
stopping treatment stopping treatment
She should wait for 6 months after
E
stopping treatment

Explanation
A 33 year old woman has just returned
from a malaria endemic area. She has
Question A(Correct answ er: D)
been taking mefloquine
9
chemoprophylaxis and is thinking about
trying for a pregnancy.

Malaria Chemoprophylaxis
• Causal prophylaxis: directed against the hepatic phase of the disease which takes ~ 7
days. Drugs should be continued for 7 days after leaving endemic area
• Suppressive prophylaxis: directed against erythrocytic phase and should be continued for
4 weeks after leaving endemic area
• Women who are planning a pregnancy: should be advised not to travel or delay
pregnancy / travel plans. If this is not possible, advice should be sought from a specialist
centre. If prophylaxis has been used, women should be advised to wait for complete excretion
of drugs before becoming pregnant. Risk of terratogenicity is low and pregnancy while taking
malaria prophylaxis is not an indication for termination
• Available evidence does not indicate that Mefloquine exposure in the first trimester is
terratogenic
Drug excretion times are as follows:
Mefloquine 3 months?
Atovaquone and proguanil (Malarone®) 2 weeks??
Doxycycline 1 week??
Proguanil 1 week

67
Options for Questions 10-10
Offer detailed scan at 12 weeks to
A Offer termination of pregnancy B
exclude congenital anomalies
Offer detailed cardiac scan at 22 weeks
C D Reassure
to exclude cardiac anomalies
Offer folic acid 5mg daily and detailed
E
scan at 12 weeks to exclude anomalies

Explanation
A 33 year old woman has just returned
from a malaria endemic area. She has
been taking mefloquine prophylaxis. She
Question A(Correct answ er: D)
has also been taking the progestogen-
10
only contraceptive pill but finds herself
unexpectedly pregnant. Ultrasound scan
confirms a 6 weeks viable pregnancy.

• Women who are planning a pregnancy: should be advised not to travel or delay
pregnancy / travel plans. If this is not possible, advice should be sought from a specialist
centre. If prophylaxis has been used, women should be advised to wait for complete excretion
of drugs before becoming pregnant. Risk of terratogenicity is low and pregnancy while taking
malaria prophylaxis is not an indication for termination
• Available evidence does not indicate that Mefloquine exposure in the first trimester is
terratogenic

Options for Questions 11-11


A Fetal monitoring B Delivery of the fetus
C Blood samples for culture and sensitivity D Maternal fluid resuscitation
E Broad spectrum antibiotics

Explanation
A 23 year old woman presents at 38
weeks gestation because she is feeling
increasingly unwell with lower abdominal
Questionpain and diarrhea and upper respiratory A(Correct answ er: D)
11 symptoms. Her temperature = 39 C, P =
132/min, BP = 76/42 mmHg and SPO2 =
95% on air. Which one is your most
important priority?

Initial assessment & resuscitation

68
• On presentation with suspected sepsis, women should have basic observations
including temperature, pulse and BP
• Pulse > 90 / min, systolic BP < 90 mmHg or MAP < 70 mmHg in the presence of
temperature > 38 C or < 36 C should prompt assessment of respiratory rate, SO2 and
urgent medical review
• Fluid resuscitation- In the absence of pre-eclampsia, cardiac disease or renal
disease, administer 1.5L (or 20ml/kg) crystalloid rapidly. Aim to maintain systolic BP >
90 mmHg or MAP > 65 mmHg, urine out-put at ≥ 0.5ml/kg/h and CVP at 8-12 mmHg
• Delivery by caesarean section may be necessary as part of resuscitation for severe
sepsis in the third trimester. Fetal monitoring should only occur once maternal
resuscitation is underway. Delivery for fetal reasons when the mother is critically ill may
result in maternal death

Options for Questions 12-12


Piperacillin – tazobactam iv +
A Flucloxacillin iv B
gentamicin iv
Piperacillin – tazobactam iv + Co-amoxiclav iv + gentamicin iv +
C D
gentamicin iv + teicloplanin iv clindamycin iv
Benzylpenicillin iv + gentamicin iv +
E
teicoplanin iv

Explanation
A 33 year old woman is admitted with
severe sepsis secondary to a wound
infection 4 days after emergency
Questioncaesarean section for suspected fetal A(Correct answ er: C)
12 compromise. MRSA swab taken on day
1 post caesarean section is positive. She
has no known allergies. Which is the
most appropriate antibiotic regimen?

Severe sepsis or septic shock(seek urgent microbiological advice):


1) Piperacillin–tazobactam [Tazocin ®] 4.5 g 8-hourly or ciprofloxacin 600 mg 12-hourly plus
gentamicin (3–5 mg/kg daily in divided doses every 8 hours by slow intravenous injection).
Tazobactam is a beta-lactamase inhibitor.
Or
2) A carbapenem such as meropenem (500 mg to 1 g 8-hourly by intravenous injection over 5
minutes or by intravenous infusion) plus gentamicin
Metronidazole 500 mg 8-hourly may be considered to provide anaerobic cover.
Risk factors for MRSA
Add teicoplanin 10 mg/kg 12-hourly for three doses then 10 mg/kg 24-hourly or linezolid 600
mg twice Daily

Options for Questions 13-13

69
10-20% if a woman has primary genital 40-45% if a woman has primary genital
A B
herpes and vaginal birth herpes and vaginal birth
5-10% if a woman with a history of
2-10% if a woman with recurrent genital
C genital herpes before pregnancy has a D
herpes has a vaginal birth
vaginal birth
Not affected by the duration of ruptured
E membrane in women with recurrent
genital herpes

Explanation
Question A(Correct answ er: B)
The risk of neonatal herpes is
13

• Recurrent lesions in labour: risk of neonatal herpes is 0-3% and should be


balanced against the risks of C/S. Routine C/S not recommended. The mode of
delivery should be discussed with the woman and individualised according to the
clinical circumstances and the woman’s preferences.
• C/S recommended for all women presenting with first-episode genital herpes at
the time of labour or within 6 weeks of the expected date of delivery (perform CS at
term or if woman presents in pre-term labour). The risk of neonatal herpes is ~41% if
delivered vaginally and increases considerably if membranes have been ruptured for
> 4h

Options for Questions 14-14


Offer genital swabs in every trimester to
A Offer caesarean section at 39 weeks B
detect infection
Advise the woman to avoid unprotected
Offer blood tests in every trimester for
C D intercourse with her partner during
herpes simplex antibodies
pregnancy
Offer prophylactic acyclovir to her
E
partner

Explanation
A 23 year old woman attends the
antenatal clinic at 20 weeks gestation.
QuestionHer partner has a history of genital A(Correct answ er: D)
14 herpes and suffers 2-3 recurrences every
year. The woman has no history of
genital herpes.

• All women should be asked at their first antenatal visit if they or their male
partner have ever had genital herpes. Female partners of men with genital herpes,
70
who themselves give no history of genital herpes, should be advised about reducing
their risk of acquiring this infection: avoiding intercourse when the partner has
lesions or using condoms during pregnancy. Value likely to be very limited
• Identification of women susceptible to genital herpes in pregnancy using type
specific antibody testing not indicated

Options for Questions 15-15


A Influenza B Hepatitis B
C Whooping cough D MMR
E H. influenzae

Explanation
QuestionWhich vaccine should not be A(Correct answ er: D)
15 administered during pregnancy?

VACCINATION????
Types of Vaccines ????
Killed organisms??
Live-attenuated organisms - replicate in the host. Provide a larger ‘dose’ and have the
advantage of inducing immunity at the natural site of infection??
Sub-unit vaccines - make use of the most antigenic subunit of the organism. These can be
produced by genetic engineering??
Toxoid - bacterial exotoxins are first detoxified by treatment with formaldehyde?????
BACTERIAL??
Live Attenuated ???
BCG - tuberculosis???
Inactivated ???
Cholera vaccine??
Pertussis vaccine??
Typhoid vaccine?????
Sub-unit ???
H. influenzae??
N. meningitides??
S. pneumoniae?????
Toxoid ???
Tetanus??
Diphtheria?????
VIRUSES??
Live Attenuated ???
MMR - measles, mumps, rubella??
Polio (Sabin)??
Varicella zoster - not in general use??
Adenovirus??

71
Yellow fever?????
Inactivated ???
Polio (Salk)??
Influenza??
Rabies??
Hepatitis A?????
Sub-unit ???
Hepatitis B??
Influenza?????
Live vaccines should not be administered during pregnancy?

Options for Questions 16-16


Hepatitis B, Hepatitis C, Chlamydia
A B Hepatitis B, Hepatitis C, HIV
trachomatis
C Hepatitis B, HIV, Bacterial vaginosis D Hepatitis C, Cytomegalovirus, HIV
E Hepatitis B, HIV, Syphilis

Explanation
QuestionScreening for which infections should be A(Correct answ er: E)
16 offered as part of routine antenatal care?

Hepatitis B ??
• Serological screening for hepatitis B virus should be offered to pregnant women so that
effective postnatal interventions can be offered to infected women to decrease the risk of
mother-to-child transmission.??
Hepatitis C ??
• Pregnant women should not be offered routine screening for hepatitis C virus because
there is insufficient evidence to support its clinical and cost effectiveness.??
HIV ??
• Pregnant women should be offered screening for HIV infection early in antenatal care
because appropriate antenatal interventions can reduce mother-to-child transmission of HIV
infection.??• A system of clear referral paths should be established in each unit or department
so that pregnant women who are diagnosed with an HIV infection are managed and treated by
the appropriate specialist teams.??
Syphilis ??
• Screening for syphilis should be offered to all pregnant women at an early stage in
antenatal care because treatment of syphilis is beneficial to the mother and baby.??
• Because syphilis is a rare condition in the UK and a positive result does not necessarily
mean that a woman has syphilis, clear paths of referral for the management of pregnant
women testing positive for syphilis should be established.??

Options for Questions 17-17


A Ultrasound scanning can be used to B Spiramycin should be used to treat

72
accurately determine the fetal maternal symptoms
consequences of the infection
Pyrimethamine + sulfonamide + folinic
Spiramycin + sulfonamide + folinic acid
C D acid should be offered to treat fetal
should be offered to treat fetal disease
infection
Ganciclovir is effective in treating fetal
E
infection

Explanation
A 34 year old woman with a small for
Question A(Correct answ er: D)
gestation age fetus at 28 weeks
17
gestation is found to have toxoplasmosis

• Prevention - Avoid contact with cat faeces / raw meat??


• Cost-effectiveness of antenatal screening not proven
• Ultrasonography or invasive testing to diagnose fetal infection. No accurate method to
predict severity of fetal disease.??
• Spiramycin therapy to infected mothers - 60% reduction in the risk of fetal infection??
• Scans / amniocentesis / cordocentesis to diagnose fetal infection??
• TOP / pyrimethamine + sulfonamide + folinic acid if fetal infection - helps arrest
progression of fetal disease??
• Infected neonates require treatment for the first year of life??

Options for Questions 18-18


Infection in the mother does not cross Infection is associated with meconium-
A B
the placenta stained liquor
Causes early onset neonatal disease
Causes early onset neonatal disease
C which is usually due to horizontally D
which typically presents as meningitis
acquired infection
Infection is typically associated with fetal
E
intra-cranial calcification

Explanation
Question A(Correct answ er: B)
Listeria monocytogenes
18

Fetal risks ??
• Trans-placental spread causes fetal infection??
• May cause chorio-amnionitis and septic miscarriage or fetal death in-utero??
• Associated with meconium stained liquor??
EARLY-ONSET
• Neonatal disease - most common after acute febrile illness in the mother??
• Neonate symptomatic at birth or within a few days of life??
73
• Associated with disseminated granulomas (Granulomatosis infantisepticum) involving
liver, placenta and other solid organs, septic shock and respiratory distress??
LATE-ONSET
• Neonatal disease - occurs in term neonates after uncomplicated pregnancy and typically
presents with meningitis. Horizontally acquired and maternal cultures are usually negative??

Options for Questions 19-19


The chancre typically appears 5-7 days The chancre typically resolves
A B
after infection spontaneously after 2-6 weeks
Lymphadenopathy typically presents 2-3 Condylomata lata typically occur 6-9
C D
weeks after infection months after infection
The chancre typically presents 6-9
E
months after infection

Explanation
QuestionWith respect to infection with treponema A(Correct answ er: B)
19 pallidum

SYPHILIS ??
• Treponema pallidum - spirochaete - cannot be cultured in-vitro Incubation period 10-90
days??
• Primary syphilis: Chancre - genital / peri-anal / rectal. Indurated painless ulcer. Regresses
spontaneously after 2-6 weeks??
• Secondary syphilis: 6-8 weeks after primary syphilis; fever, malaise, maculo-papular rash,
lymphadenopathy, serpiginous mouth ulcers. Condylomata lata??
• Latent syphilis: May persist for up to 30 years. Not sexually transmissible??
• Tertiary syphilis: Gumma. Skin / respiratory tract /sub-periosteal aspect of long bones /
joints - Charcot's joints??
• Quaternary syphilis: Aortic aneurysms / aortitis, CNS complications - Tabes dorsalis /
generalised paralysis of the insane.??

Options for Questions 20-20


A Recommend caesarean section B Recommend planned vaginal birth
Daily acyclovir until 36 weeks then Daily acyclovir from 36 weeks with
C D
planned vaginal birth planned vaginal birth
Daily acyclovir until delivery with
E
planned vaginal birth

Explanation
A 24 year old woman with 3 previous
Question A(Correct answ er: E)
vaginal births presents with a first
20
episode of genital herpes at 34 weeks

74
gestation. Treatment with acyclovir has
been commenced. She attends at 37
weeks gestation to discuss mode of
delivery. PCR from genital swab showed
Herpes simplex type 1. Blood taken at
the time of the acute infection showed
Herpes simplex type 1 IgG positive and
IgM negative; Herpes simplex type II IgG
negative and IgM negative.

HSV INFECTION AFTER 28 WEEKS


 Effect on perinatal morbidity (preterm labour and low birth weight) and stillbirth is uncertain
 Treatment should be in line with clinical condition with oral (or intravenous for disseminated
HSV) aciclovir (400 mg three times daily, usually for 5 days) plus daily suppressive aciclovir
400 mg three times daily until delivery.
 Caesarean section should be the recommended for all women developing first episode genital
herpes after 28+0 weeks, particularly those developing symptoms within 6 weeks of expected
delivery, as the risk of neonatal transmission of HSV is very high at 41%.
 Up to 15% of cases where a woman presents with a first episode of clinical HSV infection will
be a recurrent infection.
 For women within 6 weeks of expected delivery, type-specific HSV antibody testing (IgG
antibodies to HSV-1 and HSV-2) will influence the advice on mode of delivery and risk of
neonatal herpes infection. The presence of antibodies of the same type as the HSV isolated
from genital swabs would confirm this episode to be a recurrence rather than a primary
infection and elective caesarean section would not be indicated. It may take 2–3 weeks for the
results to become available.
 Initial plan of delivery should be based on the assumption that all first episode lesions are
primary genital herpes. This plan can be modified if HSV antibody test results subsequently
confirm a recurrent infection.
* serology = recurrent infection rather than primary infection.

Urogynaecology

Options for Questions 1-1


If she does not report mixed urinary
If she does not report pure urgency
incontinence, there is a 10% chance
incontinence, there is a 1-2% chance
A she will be found to have urodynamic B
she will be found to have detrusor over-
stress incontinence plus detrusor over-
activity on multichannel cystometry
activity on multichannel cystometry
If she does not report pure urinary If she reports mixed urinary
C D
stress incontinence, there is a 1-2% incontinence, there is a 90% chance

75
chance she will be found to have she will be found to have urodynamic
urodynamic stress incontinence on stress incontinence plus detrusor over-
multichannel cystometry activty on multichannel cystometry
If she reports urgency incontinence but
no other symptoms, there is a 90%
E chance she will be found to have
detrusor over-activity on multichannel
cystometry

Explanation
QuestionA 67 year old woman has been referred A(Correct answ er: A)
1 because of urinary incontinence

Diagnostic accuracy of urinary symptoms


Likelihood of symptoms predicting urodynamic stress incontinence or detrusor overactivity on
multichannel cystometry is shown in table.
Median positive Median negative
Incontinence Median sensitivity Median specificity
predictive value predictive value
symptoms (range) (range)
(range) (range)
Stress UI
66% (17–83%) 83% (49–92%) 70% (41–95%) 69% (49–85%)

Mixed UI
68% (42–85%) 77% (34–89%) 35% (18–70%) 90% (80–97%)

Urgency UI 45% (14–86%) 96% (81–98%) 73% (25–81%)


91% (79–98%)

 The Negative Predictive Value is important in assessing whether urodynamics provide


additional information compared with clinical history. This summarises the extent to
which a negative history is associated with a negative finding on urodynamics.
 If a woman does not report mixed UI, the probability of finding urodynamic stress
incontinence (USI) plus DO on cystometry is small (~10%), therefore urodynamic testing
might be said to offer little additional diagnostic value.
 If a woman does not report pure urgency UI, the probability of finding DO on cystometry
is small (~10%), therefore urodynamic testing offers little added diagnostic value.
 15–51% (median 31%) of women who do not report pure stress UI may be found to
have USI on cystometry.
 Urodynamic testing is not required before initiating conservative treatment

Options for Questions 2-2


A Pad test B Fluid-Bridge test
Ultrasound assessment of residual
C D Q-tip test
volume
E Bonney test

76
Explanation
Which one of the following tests is
Question A(Correct answ er: C)
recommended in the initial assessment
2
of women with urinary incontinence?

The following should not be offered


 Cystoscopy in the initial assessment
 Pad tests
 Q-tip, Bonney, Marshall and Fluid-Bridge tests
 Magnetic resonance imaging, computed tomography
 Ultrasound is not recommended other than for the assessment of residual urine volume.

Options for Questions 3-3


Women with stress incontinence and a Women with urgency incontinence and
A BMI over 30 should be advised to lose B a BMI over 30 should be advised to lose
weight weight
Women with stress incontinence should
Women with stress incontinence should
C D be advised to reduce their level of
be advised to reduce caffeine intake
exercise
Women with stress incontinence should
E
be advised to reduce fluid intake

Explanation
QuestionWith respect to lifestyle advice for A(Correct answ er: B)
3 women with urinary incontinence

Lifestyle advice
 Recommend a trial of caffeine reduction to women with OAB.
 Consider advising modification of high or low fluid intake in women with UI or OAB.
 Advise women with UI or OAB who have a BMI greater than 30 to lose weight.

Options for Questions 4-4


If bladder training does not achieve
Bladder training should be offered for a satisfactory benefit after 6 weeks, a
A B
minimum of 6 months combination of bladder training and anti-
cholinergic drugs should be offered
Electrical stimulation is an alternative to Pelvic floor muscle training is
C D
bladder training recognized first line treatment
Trans-cutaneous sacral nerve
stimulation should be offered if the
E
woman does not wish to undergo
bladder training

77
Explanation
A healthy 43 year old woman has been
Question A(Correct answ er: B)
referred because of urgency urinary
4
incontinence

Bladder training
 Offer bladder training lasting for a minimum of 6 weeks as first-line treatment to women with
urgency or mixed UI.
 If women do not achieve satisfactory benefit from bladder training programmes, the
combination of an OAB drug with bladder training should be considered if frequency is a
troublesome symptom.
The following should not be offered
 Electrical stimulation in the treatment of women with OAB.
 Electrical stimulation in combination with pelvic floor muscle training.
 Perineometry or pelvic floor electromyography as biofeedback as a routine part of pelvic floor
muscle training.
 Transcutaneous sacral nerve stimulation to treat OAB in women.
 Transcutaneous posterior tibial nerve stimulation for OAB.

Options for Questions 5-5


A rise in detrusor pressure of greater
A rise in detrusor pressure of greater
A B than or equal to 15 mmHg during
than 15 mmHg during filling cystometry
micturiction
A rise in detrusor pressure of greater A rise in detrusor pressure of greater
C than 15 cm H2O during filling D than or equal to 15 cm H2O during
cystometry micturiction
A rise in detrusor pressure greater than
E
15 mmHg in the presence of urgency

Explanation
Question A(Correct answ er: C)
Detrusor over-activity is characterized by
5

Detrusor Over-activity
 Occurrence of uncontrolled spontaneous contraction of detrusor muscle during filling, or on
provocation, while the patient is actively trying to inhibit micturition??
 Diagnosis is therefore made at filling cystometry:??
1. Rise in detrusor pressure of >15cm H2O?
2. Rise in detrusor pressure of <15cm H2O in the presence of urgency or urge incontinence??
 A rise in detrusor pressure during filling may also be caused by a low compliance bladder.
 Detrusor over-activity may co-exist with urodynamic stress incontinence??

78
 Presents with symptoms of frequency, urgency / urge incontinence, nocturia, nocturnal
enuresis, incontinence during coitus, or stress incontinence.??
 Cystometry is not essential for diagnosis in women without stress incontinence.??
 In women complaining of mixed symptoms including stress incontinence, empirical treatment
for detrusor overa-ctivity with conservative management of stress incontinence may be
undertaken, with urodynamic assessment only if treatment fails or the woman is being
considered for incontinence surgery.

Options for Questions 6-6


The absolute probability of discontinuing The absolute probability of discontinuing
A B
from treatment at 4 weeks is 1-2% from treatment at 12 weeks is 15-30%
The absolute probability of discontinuing The absolute probability of discontinuing
C D
from treatment at 4 weeks is 7-11% from treatment at 12 weeks is 5-7%
The absolute probability of discontinuing
E
from treatment at 4 weeks is over 30%

Explanation
A 45 year old woman presents with
urinary frequency, urgency and urge
Question A(Correct answ er: B)
incontinence but no other symptoms.
6
She is considering treatment with anti-
cholinergic drugs.

Medical therapy

• The absolute probability of being continent at 4 weeks ranged from 16% for
tolterodine (extended release) to 27% for oxybutynin (immediate release)
• The absolute probability of being continent at 12 weeks ranged from 12% for
tolterodine (extended release) to 21% for oxybutynin (immediate release)
• The absolute probability of discontinuing from treatment at 4 weeks range from 4%
for solifenacin and darifenacin to 11% for oxybutynin (immediate release) and
propiverine (immediate release)
• The absolute probability of discontinuing from treatment at 12 weeks range from 15%
for solifenacin and darifenacin to 31% for oxybutynin (immediate release) and
propiverine (immediate release)
• The cost per year of OAB drugs ranged from £364 to £578. The difference in
effectiveness was very small – a difference less than of 0.01 QALY between the most
effective (oxybutynin immediate release) and least effective drug (tolterodine extended
release) over a year
• The weekly cost of oxybutynin (immediate release), and tolterodine (immediate
release) were below the cost of no treatment since the latter included £8 per week for
incontinence pads.

79
• Oxybutynin (immediate release) was the most cost-effective first-line antimuscarinic
therapy. All other drugs were more expensive and less effective than oxybutynin
immediate release.
• The probability of a drug being the most cost effective at £20,000 per QALY was
highest for oxybutynin (immediate release) and tolterodine (immediate release). All
other drugs had no more than a 5% chance of being cost effective at that threshold.

Options for Questions 7-7


The weekly cost of ‘no treatment’ is
The most cost-effective treatment is
A higher than the weekly cost of anti- B
solifenacin
cholinergic drugs
The most cost-effective treatment is The least cost-effective drug is
C D
tolterodine extended release oxybutynin immediate release
The drug with the worse side-effect
E
profile is oxybutynin immediate release

Explanation
QuestionA 46 year old woman complains of A(Correct answ er: A)
7 urinary urge incontinence

Medical therapy
• The absolute probability of being continent at 4 weeks ranged from 16% for
tolterodine (extended release) to 27% for oxybutynin (immediate release)
• The absolute probability of being continent at 12 weeks ranged from 12% for
tolterodine (extended release) to 21% for oxybutynin (immediate release)
• The absolute probability of discontinuing from treatment at 4 weeks range from 4%
for solifenacin and darifenacin to 11% for oxybutynin (immediate release) and
propiverine (immediate release)
• The absolute probability of discontinuing from treatment at 12 weeks range from 15%
for solifenacin and darifenacin to 31% for oxybutynin (immediate release) and
propiverine (immediate release)
• The cost per year of OAB drugs ranged from £364 to £578. The difference in
effectiveness was very small – a difference less than of 0.01 QALY between the most
effective (oxybutynin immediate release) and least effective drug (tolterodine extended
release) over a year
• The weekly cost of oxybutynin (immediate release), and tolterodine (immediate
release) were below the cost of no treatment since the latter included £8 per week for
incontinence pads.
• Oxybutynin (immediate release) was the most cost-effective first-line antimuscarinic
therapy. All other drugs were more expensive and less effective than oxybutynin
immediate release.
• The probability of a drug being the most cost effective at £20,000 per QALY was
highest for oxybutynin (immediate release) and tolterodine (immediate release). All
other drugs had no more than a 5% chance of being cost effective at that threshold.

80
Options for Questions 8-8
Urodynamic studies confirm detrusor The woman has been trained in clean
A B
over-activity intermittent self-catheterisation
The woman is able and willing to
Medical therapy has been unsuccessful
C perform clean intermittent self- D
or not tolerated
catheterisation
Per-cutaneous sacral nerve stimulation
E
has been unsuccessful or declined

Explanation
A 56 year old woman complains of
severe urinary urgency and urge
Question A(Correct answ er: E)
incontinence. Which one is not an
8
essential criterion for botulinum toxin
treatment?

Botulinum-A toxin
Pre-treatment management & counselling
• Start treatment with botulinum toxin A only if women:
a) Have been trained in clean intermittent catheterisation and have performed the technique
successfully, and
b) Are able and willing to perform clean intermittent catheterisation on a regular basis for as
long as needed.

• Discuss the risks and benefits of treatment with botulinum toxin A with women before
seeking informed consent including
i) the likelihood of being symptom free or having a large reduction in
symptoms
ii) the risk of clean intermittent catheterisation and the potential for it to be
needed for variable lengths of time after the effect of the injections has worn off
iii) the absence of evidence on duration of effect between treatments and the
long-term efficacy and risks
iv) the risk of adverse effects, including an increased risk of urinary tract
infection.

Options for Questions 9-9


If initial treatment is unsuccessful, a
A The recommended dose is 150 units B
repeat injection should be offered
A dose of 100 units is associated with a
A dose of 300 units has been shown to
C lower risk of needing intermittent self- D
be more effective than lower doses
catheterisation
E If treatment is unsuccessful, the woman

81
should be offered per-cutaneous
posterior tibial nerve stimulation

Explanation
With respect to the use of Botulinum
Question A(Correct answ er: C)
toxin for the treatment of detrusor
9
instability

Offer Botulinum toxin A


• After an MDT review, offer bladder wall injection with botulinum toxin A to women
with OAB caused by proven detrusor over-activity that has not responded to
conservative management including drug treatment
• Use 200 units when offering botulinum toxin A.
• Consider 100 units of botulinum toxin A for women who would prefer a dose with a
lower chance of catheterisation and accept a reduced chance of success.

Failure of botulinum toxin A


• If the first botulinum toxin A treatment has no effect discuss with the MDT
• Consider percutaneous sacral nerve stimulation after MDT review

If the woman is unable / unwilling to perform intermittent self-catheterisation


• Offer percutaneous sacral nerve stimulation after MDT review if OAB has not
responded to conservative management including drugs

Options for Questions 10-10


A Previous vaginal delivery B Marfan’s syndrome
C Ehlers-Danlos syndrome D Spina bifida
E Reiter’s syndrome

Explanation
QuestionWhich one is not a recognized risk factor A(Correct answ er: E)
10 for pelvic organ prolapse?

Pelvic organ prolapse


• Approximately 35% of the women who have a vaginal hysterectomy for prolapse will
later require additional prolapse surgery.
• Uterine or vault prolapse has a major impact on quality of life and on the use of NHS
services
• Vaginal delivery is the greatest risk factor
• Nulliparous women with connective tissue disorders (e.g., Ehlers-Danlos or Marfan’s
syndrome) or women with spina bifida and poorly functioning pelvic floor muscles are
also at increased risk and can present with advanced POP at a young age.

82
Options for Questions 11-11
Is recommended for pelvic organ
Loses the majority of its tensile strength
A prolapse repair in sexually active B
within 21 days
women
Maintains the majority of its tensile Is used to form a permanent bridge
C D
strength for over 90 days between anatomical structures
Is recommended for use in sling
E operations for urinary stress
incontinence

Explanation
Question A(Correct answ er: B)
Absorbable synthetic (polygalactin) mesh
11

USE OF MESH FOR PROLAPSE SURGERY


Absorbable synthetic mesh (polyglactin)
• Loses 75% of its strength after 21 days.
• Act as a supportive material for the formation of granulation tissue,
• Not suitable for operations which depend on the mesh to form a permanent bridge
between two anatomical structures. Not suitable for prolapse repair. Adequate fibrous
tissue incorporation does not occur before hydrolysis of the polygalactin mesh.
Combination of absorbable and non-absorbable mesh?
• Use the permanent strength of a non-absorbable mesh to provide the bridge function
but are covered with other materials to decrease the risk of erosion and promote
integration with the host tissues (e.g. collagen, and polyglactin).

Options for Questions 12-12


Supervised pelvic floor muscle training Training should include at least 8
A should be undertaken for at least 6 B contractions every minute for 30
months minutes
Higher intensity pelvic floor muscle
Training is not recommended for women training has been shown to have a
C D
with mixed urinary incontinence higher short-term success rate than
lower intensity training
Should only be offered to women who
E decline minimally invasive surgical
procedures

Explanation
QuestionPelvic floor muscle training to treat A(Correct answ er: D)
12 urinary stress incontinence

83
PELVIC FLOOR MUSCLE TRAINING (PFMT) AND VAGINAL CONES
• Offer a trial of supervised pelvic floor muscle training of at least 3 months’ duration as
first-line treatment to women with stress or mixed urinary incontinence.
• Pelvic floor muscle training programmes should comprise at least 8 contractions
performed 3 times per day.
• Do not use perineometry or pelvic floor electromyography as biofeedback as a
routine part of pelvic floor muscle training.
• Continue an exercise programme if pelvic floor muscle training is beneficial.
• Daily PFMT is an effective treatment for stress or mixed urinary incontinence
compared with no treatment over the short term.
• Other than occasional cases of pain or discomfort, no other adverse effects were
noted.
• In studies of up to 1 year, higher intensity PFMT regimens confer greater subjective
cure or Improvement than lower intensity regimens. Over the longer term, differences
between these groups are not sustained.
• There is a lack of evidence for optimum training regimens for PFMT.
• There is no additional benefit from the use of PFMT in women undergoing treatment
with tolterodine for OAB

Options for Questions 13-13


A Pregnancy B Retroverted uterus
C Menstruation D Untreated vaginal infection
E Severe utero-vaginal prolapse

Explanation
A 37 year old woman presents with
Questionurinary stress incontinence. Which one is A(Correct answ er: B)
13 not a contra-indication to the use of
vaginal cones

PELVIC FLOOR MUSCLE TRAINING (PFMT) AND VAGINAL CONES


• In women with stress urinary incontinence, vaginal cones are more effective than no
treatment over the short term.
• There is no evidence of a difference in effectiveness between cones and PFMT.
• Compared with PFMT, cones are associated with more adherence problems.
• One study suggested that the training time for using vaginal cones is one-third of that
for PFMT, which would make vaginal cones cheaper than PFMT. However, it is not
clear what the appropriate training regimen should be for women using vaginal cones.
• Vaginal cones are not suitable for all women. Cones are inappropriate for use in
some circumstances, such as when there is a moderate to severe prolapse, too narrow
or too capacious a vagina causing difficulty with insertion or misplacement of the cone,
untreated atrophic vaginitis, vaginal infection, or during menstruation or pregnancy.

84
• Evidence does not indicate additional benefit from biofeedback with PFMT in
comparison with PFMT alone in treating UI.
• Biofeedback with PFMT is more costly than PFMT alone and therefore is not cost
effective.
• There are limited data on the use of magnetic therapy for urinary incontinence, and
its role in treatment of women is unclear
• PFMT is more cost effective than duloxetine alone, as first-line treatment for stress
urinary incontinence.
• Electrical stimulation and/or biofeedback should be considered in women who cannot
actively contract pelvic floor muscles in order to aid motivation and adherence to
therapy

Options for Questions 14-14


A Retropubic ‘bottom-up’ B Retropubic ‘top-down’
C Transobturator ‘inside-out’ D Transobturator ‘outside-in’
E Retropubic ‘inside-out’

Explanation
Which one is not a recognized technique
Question A(Correct answ er: E)
used in mid-urethral tape procedures for
14
urinary stress incontinence?

Synthetic mid-urethral tape

• Synthetic mid-urethral tapes are inserted via a small sub-urethral vaginal incision and
small incisions in the supra-pubic or perineal area, depending on the type of trocar
used.
• As a result of the minimal access approach, hospital stay and morbidity are reduced
and recovery is quicker.
• Complications include tape erosion, pain and the risk of vessel/nerve or organ
damage due to the minimal access approach.
• Different approaches can be used, including retropubic‘bottom-up’ and ‘top-down’
and transobturator‘inside-out’ and ‘outside-in’.
• More recently single incision approaches have been introduced whereby the
synthetic tape is inserted via a single vaginal incision.

Options for Questions 15-15


The risk of tissue injury is significantly
The risk of urinary retention is lower with
lower with single incision procedures
A B single incision procedures compared to
compared to retropubic bottom-up
retropubic bottom-up procedures
procedures
The risk of tape erosion is higher with There is no significant difference in
C D
single incision procedures compared to continence rates when single incision

85
retropubic bottom-up procedures procedures are compared to retropubic
bottom-up procedures
Patient satisfaction is lower with single
E incision procedures compared to
retropubic bottom-up procedures

Explanation
QuestionWith respect to surgical treatment A(Correct answ er: D)
15 options for urinary stress incontinence

Retropubic‘bottom-up’ versus single incision


Patient satisfaction with treatment
• One RCT showed no difference in clinical benefit between retropubic‘bottom-up’ and
single incision.
Continence status
• Two RCTs showed no difference in clinical benefit between retropubic‘bottom-up’ and
single incision.
Tissue injury
• Five RCTs showed no difference in clinical benefit between retropubic‘bottom-up’ and
single incision.
Erosion rate
• One RCT showed no difference in clinical benefit between retropubic‘bottom-up’ and
single incision.
Retention
• One RCT showed no difference in clinical benefit between retropubic‘bottom-up’ and
single incision.

Options for Questions 16-16


Inside-out procedures are associated Inside-out procedures are associated
A with higher patient satisfaction rates B with higher risk of tissue injury than
than outside-in procedures outside-in procedures
There is no significant difference in
Inside-out procedures are associated
continence-specific quality of life when
C with higher risk of tape erosion than D
inside-out procedures are compared to
outside-in procedures
outside-in procedures
Inside-out procedures are associated
E with a higher risk of voiding dysfunction
than outside-in procedures

Explanation
With respect to transobturator
Question A(Correct answ er: D)
procedures used in the management of
16
stress urinary incontinence

86
Transobturator‘outside-in’ versus transobturator‘inside-out’
Patient satisfaction with treatment
• Two RCTs showed no difference in clinical benefit between transobturator‘inside-out’
and transobturator‘outside-in’.
Continence status
• Two RCTs showed a clinical benefit in favour of transobturator‘inside-out’.
Incontinence-specific quality of life
• One RCT showed no difference in clinical benefit between transobturator‘inside-out’
and transobturator‘outside-in’.
Tissue injury
• Three RCTs showed a clinical benefit in favour of transobturator‘inside-out’.
Erosion rate
• Two RCTs showed no difference in clinical benefit between transobturator‘inside-out’
and transobturator‘outside-in’.

Options for Questions 17-17


Maximum prolapse is present when the
Assessment should not be performed
A vaginal wall protrudes through the B
with the woman standing
introitus
Points above the hymen are indicated Points below the hymen are indicated
C D
by a plus (+) sign by a plus (+) sign
There are 2 reference points on the
E anterior vaginal wall and 3 reference
points on the posterior vaginal wall

Explanation
QuestionWith respect to the POP-Q assessment A(Correct answ er: D)
17 of pelvic organ prolapse

POPQ Assessment
• Based on demonstrating the maximum extent of prolapse.
• Maximum prolapse is present when the vaginal wall becomes tight during straining, traction
on the prolapse causes no further descent or the patient confirms that protrusion is maximal
(for instance, using a mirror). A standing, straining examination should be done to confirm that
the full extent of the prolapse was observed in other positions used.
• The fixed point of reference for all POPQ measurements is the plane of the hymen.
• Six points (two on the anterior vaginal wall, two on the superior vagina, and two on the
posterior vaginal wall) are measured with reference to the plane of the hymen.
• Points above (proximal to) the hymen are described by the distance from the hymenal plane
in centimeters preceded by a minus sign (-2 cm), and points below (distal to) the hymen are
preceded by a positive sign (+2 cm).
• A full description of the examination should include type of speculum / retractors used, patient
position, bladder and rectal fullness

87
Options for Questions 18-18
A Abdominal sacro-colpopexy B Sacro-spinous fixation
C Ileo-coccygeus fixation D Laparoscopic sacro-colpopexy
E Colpocliesis

Explanation
A healthy 57 year old woman presents
with vaginal vault prolapse 10 years after
Question A(Correct answ er: B)
vaginal hysterectomy. Which operation is
18
typically associated with deviation of the
vaginal axis?

Surgical treatment
1) Abdominal sacro-colpopexy
2) Sacro-spinous fixation
• There is currently no evidence to recommend one procedure over the other
• Operating surgeon should be experienced in both procedures
• Sacro-spinous fixation requires adequate vaginal length to reach the sacro-spinous ligament
• Vaginal procedure may be appropriate in the presence of vaginal wall prolapse. However,
prolapse may also be reduced abdominally or laparoscopically
• Vaginal procedure more suitable for frail women because of morbidity associated with
abdominal incision
• Abdominal procedure appropriate if woman is undergoing laparotomy for other reasons and
is more suitable for sexually active women as sacro-spinous fixation results in deviation of the
vaginal axis and may be associated with dyspareunia. Sacro-spinous fixation may cause
vaginal narrowing and shortening especially if combined with anterior / posterior repair
3) Ileo-coccygeus fixation is currently not recommended
4) Vaginal utero-sacral ligament suspension is effective but associated with a risk of ureteric
injury and should be used with caution
5) Laparoscopic sacro-colpopexy appears to be as effective as open procedures but requires
operative laparoscopic skills and longer operating times.
6) Colpocleisis should be considered in women who do not wish to retain sexual function.
Short operating time and low risk of morbidity makes this a useful option in women who are
unfit for major surgery.

Options for Questions 19-19


A Cyclic AMP B Cyclic GMP
C Calcium D Inositol triphosphate
E Inositol monophosphate

Explanation

88
Detrusor relaxation is mainly mediated
Question A(Correct answ er: A)
by pathways involving which second
19
messenger system?

Detrusor relaxation is mainly mediated by the cyclic AMP pathway activated via the fixation of
noradrenalin to β –adrenoreceptors
Options for Questions 20-20
Have more cardiovascular side-effects Have more cardiovascular side-effects
A B
than Beta-1 agonists than Beta-2 agonists
Increase bladder capacity without an Increase bladder capacity with an
C D
increase in residual volume increase in residual volume
Have no effect on bladder function in
E
humans

Explanation
Question A(Correct answ er: C)
Beta-3 receptor agonists
20
Compared with other agents (including antimuscarinics), β3 agonists increase bladder capacity
with no change in micturition pressure and residual volume, supporting the principle of β3-AR
agonism as a new therapeutic approach to OAB.
Receptors on smooth muscle are thought to be the main site of action for these agents in
treating detrusor overactivity, although they also promote relaxation by directly influencing
urothelial functions
These drugs have fewer cardiovascular side effects compared with β1,2 agonists
Mirabegron has a marketing authorisation in the UK for the 'symptomatic treatment of urgency,
increased micturition frequency and/or urgency incontinence as may occur in patients with
overactive bladder (OAB)'.
Formulated as oral controlled absorption system (OCAS) tablets - a modified release system
(extended release or prolonged release) that allows the release of drug from the tablets for an
extended period.
Available in two strengths of 50 mg orally once daily dose, with or without food and 25 mg for
patients with severe renal or moderate hepatic impairment).

Pre-term labour / PPROM –


Options for Questions 1-1
A Risk of necrotising enterocolitis B Risk of neonatal death
Risk of systemic infection in the first 7
C D Risk of intra-ventricular haemorrhage
days of life
E Risk of admission to neonatal intensive

89
care unit

Explanation
Which adverse outcome has not been
Questionshown to be significantly reduced in A(Correct answ er: C)
1 women at risk of pre-term birth treated
with antenatal corticosteroids?

A Cochrane review of 21 studies showed that treatment of women at risk of preterm birth with
a single course of antenatal corticosteroids reduced the risk of
 Neonatal death by 31% (95% CI 19–42%)
 RDS by 44% (95% CI 31–57%)
 Intraventricular haemorrhage by 46% (95% CI 31%–67%).
Antenatal corticosteroid use is also associated with a reduction in necrotising enterocolitis,
respiratory support, intensive care admissions and systemic infections in the first 48 hours of
life compared with no treatment or treatment with placebo
Treatment with antenatal corticosteroids does not increase risk to the mother of death,
chorioamnionitis or puerperal sepsis.
Treatment with antenatal corticosteroids is associated with an overall reduction in
 Neonatal death (RR 0.69, 95% CI 0.58 to 0.81)
 RDS (RR 0.66, 95% CI 0.59 to 0.73)
 Cerebroventricular haemorrhage (RR 0.54, 95% CI 0.43 to 0.69)
 Necrotising enterocolitis (RR 0.46, 95% CI 0.29 to 0.74)
 Respiratory support, intensive care admissions (RR 0.80, 95% CI 0.65 to 0.99)
 Systemic infections in the first 48 hours of life (RR 0.56, 95% CI 0.38 to 0.85)

Options for Questions 2-2


A 23+0 and 34+0 weeks B 23+0 and 34+6 weeks
C 24+0 and 34+0 weeks D 24+0 and 34+6 weeks
E 24+0 and 36+6 weeks

Explanation
Antenatal corticosteroids should be
Question A(Correct answ er: D)
offered to women at risk of pre-term birth
2
between

RCOG guidelines
 Clinicians should offer a single course of antenatal corticosteroids to women between 24+0and
34+6 weeks of gestation who are at risk of preterm birth.
 Antenatal corticosteroids can be considered for women between 23+0and 23+6weeks of
gestation who are at risk of preterm birth.

Options for Questions 3-3

90
Between 24 hours and 7 days after
A Within 7 days of the first dose B
administration of the first dose
Between 24 hours and 7 days after Between 24 hours and 7 days after
C D
administration of the second dose admission to hospital
Within 24 hours of administration of the
E
second dose

Explanation
Antenatal corticosteroids are most
Question A(Correct answ er: C)
effective in reducing the risk of RDS in
3
pregnancy that deliver

Antenatal corticosteroids are most effective in reducing RDS in pregnancies that deliver 24
hours after and up to 7 days after administration of the second dose of antenatal
corticosteroids.
 Reduction in RDS is seen in infants born up to 7 days after the first dose (RR 0.46, 95% CI
0.35–0.60).
 No reduction in neonatal death, RDS or cerebro-ventricular haemorrhage is seen in infants
delivered more than 7 days after treatment with antenatal corticosteroids.
 Antenatal corticosteroid use reduces neonatal death even when infants are born less than 24
hours after the first dose has been given (RR 0.53, 95% CI 0.29–0.96) and therefore should
still be given even if delivery is expected within this time

Options for Questions 4-4


Have been shown to reduce the risk of Have been shown to reduce the risk of
A neonatal death in infants delivered more B of neonatal death in infants delivered
than 7 days after treatment less than 24 hours after the first dose
Should not be offered if delivery is Should not be offered if delivery is
C expected less than 24 hours after D expected less than 24 hours after
administration of the first dose administration of the second dose
Have been shown to reduce the risk of
E RDS in infants delivered more than 7
days after treatment

Explanation
Question A(Correct answ er: B)
Antenatal corticosteroids
4

Antenatal corticosteroids are most effective in reducing RDS in pregnancies that deliver 24
hours after and up to 7 days after administration of the second dose of antenatal
corticosteroids.
 Reduction in RDS is seen in infants born up to 7 days after the first dose (RR 0.46, 95% CI
0.35–0.60).
91
 No reduction in neonatal death, RDS or cerebro-ventricular haemorrhage is seen in infants
delivered more than 7 days after treatment with antenatal corticosteroids.
 Antenatal corticosteroid use reduces neonatal death even when infants are born less than 24
hours after the first dose has been given (RR 0.53, 95% CI 0.29–0.96) and therefore should
still be given even if delivery is expected within this time

Options for Questions 5-5


Offer 2 doses of im betamethasone 24h Offer 2 doses of im betamethasone 12h
A B
apart apart
Betamethasone is not indicated
Betamethasone is not indicated
C D because gestation age is over 34+6
because labour is being induced
weeks
Betamethasone is indicated because
E induction of labour is associated with an
increased risk of caesarean section

Explanation
A 34 year old woman attends the fetal
medicine unit at 35+0 weeks gestation
for ultrasound monitoring because of a
Questionsmall for gestational age fetus. Following A(Correct answ er: A)
5 the scan, a decision is made to induce
her labour over the next 24-48 hours
because of abnormal umbilical artery
Dopplers.

Pregnancies affected by fetal growth restriction between 24+0 and 35+6 weeks of gestation at
risk of delivery should receive a single course of antenatal corticosteroids.

Options for Questions 6-6


A 20% B 40%
C 60% D 80%
E 90%

Explanation
A healthy 23 year old woman presents
with confirmed rupture of the membranes
at 38 weeks gestation. She has no
Question A(Correct answ er: C)
uterine contractions and is feeling good
6
fetal movements. She should be
informed that the likelihood of
spontaneous labour in the next 24 hours

92
is

• Common – pre-labour rupture of the membranes at term occurs in up to 10% of


pregnancies while pre-term pre-labour rupture occurs in 3% of pregnancies
• Pre-term pre-labour rupture of the membranes is the commonest single identifiable
cause of pre-term delivery, occurring in 30-40% of pre-term deliveries
• Associated with an increased risk of fetal and maternal infection especially if
prolonged (over 24h before the onset of labour).

Options for Questions 7-7


A 1% B 3%
C 5% D 10%
E 15%

Explanation
QuestionPre-labour rupture of the membranes at A(Correct answ er: D)
7 term occurs in what % of pregnancies?

• Common – pre-labour rupture of the membranes at term occurs in up to 10% of


pregnancies while pre-term pre-labour rupture occurs in 3% of pregnancies
• Pre-term pre-labour rupture of the membranes is the commonest single identifiable
cause of pre-term delivery, occurring in 30-40% of pre-term deliveries
• Associated with an increased risk of fetal and maternal infection especially if
prolonged (over 24h before the onset of labour).

Options for Questions 8-8


A 1% B 3%
C 5% D 10%
E 15%

Explanation
Preterm pre-labour rupture of the
Question A(Correct answ er: B)
membranes at term occurs in what % of
8
pregnancies?

• Common – pre-labour rupture of the membranes at term occurs in up to 10% of


pregnancies while pre-term pre-labour rupture occurs in 3% of pregnancies
• Pre-term pre-labour rupture of the membranes is the commonest single identifiable
cause of pre-term delivery, occurring in 30-40% of pre-term deliveries
• Associated with an increased risk of fetal and maternal infection especially if
prolonged (over 24h before the onset of labour).

93
Options for Questions 9-9
Pre-term pre-labour rupture of the
A B Urinary tract infection
membranes
C Pre-eclampsia D Ante-partum haemorrhage
E Chorioamnionitis

Explanation
QuestionWhich one is the commonest single A(Correct answ er: A)
9 identifiable cause of pre-term delivery?

• Common – pre-labour rupture of the membranes at term occurs in up to 10% of


pregnancies while pre-term pre-labour rupture occurs in 3% of pregnancies
• Pre-term pre-labour rupture of the membranes is the commonest single identifiable
cause of pre-term delivery, occurring in 30-40% of pre-term deliveries
• Associated with an increased risk of fetal and maternal infection especially if
prolonged (over 24h before the onset of labour).

Options for Questions 10-10


Empirical treatment as pre-labour Ultrasound scan for liquor volume and
A B
rupture of the membranes reassure if normal liquor
Ultrasound scan for liquor volume and FBC and CRP and treat as ruptured
C treat as ruptured membranes if D membranes if raised CRP or white cell
oligohydramnios count
Give the woman a pad to wear and
E
inspect pad after 1-2 hours

Explanation
A healthy 23 year old woman attends the
assessment unit because she has been
experiencing damp underwear for about
8 hours. She is 38 weeks pregnant in her
first pregnancy. She is feeling good fetal
Question A(Correct answ er: E)
movements and there are no uterine
10
contractions. She is wearing a pad which
is slightly stained examination. After
speculum examination, you remain
uncertain about a diagnosis of pre-labour
rupture of the membranes.

94
Examination
• Examination of woman’s pad or underwear – presence of fluid may remove the need
for speculum examination
• Speculum examination if uncertainty from history and examination of pad or
underwear. Digital vaginal examination should not be performed
• If uncertainty remains, speculum examination may be repeated after the woman has
been supine for a period of time to identify any pooling of liquor in the vagina or the
woman may be given a pre-weighed pad which is re-weighed after a period of time
• Vaginal swabs and CRP are not indicated and ultrasound scanning is unreliable
• Ferning of amniotic fluid on a microscope slide or pH assessment using Nitrazine paper are
unreliable. Newer devices to detect the presence of amniotic fluid in the vagina are claimed to
be more reliable but are not used routinely
Options for Questions 11-11
A HVS, CRP, fetal heart rate B LVS, CRP, fetal heart rate
C HVS, LVS, fetal heart rate D CRP, fetal heart rate
E Fetal heart rate

Explanation
A healthy 23 year old woman attends the
assessment unit because of a sudden
gush of vaginal fluid at 38 weeks
gestation in her first pregnancy. She
Questioncontinues to drain clear fluid and is A(Correct answ er: E)
11 feeling good fetal movements but there
are no uterine contractions. She is
wearing a pad which is wet on
examination. The following investigations
are recommended

Investigations at term
• Vaginal swabs and CRP are not indicated and ultrasound scanning is unreliable
• Ferning of amniotic fluid on a microscope slide or pH assessment using Nitrazine
paper are unreliable. Newer devices to detect the presence of amniotic fluid in the
vagina are claimed to be more reliable but are not used routinely
• Fetal heart rate should be monitored

Options for Questions 12-12


A High vaginal swab B Urethral and high vaginal swab
C High vaginal swab + low vaginal swab D Urethral, low vaginal and perineal swab
E High vaginal swab and CRP

Explanation

95
A 23 year old woman presents with
spontaneous rupture of the membranes
at 26 weeks gestation. She is draining
Question A(Correct answ er: D)
clear liquor and there are no uterine
12
contractions. Which is the most
appropriate test to screen for Group B
streptococcus colonization?

Investigations pre-term
• Swabs for microscopy, culture and sensitivity – endocervical, high vaginal and a
single urethral, low vaginal and perineal swab for isolation of Group B streptococcus
• FBC and CRP as base-line / markers of infection
• Fetal heart rate monitored by CTG
• Ultrasound scan for fetal presentation as mal-presentation is more common and
presentation may not be ascertained clinically

Options for Questions 13-13


Await spontaneous labour with
Await spontaneous labour with
A B caesarean section at 38 weeks after
caesarean section at 39 weeks
steroids
Await spontaneous labour with Offer caesarean section after a course
C D
caesarean section at 38 weeks of steroids
Offer induction of labour if not in labour
E
after 24 hours

Explanation
A 33 year old woman with one previous
caesarean section attends for a
discussion on mode of delivery at 36
weeks gestation. The agreed plan is for
elective caesarean section at 39 weeks
with caesarean section if she presents in
Question A(Correct answ er: D)
labour. Induction of labour has been
13
declined. She presents with spontaneous
rupture of the membranes at 37 weeks.
There is clear liquor draining with good
fetal movements. Maternal and fetal
observations are normal and there is no
uterine activity.

• IOL has been declines


• Elective CS is planned with CS if labour – plan has to be accepted as reasons not
indicated in the question

96
• Ruptured membranes at term – offer birth if not in spontaneous labour after 24 h
• Appropriate to use this time to administer a course of steroids even though
evidence for benefit is very limited

Options for Questions 14-14


Corticosteroids, erythromycin for 10
Erythromycin for 10 days and induction
A days and induction of labour at 37 B
of labour at 38 weeks
weeks
Corticosteroids, erythromycin for 10
Erythromycin for 10 days and induction
C days and induction of labour at 36 D
of labour after 24 hours
weeks
Discuss risks and benefits of induction
E
of labour and expectant management

Explanation
A healthy 26 year old woman presents
with pre-labour rupture of the
Questionmembranes at 35 +2 weeks gestation. A(Correct answ er: E)
14 She is draining clear liquor and feeling
good fetal movements. Maternal and
fetal observations are normal.

Counselling & Treatment before term


• The main risks are prematurity and maternal & fetal / neonatal infection. Prolonging
pregnancy reduces the risks associated with prematurity but increases the risks of
infection and these risks should be discussed with the woman to enable a joint informed
decision on timing of delivery
• Although good quality evidence is lacking, it is generally accepted that before 34
weeks, the balance of risks is in favour of prolonging pregnancy
• After 34 weeks, it may be appropriate to manage pregnancy as for premature rupture
of the membranes at term after discussion of risks and benefits with the woman and in
collaboration with the neonatologists

Options for Questions 15-15


Erythromycin, co-amoxiclav and
A B Erythromycin and co-amoxiclav
erythromycin + co-amoxiclav
Erythromycin and erythromycin + co- Erythromycin, benzyl-penicillin and
C D
amoxiclav erythromycin + benzyl-penicillin
E Erythromycin and benzyl-penicillin

Explanation

QuestionWhich antibiotics were studied in the A(Correct answ er: A)

97
15 ORACLE I randomized trial?

Broad spectrum antibiotics for preterm, prelabour rupture of fetal membranes: the ORACLE I
randomised trial
AIM OF THIS STUDY ??
• To examine the effects of prophylactic antibiotics and choice of antibiotic on
neonatal outcome in PPROM??
STUDY DESIGN ??
• Randomised multi-centre trial??
Entry criteria
• PPROM at <37 weeks gestation where the need to prescribe antibiotics was
uncertain??
Exclusion criteria
• Antibiotics already prescribed or thought necessary, imminent delivery, contra-
indications to antibiotic use??
The primary outcome measure was a composite of neonatal death, chronic lung disease
(receiving daily supplementary oxygen at age 36 weeks post conception), major cerebral
abnormality on ultrasound scan before discharge??

RESULTS ??
• Women were randomised to receive erythromycin 250mg (n = 1197), co-
amoxiclav 325mg (n = 1212), both antibiotics (n = 1192) or placebo (n = 1225) four
times daily for 10 days or until delivery??
• Erythromycin was associated with a significantly lower risk of the primary
composite outcome compared to placebo (12.7% Vs 15.2%; p = 0.08)??
• Co-amoxiclav only and co-amoxiclav + erythromycin was not significantly
different from placebo with respect to the primary composite outcome??
• Erythromycin was associated with: ?1) A significant reduction in delivery within
48h??2) A significant reduction in the proportion of babies requiring >21% O2??3) A
significant reduction in the proportion of babies requiring surfactant treatment??4) A
significant reduction in the proportion of positive neonatal blood cultures??
• Co-amoxiclav was associated with: ??1) A significant reduction in delivery within
48h??2) A significant reduction in delivery within 7 days??3) A significant reduction
in the proportion of babies requiring >21% O2??4) A significant reduction in the
proportion of babies requiring surfactant treatment??5) A significant Increase in the
proportion of babies with suspected and proven necrotising enterocolitis (4.1% Vs
2.7% and 1.9% Vs 0.5% respectively)??
• Neither antibiotic significantly reduced the risk of perinatal death or maternal
morbidity (length of hospital stay and antibiotic use) although the combination of
erythromycin and co-amoxiclav was associated with a significant reduction in the
need for maternal antibiotic prescription within 14 days.?

Options for Questions 16-16


Corticosteroids, tocolytics and
A B Corticosteroids + tocolytics
erythromycin

98
C Corticosteroids + erythromycin D Tocolytics + erythromycin
Corticosteroids + tocolytics + benzyl-
E
penicillin

Explanation
A 23 year old woman presents with
uterine contractions every 3 minutes at
30 weeks gestation. There is no vaginal
bleeding or discharge and she is feeling
Question A(Correct answ er: B)
good fetal movements. The fetus is in a
16
breech presentation and the fetal heart
rate is normal. On speculum examination
the cervix is fully effaced and 1 cm
dilated. Fetal fibronectin test is positive.

• Threatened pre-term labour


• Tocolytics to buy time for corticosteroids / in-utero transfer
• Prophylactic antibiotics do not improve outcome (ORACLE II)

Options for Questions 17-17


A 2% B 0.9%
C 0.1% D 0.01%
E 0.03%

Explanation
QuestionThe incidence of cerebral palsy in term A(Correct answ er: C)
17 infants is

• The incidence of cerebral palsy decreases significantly with increasing


gestational age: 14.6% at 22–27 weeks of gestation, 6.2% at 28–31 weeks, 0.7% at 32–
36 weeks and 0.1% in term infants.
• Twenty-five percent of all cases of cerebral palsy are in infants born at less than
34 weeks of gestation.

Options for Questions 18-18


A Before 26+0 weeks gestation B Before 28+0 weeks gestation
C Before 30 +0 weeks gestation D Before 34+6 weeks gestation
E Before 36+6 weeks gestation

Explanation

99
Magnesium sulphate for fetal neuro-
Question A(Correct answ er: C)
protection should be considered for
18
women at risk of pre-term birth

RCOG SAC Report


• Magnesium sulphate should be considered in women at less than 30 weeks of
gestation.
• While benefit has been observed at more advanced gestations, the magnitude of
effect is likely to be largest at earliest gestations and limitation of resources makes an
upper limit of 30 weeks a pragmatic choice

Options for Questions 19-19


A Women with pre-eclampsia B Women with sepsis
Women expected to give birth within 4
C Women with ruptured membranes D
hours
E None of the above

Explanation
QuestionMagnesium sulphate for fetal neuro- A(Correct answ er: E)
19 protection should not be used in

• All studies described included multiple pregnancies, those where delivery was
expected within 24 hours and those with premature rupture of membranes.
• This suggests that irrespective of the indication any infant delivering preterm
might be expected to benefit from the observed reduction in cerebral palsy risk

Options for Questions 20-20


In women with two or more previous In women with a previous pre-term birth
A B
pre-term births before 28 weeks gestation
In women with ultrasound evidence of In women with a positive fetal fibronectin
C D
funnelling before 24 weeks gestation test before 24 weeks gestation
E Only as part of a clinical trial

Explanation
QuestionProgesterone should be used to prevent A(Correct answ er: E)
20 pre-term birth
RCOG website
• Although a reduction in preterm birth seems attractive, there is little evidence as
yet for short term benefit to the baby, even if progesterone does prevent preterm
delivery.
• Furthermore, there is no evidence of long term benefit for the baby. This may not
just be “absence of evidence” as it is becoming increasingly recognised that a delay in

100
delivery could have adverse effects if the fetus remains in an adverse intrauterine
environment.
• The RCOG therefore endorses current recommendations that, in women at high
risk of preterm delivery, progesterone administration should be restricted to clinical trials
to determine whether its use is associated with improved fetal, neonatal and/or infant
outcome.

Menopause
Options for Questions 1-1
A Risk of necrotising enterocolitis B Risk of neonatal death
Risk of systemic infection in the first 7
C D Risk of intra-ventricular haemorrhage
days of life
Risk of admission to neonatal intensive
E
care unit

Explanation
Which adverse outcome has not been
Questionshown to be significantly reduced in A(Correct answ er: C)
1 women at risk of pre-term birth treated
with antenatal corticosteroids?

A Cochrane review of 21 studies showed that treatment of women at risk of preterm birth with
a single course of antenatal corticosteroids reduced the risk of
 Neonatal death by 31% (95% CI 19–42%)
 RDS by 44% (95% CI 31–57%)
 Intraventricular haemorrhage by 46% (95% CI 31%–67%).
Antenatal corticosteroid use is also associated with a reduction in necrotising enterocolitis,
respiratory support, intensive care admissions and systemic infections in the first 48 hours of
life compared with no treatment or treatment with placebo
Treatment with antenatal corticosteroids does not increase risk to the mother of death,
chorioamnionitis or puerperal sepsis.
Treatment with antenatal corticosteroids is associated with an overall reduction in
 Neonatal death (RR 0.69, 95% CI 0.58 to 0.81)
 RDS (RR 0.66, 95% CI 0.59 to 0.73)
 Cerebroventricular haemorrhage (RR 0.54, 95% CI 0.43 to 0.69)
 Necrotising enterocolitis (RR 0.46, 95% CI 0.29 to 0.74)
 Respiratory support, intensive care admissions (RR 0.80, 95% CI 0.65 to 0.99)
 Systemic infections in the first 48 hours of life (RR 0.56, 95% CI 0.38 to 0.85).

101
Options for Questions 2-2
Between 24 hours and 7 days after
A Within 7 days of the first dose B
administration of the first dose
Between 24 hours and 7 days after Between 24 hours and 7 days after
C D
administration of the second dose admission to hospital
Within 24 hours of administration of the
E
second dose

Explanation
Antenatal corticosteroids are most
Question A(Correct answ er: C)
effective in reducing the risk of RDS in
2
pregnancy that deliver

Antenatal corticosteroids are most effective in reducing RDS in pregnancies that deliver 24
hours after and up to 7 days after administration of the second dose of antenatal
corticosteroids.
 Reduction in RDS is seen in infants born up to 7 days after the first dose (RR 0.46, 95% CI
0.35–0.60).
 No reduction in neonatal death, RDS or cerebro-ventricular haemorrhage is seen in infants
delivered more than 7 days after treatment with antenatal corticosteroids.
 Antenatal corticosteroid use reduces neonatal death even when infants are born less than 24
hours after the first dose has been given (RR 0.53, 95% CI 0.29–0.96) and therefore should
still be given even if delivery is expected within this time

Options for Questions 3-3


Have been shown to reduce the risk of Have been shown to reduce the risk of
A neonatal death in infants delivered more B of neonatal death in infants delivered
than 7 days after treatment less than 24 hours after the first dose
Should not be offered if delivery is Should not be offered if delivery is
C expected less than 24 hours after D expected less than 24 hours after
administration of the first dose administration of the second dose
Have been shown to reduce the risk of
E RDS in infants delivered more than 7
days after treatment

Explanation
Question A(Correct answ er: B)
Antenatal corticosteroids
3

Antenatal corticosteroids are most effective in reducing RDS in pregnancies that deliver 24
hours after and up to 7 days after administration of the second dose of antenatal
corticosteroids.

102
 Reduction in RDS is seen in infants born up to 7 days after the first dose (RR 0.46, 95% CI
0.35–0.60).
 No reduction in neonatal death, RDS or cerebro-ventricular haemorrhage is seen in infants
delivered more than 7 days after treatment with antenatal corticosteroids.
 Antenatal corticosteroid use reduces neonatal death even when infants are born less than 24
hours after the first dose has been given (RR 0.53, 95% CI 0.29–0.96) and therefore should
still be given even if delivery is expected within this time

Options for Questions 4-4


A reduction in birth weight with an
A reduction in neonatal head
A B increase in neonatal head
circumference and birth weight
circumference
An increase in birth weight with a
An increase in the long-term risk of type
C reduction in neonatal head D
II diabetes
circumference
An increase in the long-term risk of
E
cardiovascular disease

Explanation
Weekly repeat courses of antenatal
Question A(Correct answ er: A)
corticosteroids have been shown to be
4
associated with

Although repeat courses reduce the occurrence and severity of neonatal lung disease (RR
0.60, 95% CI 0.48–0.75) and the risk of serious health problems in the first few weeks of life
(RR 0.79, 95% CI 0.67–0.93), they are associated with being small for gestational age at birth
(RR 1.63, 95% CI 1.12–2.37). Babies exposed to multiple courses were shorter (44.5 cm
versus 45.4 cm, P<0.001), and had a smaller head circumference (31.1 cm versus 31.7
cm, P<0.001). A regime of repeat doses should not be recommended.
 A single rescue course may be considered with caution in pregnancies where the initial course
was given at less than 26+0 weeks of gestation. Senior opinion should be sought if a rescue
course is to be considered.

Options for Questions 5-5


A 20% B 40%
C 60% D 80%
E 90%

Explanation
A healthy 23 year old woman presents
Question A(Correct answ er: C)
with confirmed rupture of the membranes
5
at 38 weeks gestation. She has no

103
uterine contractions and is feeling good
fetal movements. She should be
informed that the likelihood of
spontaneous labour in the next 24 hours
is

PRE-LABOUR RUPTURE OF THE MEMBRANES (Premature rupture of the membranes)


• At term, about 60% of women will go into spontaneous labour within 24 hours of
membrane rupture. Before term, about 50% of women will deliver within 7 days of
membrane rupture, the majority of these delivering in the first 24h. Of those who remain
undelivered after 7 days, about 50% will deliver every subsequent week.

Options for Questions 6-6


A 5% B 15%
C 35% D 50%
E 75%

Explanation
A healthy 23 year old woman presents
with confirmed rupture of the membranes
at 28 weeks gestation. She has no
Question A(Correct answ er: D)
uterine contractions and is feeling good
6
fetal movements. She should be
informed that the likelihood of
spontaneous labour in the next 7 days is

PRE-LABOUR RUPTURE OF THE MEMBRANES (Premature rupture of the membranes)


• At term, about 60% of women will go into spontaneous labour within 24 hours of
membrane rupture. Before term, about 50% of women will deliver within 7 days of
membrane rupture, the majority of these delivering in the first 24h. Of those who remain
undelivered after 7 days, about 50% will deliver every subsequent week.

Options for Questions 7-7


A 1% B 3%
C 5% D 10%
E 15%

Explanation
Preterm pre-labour rupture of the
Question A(Correct answ er: B)
membranes at term occurs in what % of
7
pregnancies?

104
• Common – pre-labour rupture of the membranes at term occurs in up to 10% of
pregnancies while pre-term pre-labour rupture occurs in 3% of pregnancies
• Pre-term pre-labour rupture of the membranes is the commonest single identifiable
cause of pre-term delivery, occurring in 30-40% of pre-term deliveries
• Associated with an increased risk of fetal and maternal infection especially if
prolonged (over 24h before the onset of labour).

Options for Questions 8-8


Pre-term pre-labour rupture of the
A B Urinary tract infection
membranes
C Pre-eclampsia D Ante-partum haemorrhage
E Chorioamnionitis

Explanation
QuestionWhich one is the commonest single A(Correct answ er: A)
8 identifiable cause of pre-term delivery?

• Common – pre-labour rupture of the membranes at term occurs in up to 10% of


pregnancies while pre-term pre-labour rupture occurs in 3% of pregnancies
• Pre-term pre-labour rupture of the membranes is the commonest single identifiable
cause of pre-term delivery, occurring in 30-40% of pre-term deliveries
• Associated with an increased risk of fetal and maternal infection especially if
prolonged (over 24h before the onset of labour).

Options for Questions 9-9


Empirical treatment as pre-labour Ultrasound scan for liquor volume and
A B
rupture of the membranes reassure if normal liquor
Ultrasound scan for liquor volume and FBC and CRP and treat as ruptured
C treat as ruptured membranes if D membranes if raised CRP or white cell
oligohydramnios count
Give the woman a pad to wear and
E
inspect pad after 1-2 hours

Explanation
A healthy 23 year old woman attends the
assessment unit because she has been
experiencing damp underwear for about
Question A(Correct answ er: E)
8 hours. She is 38 weeks pregnant in her
9
first pregnancy. She is feeling good fetal
movements and there are no uterine
contractions. She is wearing a pad which

105
is slightly stained examination. After
speculum examination, you remain
uncertain about a diagnosis of pre-labour
rupture of the membranes.

Examination
• Examination of woman’s pad or underwear – presence of fluid may remove the need
for speculum examination
• Speculum examination if uncertainty from history and examination of pad or
underwear. Digital vaginal examination should not be performed
• If uncertainty remains, speculum examination may be repeated after the woman has
been supine for a period of time to identify any pooling of liquor in the vagina or the
woman may be given a pre-weighed pad which is re-weighed after a period of time
• Vaginal swabs and CRP are not indicated and ultrasound scanning is unreliable
• Ferning of amniotic fluid on a microscope slide or pH assessment using Nitrazine
paper are unreliable. Newer devices to detect the presence of amniotic fluid in the
vagina are claimed to be more reliable but are not used routinely

Options for Questions 10-10


Corticosteroids, erythromycin for 10
Erythromycin for 10 days and induction
A days and induction of labour at 37 B
of labour at 38 weeks
weeks
Corticosteroids, erythromycin for 10
Erythromycin for 10 days and induction
C days and induction of labour at 36 D
of labour after 24 hours
weeks
Discuss risks and benefits of induction
E
of labour and expectant management

Explanation
A healthy 26 year old woman presents
with pre-labour rupture of the
Questionmembranes at 35 +2 weeks gestation. A(Correct answ er: E)
10 She is draining clear liquor and feeling
good fetal movements. Maternal and
fetal observations are normal.

Counselling & Treatment before term


• The main risks are prematurity and maternal & fetal / neonatal infection. Prolonging
pregnancy reduces the risks associated with prematurity but increases the risks of
infection and these risks should be discussed with the woman to enable a joint informed
decision on timing of delivery
• Although good quality evidence is lacking, it is generally accepted that before 34
weeks, the balance of risks is in favour of prolonging pregnancy

106
• After 34 weeks, it may be appropriate to manage pregnancy as for premature rupture
of the membranes at term after discussion of risks and benefits with the woman and in
collaboration with the neonatologists

Options for Questions 11-11


Compared intravenous antibiotics to Compared corticosteroids to placebo in
A placebo in women with pre-term pre- B women with pre-term pre-labour rupture
labour rupture of the membranes of the membranes
Compared oral antibiotics to placebo in Compared tocolytics to placebo in
C women with pre-labour rupture of the D women with pre-labour rupture of the
membranes membranes
Compared oral antibiotics to placebo in
E women with pre-term pre-labour rupture
of the membranes

Explanation
Question A(Correct answ er: E)
The ORACLE I randomized trial
11

Broad spectrum antibiotics for preterm, prelabour rupture of fetal membranes: the ORACLE I
randomised trial
AIM OF THIS STUDY ??
• To examine the effects of prophylactic antibiotics and choice of antibiotic on
neonatal outcome in PPROM??
STUDY DESIGN ??
• Randomised multi-centre trial??
Entry criteria
• PPROM at <37 weeks gestation where the need to prescribe antibiotics was
uncertain??
Exclusion criteria
• Antibiotics already prescribed or thought necessary, imminent delivery, contra-
indications to antibiotic use??
The primary outcome measure was a composite of neonatal death, chronic lung disease
(receiving daily supplementary oxygen at age 36 weeks post conception), major cerebral
abnormality on ultrasound scan before discharge??
RESULTS ??
• Women were randomised to receive erythromycin 250mg (n = 1197), co-
amoxiclav 325mg (n = 1212), both antibiotics (n = 1192) or placebo (n = 1225) four
times daily for 10 days or until delivery??
• Erythromycin was associated with a significantly lower risk of the primary
composite outcome compared to placebo (12.7% Vs 15.2%; p = 0.08)??
• Co-amoxiclav only and co-amoxiclav + erythromycin was not significantly
different from placebo with respect to the primary composite outcome??

107
• Erythromycin was associated with: ?1) A significant reduction in delivery within
48h??2) A significant reduction in the proportion of babies requiring >21% O2??3) A
significant reduction in the proportion of babies requiring surfactant treatment??4) A
significant reduction in the proportion of positive neonatal blood cultures??
• Co-amoxiclav was associated with: ??1) A significant reduction in delivery within
48h??2) A significant reduction in delivery within 7 days??3) A significant reduction
in the proportion of babies requiring >21% O2??4) A significant reduction in the
proportion of babies requiring surfactant treatment??5) A significant Increase in the
proportion of babies with suspected and proven necrotising enterocolitis (4.1% Vs
2.7% and 1.9% Vs 0.5% respectively)??
• Neither antibiotic significantly reduced the risk of perinatal death or maternal
morbidity (length of hospital stay and antibiotic use) although the combination of
erythromycin and co-amoxiclav was associated with a significant reduction in the
need for maternal antibiotic prescription within 14 days.?

Options for Questions 12-12


A 500 mg tds B 500 mg qds
C 250 mg tds D 250 mg qds
E 500 mg bd

Explanation
QuestionThe dose of erythromycin used in the A(Correct answ er: D)
12 ORACLE I randomized trial was

Broad spectrum antibiotics for preterm, prelabour rupture of fetal membranes: the ORACLE I
randomised trial
AIM OF THIS STUDY ??
• To examine the effects of prophylactic antibiotics and choice of antibiotic on
neonatal outcome in PPROM??
STUDY DESIGN ??
• Randomised multi-centre trial??
Entry criteria
• PPROM at <37 weeks gestation where the need to prescribe antibiotics was
uncertain??
Exclusion criteria
• Antibiotics already prescribed or thought necessary, imminent delivery, contra-
indications to antibiotic use??
The primary outcome measure was a composite of neonatal death, chronic lung disease
(receiving daily supplementary oxygen at age 36 weeks post conception), major cerebral
abnormality on ultrasound scan before discharge??
RESULTS ??
• Women were randomised to receive erythromycin 250mg (n = 1197), co-
amoxiclav 325mg (n = 1212), both antibiotics (n = 1192) or placebo (n = 1225) four
times daily for 10 days or until delivery??

108
• Erythromycin was associated with a significantly lower risk of the primary
composite outcome compared to placebo (12.7% Vs 15.2%; p = 0.08)??
• Co-amoxiclav only and co-amoxiclav + erythromycin was not significantly
different from placebo with respect to the primary composite outcome??
• Erythromycin was associated with: ?1) A significant reduction in delivery within
48h??2) A significant reduction in the proportion of babies requiring >21% O2??3) A
significant reduction in the proportion of babies requiring surfactant treatment??4) A
significant reduction in the proportion of positive neonatal blood cultures??
• Co-amoxiclav was associated with: ??1) A significant reduction in delivery within
48h??2) A significant reduction in delivery within 7 days??3) A significant reduction
in the proportion of babies requiring >21% O2??4) A significant reduction in the
proportion of babies requiring surfactant treatment??5) A significant Increase in the
proportion of babies with suspected and proven necrotising enterocolitis (4.1% Vs
2.7% and 1.9% Vs 0.5% respectively)??
• Neither antibiotic significantly reduced the risk of perinatal death or maternal
morbidity (length of hospital stay and antibiotic use) although the combination of
erythromycin and co-amoxiclav was associated with a significant reduction in the
need for maternal antibiotic prescription within 14 days.?

Options for Questions 13-13


A significant reduction in perinatal A significant reduction in
A B
mortality chorioamnionitis
A significant reduction in the proportion No significant reduction in deliveries
C D
of babies requiring surfactant treatment within 7 days
No significant difference in the rate of
E
positive neonatal blood cultures

Explanation
In the ORACLE I randomized trial of
Questionantibiotics for pre-term pre-labour rupture A(Correct answ er: C)
13 of the membranes, erythromycin was
reported to be associated with

• Erythromycin was associated with: ?1) A significant reduction in delivery within


48h??2) A significant reduction in the proportion of babies requiring >21% O2??3) A
significant reduction in the proportion of babies requiring surfactant treatment??4) A
significant reduction in the proportion of positive neonatal blood cultures??
• Co-amoxiclav was associated with: ??1) A significant reduction in delivery within
48h??2) A significant reduction in delivery within 7 days??3) A significant reduction
in the proportion of babies requiring >21% O2??4) A significant reduction in the
proportion of babies requiring surfactant treatment??5) A significant Increase in the
proportion of babies with suspected and proven necrotising enterocolitis (4.1% Vs
2.7% and 1.9% Vs 0.5% respectively)??

109
• Neither antibiotic significantly reduced the risk of perinatal death or maternal
morbidity (length of hospital stay and antibiotic use) although the combination of
erythromycin and co-amoxiclav was associated with a significant reduction in the
need for maternal antibiotic prescription within 14 days.?

Options for Questions 14-14


No significant reduction in deliveries A significant increase in the risk of
A B
within 7 days confirmed necrotizing enterocolitis
A significant increase in the proportion A significant reduction in perinatal
C D
of babies requiring surfactant treatment mortality
A significant reduction in maternal
E
sepsis

Explanation
In the ORACLE I randomized trial of
Questionantibiotics for pre-term pre-labour rupture A(Correct answ er: B)
14 of the membranes, co-amoxiclav was
reported to be associated with

Erythromycin was associated with: ?


1) A significant reduction in delivery within 48h??
2) A significant reduction in the proportion of babies requiring >21% O2??
3) A significant reduction in the proportion of babies requiring surfactant treatment??
4) A significant reduction in the proportion of positive neonatal blood cultures??
Co-amoxiclav was associated with: ??
1) A significant reduction in delivery within 48h??
2) A significant reduction in delivery within 7 days??
3) A significant reduction in the proportion of babies requiring >21% O2??
4) A significant reduction in the proportion of babies requiring surfactant
treatment??
5) A significant Increase in the proportion of babies with suspected and proven
necrotising enterocolitis (4.1% Vs 2.7% and 1.9% Vs 0.5% respectively)??
• Neither antibiotic significantly reduced the risk of perinatal death or maternal
morbidity (length of hospital stay and antibiotic use) although the combination of
erythromycin and co-amoxiclav was associated with a significant reduction in the
need for maternal antibiotic prescription within 14 days.?

Options for Questions 15-15


A Before 22 weeks gestation B At 22 – 25 weeks gestation
C At 23 – 34 weeks gestation D After 37 weeks gestation
Before 22 weeks and after 37 weeks
E
gestation

110
Explanation
QuestionIn normal pregnancy, fetal fibronectin is A(Correct answ er: E)
15 detectable in cervico-vaginal secretions

Fetal fibronectin test (fFN)


• Detect the presence of fFN, an extracellular matrix protein located in choriodecidual
junction, in cervicovaginal secretions.
• Detection is achieved via enzyme immunoassay.
• In a normal pregnancy, fFN is detectable before maternal decidua and fetal
membranes are completely fused at 22 weeks, and after 37 weeks when fFN loses
adhesive properties
• Positive testing between 24 and 35 weeks is associated with preterm delivery

Options for Questions 16-16


A Less than 10% B 20 – 30%
C 40 – 50% D 60 – 70%
E Over 90%

Explanation
QuestionThe fetal fibronectin test has a negative A(Correct answ er: E)
16 predictive value of

Fetal fibronectin test (fFN)


• NPV of fFN testing is between 92 and 100%.
• A negative fFN translates to an over 90% likelihood that delivery will not occur
within 2 weeks of the testing
• Negative fFN tests helps avoid unnecessary hospitalization and interventions such
as steroids to induce lung maturation

Options for Questions 17-17


Significantly reduce the risk of pre-term
A B Significantly reduce perinatal mortality
birth
Significantly reduce the risk of birth
Significantly reduce the risk of serious within 48 hours of commencing
C D
perinatal morbidity treatment but not the risk of serious
perinatal morbidity
Significantly reduce the risk of birth
E within 7 days of commencing treatment
and perinatal mortality

Explanation

111
Question A(Correct answ er: D)
Tocolytics have been shown to
17

EFFICACY OF TOCOLYTICS ??
• Tocolytics reduce the proportion of deliveries occurring within 48h of commencing
treatment and may prolong pregnancy for up to 7 days??
• Used on their own, tocolytics do not reduce the incidence of pre-term delivery,
serious morbidity or perinatal mortality??
• Oral maintenance tocolytic therapy is ineffective?
• Some cases of pre-term delivery (up to 40% below 32 weeks) are associated with
infection. Any delay in delivery may be detrimental to the fetus / neonate??
• Tocolytic drugs may also be harmful to the fetus / neonate, although this has not
been demonstrated. Any potential benefits from delayed delivery may be cancelled by
potential adverse effects of the drug??

Options for Questions 18-18


A Before 26+0 weeks gestation B Before 28+0 weeks gestation
C Before 30 +0 weeks gestation D Before 34+6 weeks gestation
E Before 36+6 weeks gestation

Explanation
Magnesium sulphate for fetal neuro-
Question A(Correct answ er: C)
protection should be considered for
18
women at risk of pre-term birth

RCOG SAC Report


• Magnesium sulphate should be considered in women at less than 30 weeks of
gestation.
• While benefit has been observed at more advanced gestations, the magnitude of
effect is likely to be largest at earliest gestations and limitation of resources makes an
upper limit of 30 weeks a pragmatic choice

Options for Questions 19-19


A Women with pre-eclampsia B Women with sepsis
Women expected to give birth within 4
C Women with ruptured membranes D
hours
E None of the above

Explanation
QuestionMagnesium sulphate for fetal neuro- A(Correct answ er: E)
19 protection should not be used in

112
• All studies described included multiple pregnancies, those where delivery was
expected within 24 hours and those with premature rupture of membranes.
• This suggests that irrespective of the indication any infant delivering preterm
might be expected to benefit from the observed reduction in cerebral palsy risk

Options for Questions 20-20


In women with two or more previous In women with a previous pre-term birth
A B
pre-term births before 28 weeks gestation
In women with ultrasound evidence of In women with a positive fetal fibronectin
C D
funnelling before 24 weeks gestation test before 24 weeks gestation
E Only as part of a clinical trial

Explanation
QuestionProgesterone should be used to prevent A(Correct answ er: E)
20 pre-term birth
RCOG website
• Although a reduction in preterm birth seems attractive, there is little evidence as
yet for short term benefit to the baby, even if progesterone does prevent preterm
delivery.
• Furthermore, there is no evidence of long term benefit for the baby. This may not
just be “absence of evidence” as it is becoming increasingly recognised that a delay in
delivery could have adverse effects if the fetus remains in an adverse intrauterine
environment.
• The RCOG therefore endorses current recommendations that, in women at high
risk of preterm delivery, progesterone administration should be restricted to clinical trials
to determine whether its use is associated with improved fetal, neonatal and/or infant
outcome.

Menstrual disorders
Options for Questions 1-1
A Other causes B Ovulatory dysfunction
C Polycistic Ovary syndrome D Oestradiol
E Ovarian failure

Explanation

113
In the FIGO classification of the causes
Question A(Correct answ er: B)
of abnormal uterine bleeding, O stands
1
for

Classification of causes and investigation of abnormal uterine bleeding: FIGO system


Munro et al. International Journal of Gynecology and Obstetrics 113 (2011) 3–13
There are 9 main categories, arranged according to the acronym PALM-COEIN
Polyp
Adenomyosis
Leiomyoma
Malignancy & hyperplasia

Coagulopathy
Ovulatory dysfunction
Endometrial
Iatrogenic
Not yet classified
• The components of the PALM group are discrete (structural) entities that can be
measured visually with imaging techniques and/or histopathology
• The COEIN group relates to non-structural entities that cannot be defined by imaging
or histopathology

Options for Questions 2-2


A From the clinical history B From MRI or ultrasound scan
Only on histology following
C D By hysteroscopy and endometrial biopsy
hysterectomy
E By laparoscopy and hysteroscopy

Explanation
A healthy 34 year old woman presents
with a 12 months history of progressively
heavy and painful menstrual bleeding.
Question A(Correct answ er: B)
She has a regular 28 day cycle and is
2
not using any contraception. Her LMP
was 2 weeks ago. A diagnosis of
adenomyosis can be made

Adenomyosis (AUB-A)
• The relationship between adenomyosis and AUB is unclear and the prevalence of
adenomyosis varies widely (5 to 70%)
• The diagnosis of adenomyosis is generally based on histopathologic evaluation of the
depth of “endometrial” tissue beneath the endometrial–myometrial interface in
hysterectomy specimens
• However, ultrasound and MRI are increasingly used for diagnosis

114
• Given limited availability of MRI, it is recommended that ultrasound criteria for
adenomyosis comprise the minimum requirements for assigning an individual the
diagnosis of adenomyosis in the PALM-COEIN classification system

Options for Questions 3-3


Intramural leiomyomas are more likely
The diagnosis should be based on
A B to be associated with abnormal uterine
clinical symptoms and hysteroscopy
bleeding than submucosal leiomyomas
Sub-serosal leiomyomas are not
included in the classification system as Leiomyomas are histologically different
C D
they are not associated with abnormal from fibroids
uterine bleeding
The diagnosis is based on ultrasound
E
identification of one or more leiomyomas

Explanation
With respect to FIGO classification of
Question A(Correct answ er: E)
abnormal uterine bleeding secondary to
3
leiomyomas

Leiomyoma (AUB-L)
• Also described as “myoma,” and “fibroid.”
• Prevalence up to 70% in Caucasians and up to 80% in women of African ancestry
• May be subendometrial, intramural, subserosal, and combinations of these
• Can be asymptomatic
• The primary classification system reflects only the presence or absence of 1 or more
leiomyomas, regardless of the location, number, and size
• The criteria for determining the presence of leiomyomas would require only
ultrasound examination confirming that 1 or more are present
• In the secondary classification system, the clinician is required to distinguish
leiomyomas involving the endometrial cavity (submucosal [SM]) from others (O)
because it is generally considered that submucosal lesions are the most likely to
contribute to AUB
• The tertiary classification system includes the categorization of intramural and
subserosal leiomyomas, in addition to a category that includes types such as the
parasitic lesions that become detached from the uterus after establishing blood supply
from another source
• When a leiomyoma abuts or distorts both the endometrium and the serosa, it is
categorized initially via the submucosal classification, then by the subserosal location—
with the 2 values separated by a hyphen
• This tertiary classification may be most useful for researchers but clinicians who
perform resectoscopic myomectomy may find it useful

Options for Questions 4-4

115
Women with oligomenorrhoea Women with oligomenorrhoea
A B
secondary to PCOS secondary to hyperprolactinaemia
Women with irregular bleeding
C Women with irregular heave periods D
secondary to dopamine agonist drugs
Women with regular heavy periods for
E
which no cause has been identified

Explanation
According to the FIGO classification of
Question A(Correct answ er: E)
abnormal uterine bleeding (AUB), which
4
group of women do not have AUB-O

Ovulatory dysfunction (AUB-O)


• Generally manifests as a combination of unpredictable timing of bleeding and
variable amount of flow (AUB), which in some cases results in heavy menstrual
bleeding
• Disorders of ovulation may have a wide range of presentations including
amenorrhea, oligomenorrhoea, and episodes of unpredictable and extreme heavy
menstrual bleeding requiring medical or surgical intervention
• Some of these presentations relate to the absence of predictable cyclical
progesterone production from the corpus luteum every 22–35 days
• AUB-O is associated with endocrinopathies (e.g. polycystic ovary syndrome,
hypothyroidism, hyperprolactinemia, mental stress, obesity, anorexia, weight loss, or
extreme exercise )
• AUB-O may also be iatrogenic, caused by gonadal steroids or drugs that affect
dopamine metabolism, such as phenothiazines and tricyclic antidepressants
Ovulatory disorders are more common at the extremes of reproductive age: adolescence and
peri-menopause

Options for Questions 5-5


Break-through bleeding in a woman
Heavy menstrual bleeding secondary to
A B taking the combined oral contraceptive
the Cu IUCD
pill
Heavy menstrual bleeding in a 37 year Irregular menstrual bleeding in a woman
C D
ole woman on warfarin taking dopamine agonists
Irregular menstrual bleeding in a woman
E
on depo-medroxyprogesterone acetate

Explanation
Which one is not classified as an
Question A(Correct answ er: C)
iatrogenic cause of abnormal uterine
5
bleeding (AUB-I)?

116
Iatrogenic (AUB-I)
• Include AUB associated with intrauterine systems and drugs that directly impact the
endometrium, interfere coagulation or influence the regulation of ovulation.
• Unscheduled bleeding that occurs during the use of gonadal steroids is termed
“breakthrough bleeding (BTB)” and is the major component of the AUB-I classification
• Systemic gonadal steroids (oestrogens, progestogens, and androgens) affect the
control of ovarian steroidogenesis and have a direct effect on the endometrium
• When unscheduled bleeding occurs in the context of cyclic administration, the
woman may be considered to have BTB and be categorized as AUB-I
• Many episodes of BTB are related to reduced circulating steroid levels secondary to
compliance issues such as missed, delayed, or erratic use of pills, transdermal patches,
or vaginal rings. Suppression of FSH production is reduced and ovarian follicles develop
and produce endogenous estradiol. Additional and irregular stimulation of the
endometrium results in BTB
• Other potential causes of reduced levels of circulating oestrogens and progestogens
include the use of anticonvulsants and antibiotics (e.g. rifampin and griseofulvin)
• Cigarette smoking can reduce levels of contraceptive steroids because of enhanced
hepatic metabolism, which may explain the relatively high incidence of BTB in smokers
• Unscheduled bleeding is common (up to 25%) in the first 3–6 months of use of the
levonorgestrel-releasing intrauterine system and this contributes to a total 5-year
cumulative discontinuation rate for bleeding problems of 16.7%
• Drugs that interfere with dopamine metabolism have the potential to cause AUB
secondary to disorders of ovulation. This includes tricyclic antidepressants (amitriptyline
and nortriptyline), phenothiazines and other drugs that inhibit serotonin uptake
• The resulting reduced inhibition of prolactin release causes disruption in the
hypothalamic–pituitary–ovarian axis and consequent disorders of ovulation, including
anovulation
• Drugs that affect serotonin uptake can cause ovulatory dysfunction amenorrhea or
irregular bleeding

Options for Questions 6-6


A Offer speculum and vaginal examination B Refer for pelvic ultrasound scan
C Offer empirical treatment D Offer endometrial biopsy
E Offer screening for genital tract infection

Explanation
A 33 year old woman presents with a 6
months history of heavy menstrual
bleeding. She has a regular 28 day cycle
Question A(Correct answ er: C)
with no inter-menstrual or post-coital
6
bleeding. Her last cervical smear was 12
months ago and she always has normal
smears. She has no other bowel or

117
genito-urinary symptoms and is not using
any contraception. Her LMP was 6 days
ago.

If the history suggests HMB without structural or histological abnormality, pharmaceutical


treatment can be started without carrying out a physical examination or other investigations at
initial consultation in primary care, unless the treatment chosen is levonorgestrel-releasing
intrauterine system (LNGIUS).
If the history suggests HMB with structural or histological abnormality, with symptoms such as
inter-menstrual or post-coital bleeding, pelvic pain and/or pressure symptoms, a physical
examination and/or other investigations (such as ultrasound) should be performed.

Options for Questions 7-7


A Age over 40 years B Post-coital bleeding
C Inter-menstrual bleeding D Pelvic pain
E Pressure symptoms

Explanation
When taking a history from a woman
presenting with heavy menstrual
Question A(Correct answ er: A)
bleeding, which feature is not suggestive
7
of a structural or histological
abnormality?

NICE GUIDELINES
If the history suggests HMB without structural or histological abnormality, pharmaceutical
treatment can be started without carrying out a physical examination or other investigations at
initial consultation in primary care, unless the treatment chosen is levonorgestrel-releasing
intrauterine system (LNGIUS).
If the history suggests HMB with structural or histological abnormality, with symptoms such as
inter-menstrual or post-coital bleeding, pelvic pain and/or pressure symptoms, a physical
examination and/or other investigations (such as ultrasound) should be performed.

Options for Questions 8-8


Women with intra-cavity fibroids on
A Women with anaemia B
ultrasound scan
Women with a uterine length greater Women with a fibroid that is palpable
C D
than 12 cm on ultrasound scan abdominally
Women with persistent inter-menstrual
E
bleeding

Explanation

118
You are giving a lecture on heavy
menstrual bleeding to general
Question A(Correct answ er: A)
practitioners. Which one will you not
8
include in your list of indications for
referral to specialist care?

Women with fibroids that are palpable abdominally or who have intracavity fibroids and/or
whose uterine length as measured at ultrasound or hysteroscopy is greater than 12 cm should
be offered immediate referral to a specialist.
· Persistent inter-menstrual bleeding is suggestive of pathology including malignancy

Options for Questions 9-9


A Age over 35 years B Age over 40 years
C Age over 45 years D Heavy bleeding causing anaemia
E Family history of endometrial cancer

Explanation
Which one is an indication for
Question A(Correct answ er: C)
endometrial biopsy in women presenting
9
with heavy menstrual bleeding?

NICE Guidelines
 A biopsy should be taken to exclude endometrial cancer or atypical hyperplasia in
 Women with persistent inter-menstrual bleeding
 Women aged 45 years and over
 Women with treatment failure or ineffective treatment

Options for Questions 10-10


A Amenorrhoea B Uterine perforation
Irregular bleeding that may last over 6
C D Irregular bleeding for up to 3 months
months
E Migration of IUS into abdominal cavity

Explanation
According to the NICE guidelines,
Questionwomen wishing to use the levonorgestrel A(Correct answ er: C)
10 IUS should be informed of which
common unwanted outcome?

INFORMATION FOR WOMEN


Women should be given information on the potential unwanted outcomes of interventions for
heavy menstrual bleeding.

119
(common=1 in 100 chance, less common=1 in 1000 chance, rare=1 in 10 000 chance, very
rare=1 in 100 000 chance)
Levonorgestrel IUS
· Common: irregular bleeding that may last for over 6 months; hormone related problems
such as breast tenderness, acne or headaches (minor and transient)
· Less common: amenorrhoea
· Rare: uterine perforation at the time of IUS insertion

Options for Questions 11-11


A Anovulation B Indigestion and diarrhoea
C Worsening of asthma D Peptic ulcers
E Peritonitis

Explanation
Common unwanted outcomes of NSAIDs
Question A(Correct answ er: B)
used to treat heavy menstrual bleeding
11
include

NSAIDS – unwanted outcomes (NICE Guidelines)


· Common: indigestion; diarrhoea
· Rare: worsening of asthma in sensitive individuals; peptic ulcers with possible bleeding
and peritonitis

Options for Questions 12-12


A Haemorrhage B Persistent vaginal discharge
C Infection causing septicaemia D Non-target embolization
E Premature ovarian failure

Explanation
Which one is a common unwanted
Question A(Correct answ er: B)
outcome of uterine artery embolization
12
used to treat heavy menstrual bleeding?

UTERINE ARTERY EMBOLISATION – Unwanted outcomes (NICE)


· Common: persistent vaginal discharge; post embolisation syndrome – pain, nausea,
vomiting and fever (not involving hospitalisation)
· Less common: need for additional surgery; premature ovarian failure, particularly in
women over 45 years old; haematoma
· Rare: haemorrhage; non-target embolisation causing tissue necrosis; infection causing
septicaemia

Options for Questions 13-13

120
Results in a 96-100% reduction in
A B Is less cost-effective than hysterectomy
menstrual blood loss
Is less cost-effective than tranexamic Is the most cost-effective treatment
C D
acid option currently available
Is the least cost-effective treatment
E
option currently available

Explanation
QuestionUse of the levonorgestrel intra-uterine A(Correct answ er: D)
13 system to treat heavy menstrual bleeding

LNG-IUS
· Produces a clinically relevant reduction in blood loss in women complaining of HMB (71
- 96%).
· The full benefit of treatment may not be seen for 6 months.
· Cost-effective when compared to hormonal and non-hormonal treatments. It generates
more QALYs at a lower cost than any other medical or surgical treatment strategy considered.
· When only treatments that provide contraceptive benefits are compared, the combined
oral contraceptive (COC) pill produces fewer QALYs at a higher cost than LNG-IUS.
· Surgical strategies produced fewer QALYs at a higher cost than LNGIUS.

Options for Questions 14-14


A 11-15% B 20-25%
C 25-30% D 35-45%
E 47-56%

Explanation
Combined oral contraceptive pills (ethinyl
estradiol 30 micrograms + levonorgestrel
Question A(Correct answ er: D)
150 micro¬ grams for 21 days) used to
14
treat heavy menstrual bleeding reduce
menstrual blood loss by

COCP
COCs (ethinyl estradiol 30 micrograms + levonorgestrel 150 micro grams for 21 days) reduces
MBL by 43%.

Options for Questions 15-15


Results in a 10-15% reduction in Reduces pain associated with heavy
A B
menstrual blood loss menstrual bleeding
C Is associated with an increased risk of D Results in more regular cycles

121
venous thrombo-embolism
Does not reduce pain or regulate
E
menstrual cycles

Explanation
QuestionWhen used to treat heavy menstrual A(Correct answ er: E)
15 bleeding, tranexamic acid

TRANEXAMIC ACID
· At dose of 2–4.5 g per day for 3–5 days from the onset of bleeding causes a clinically
significant reduction in MBL (29-58%)
· When hormonal treatments are not considered acceptable, tranexamic acid generates
more QALYs at a lower cost than NSAIDs, and more QALYs but at a greater cost than a
strategy of no treatment
· Does not reduce dysmenorrhea / pain associated with bleeding
· Is not a contraceptive
· Does not regulate cycles, so advice and suitable additional treatment should be given, if
required.

Options for Questions 16-16


Are more effective than danazol in Are less effective than tranexamic acid
A reducing blood loss in women with B in reducing blood loss in women with
heavy menstrual bleeding heavy menstrual bleeding
Can be used in women with heavy Are more cost-effective than the
C menstrual bleeding secondary to Von D levonorgestrel IUS in the treatment of
Willebrand’s disease heavy menstrual bleeding
Have been shown to be effective in
E treating heavy menstrual bleeding in
women with fibroids

Explanation
Question A(Correct answ er: B)
Non-steroidal anti-inflammatory drugs
16

NSAIDs
· Mefenamic acid or naproxen produce a clinically important reduction in MBL (20-49%).
· Not as effective as danazol or tranexamic acid, but had a better adverse effect profile
than danazol.
· Cost-effective when compared with no treatment, but generated fewer QALYs at a
greater cost than LNG-IUS or tranexamic acid.
· Not contraceptives
· Beneficial for the treatment of dysmenorrhoea
· Should not be used where it is thought that HMB is caused by bleeding disorders

122
· Due to the cyclical nature of use, adverse effects are reduced
· No evidence regarding the effect of NSAIDs on HMB in the presence of uterine fibroids,
as women with fibroids were excluded from the trials

Options for Questions 17-17


A Tranexamic acid B Naproxen
C Mefenamic acid D Levonorgestrel IUS
E Combined oral contraceptive pill

Explanation
A healthy 33 year old woman presents
with a 6 months history of heavy
menstrual bleeding and no other
Questionsymptoms. Her LMP was 6 days ago and A(Correct answ er: D)
17 she has a regular 32 days cycle. She is
sexually active but not planning a
pregnancy. Which one is the
recommended first line treatment option?

NICE RECOMMENDATIONS
· Medical treatment should be considered if no structural or histological abnormality or for
fibroids less than 3 cm in diameter which are not distorting the cavity
· Determine whether hormonal contraception is acceptable to the woman before
recommending treatment
· If both hormonal and non-hormonal treatments are acceptable, treatments should be
considered in the following order
1. levonorgestrel-releasing IUS provided long-term (at least 12 months) use is anticipated
2. tranexamic acid or NSAIDs or COCP
3. norethisterone (15 mg) daily from days 5 to 26 of the menstrual cycle, or injected long acting
progestogens.

Options for Questions 18-18


A Offer diagnostic laparoscopy B Offer MRI to exclude adenomyosis
C Offer tranexamic acid D Offer mefenamic acid
Offer the combined oral contraceptive
E
pill

Explanation
A 32 year old woman presents with a 6
Questionmonths history of heavy and painful A(Correct answ er: D)
18 periods. Her LMP was 3 weeks ago and
she has a regular 28 day cycle. There is

123
no inter-menstrual or post-coital
bleeding. She is sexually active but does
not want to use contraceptives. Pelvic
ultrasound scan is normal.

If hormonal treatments are not acceptable, then either tranexamic acid or NSAIDs can be
used.
· Women offered the LNG-IUS should be advised of anticipated changes in the bleeding
pattern and be advised to persevere for at least 6 cycles to see the benefits of the treatment
· If medical treatment is required while investigations and definitive treatment are being
organised, either tranexamic acid or NSAIDs should be used.
· When HMB coexists with dysmenorrhoea, NSAIDs should be preferred to tranexamic
acid.

Options for Questions 19-19


Endometrial ablation should only be Contraception is no longer needed after
A offered if medical treatment has been B endometrial ablation using second-
unsuccessful generation techniques
Endometrial ablation is not Endometrial ablation can be offered to
C recommended in women with uterine D women with uterine size up to 10 weeks
fibroids pregnancy
Endometrial thinning is not required
E
prior to microwave endometrial ablation

Explanation
With respect to the use of endometrial
Question A(Correct answ er: D)
ablation in women with heavy menstrual
19
bleeding

ENDOMETRIAL ABLATION
· Should be considered where bleeding is having a severe impact on quality of life and the
woman does not want to conceive in the future.
· May be offered as an initial treatment for HMB after full discussion with the woman of
the risks and benefits and of other treatment options.
· Women must be advised to avoid subsequent pregnancy and on the need to use
effective contraception after endometrial ablation.
· Endometrial ablation should be considered in women who have a normal uterus and
also those with small uterine fibroids (less than 3 cm in diameter).
· In women with HMB alone, with uterus no bigger than a 10 week pregnancy,
endometrial ablation should be considered preferable to hysterectomy.
· All women considering endometrial ablation should have access to a second-generation
ablation technique.

124
· Second-generation ablation techniques should be used where no structural or
histological abnormality is present. The second-generation techniques recommended for
consideration are as follows:
1. Impedance-controlled bipolar radiofrequency ablation
2. Fluid-filled thermal balloon endometrial ablation (TBEA)
3. Microwave endometrial ablation (MEA)
4. Free fluid thermal endometrial ablation
In TBEA, endometrial thinning is not needed
In MEA, scheduling of surgery for postmenstrual phase is an alternative to endometrial thin-
ning.
First-generation ablation techniques (rollerball endometrial ablation (REA) and trans-cervical
resection of the endometrium (TCRE)) are appropriate if hysteroscopic myomectomy is to be
included in the procedure.

Options for Questions 20-20


A List for hysterectomy + BSO B Refer for genetic counselling
Explain that BSO is not recommended Screen for BRCA-1 and BRCA-2
C D
because of her age mutations
Offer yearly pelvic ultrasound scans and
E
mammograms after hysterectomy

Explanation
A 35 year old multiparous woman
presents with a 3 year history of heavy
menstrual bleeding than has not
responded to medical treatment. Her BMI
is 26 kg/m2, and she does not want any
Questionmore children. Her mother developed A(Correct answ er: B)
20 breast cancer aged 42 and her sister
developed ovarian cancer aged 46. A
decision has been taken to undertake
hysterectomy and the woman wishes to
consider bilateral oophorectomy because
of the family history of malignancy.
PROPHYLACTIC OOPHRECTOMY
· Removal of healthy ovaries at the time of hysterectomy should not be undertaken.
· Removal of ovaries should only be undertaken with the express wish and consent of the
woman
· Women with a significant family history of breast or ovarian cancer should be referred
for genetic counselling prior to a decision about oophorectomy.
· In women under 45 years old considering hysterectomy for HMB with other symptoms
that may be related to ovarian dysfunction (for example, premenstrual syndrome), a trial of
pharmaceutical ovarian suppression for at least 3 months should be used as a guide to the
need for oophorectomy.

125
· If removal of ovaries is being considered, the impact of this on the woman’s wellbeing
and the possible need for HRT should be discussed.
· Women considering bilateral oophorectomy should be informed about the impact of this
treatment on the risk of ovarian and breast cancer.

Gynae Surgical Practic

Options for Questions 1-1


Should start at the umbilicus and end at the Should start at the simphysis pubis and end
A B
simphysis pubis at the umbilicus
Should not be done using the internal mass
C Should start from both ends of the incision D
closure technique
Should be done using 0 or 1 absorbable
E
suture

Explanation
Question A(Correct answ er: C)
Closure of the mid-line abdominal incision
1

Closure of mid-line abdominal incision

• Closure of the sub-cutaneous fat will close any potential dead space and reduce the area for
seroma accumulation. However, sub-cutaneous closure has not been shown to be beneficial.
• The skin can be closed using 2,0 or 3,0 absorbable suture in a running sub-cuticular fashion
or with monofilament non-absorbable sub-cuticular or interrupted suture. Staple closure is an
alternative to suturing the skin. In a study comparing scar cosmesis at 6 months, no difference
in appearance existed in patients with suture versus staple skin closure
• Internal mass closure uses a far-far, near-near suturing technique (Smead-Jones closure). All
abdominal wall structures are included in the far-far portion. Only the anterior fascia is included
in the near-near bite. The initial stitch is similar to the single-layer mass closure. The second
bite only includes the anterior rectus fascia, approximately 0.5 cm from the fascial edge.
Closure can be interrupted or continuous.
• Both mass closure techniques require starting from each end of the incision, securing the
suture with 5 knots at each end or passing the needle through the loop if a looped suture is
used

Options for Questions 2-2

126
The aponeuroses of the external oblique,
The aponeuroses of the internal oblique
internal oblique and transversus abdominis
A B and external oblique muscles should be
muscles should be closed at the lateral
closed at the lateral margins of the incision
margins of the incision
Only the aponeurosis of the external The aponeuroses of the oblique muscles
C oblique muscle should be closed at the D should not be closed lateral to the rectus
lateral margins of the incision abdominis
Only the aponeuroses of the internal
oblique and transversus abdominis muscles
E
should be closed at the lateral margins of
the incision

Explanation
You have undertaken a caesarean section
through a low transverse abdominal incision. For
Question A(Correct answ er: C)
access, the incision has been extended laterally
2
into the external and internal oblique muscles.
During closure

If a transverse incision is extended laterally beyond the edge of the rectus abdominis muscle and into
the substance of the external and internal oblique muscles, injury to the ilioinguinal and iliohypogastric
nerves can occur. Lateral sutures should only be placed in the external oblique aponeurosis.
Options for Questions 3-3
A The inferior epigastric artery B The ilioinguinal and iliohypogastric nerves
C The ilioinguinal and femoral nerves D The iliohypogastric and femoral nerves
The superficial femoral and deep femoral
E
nerves

Explanation
Extension of a low transverse abdominal incision
Question A(Correct answ er: B)
beyond the lateral margins of the rectus
3
abdominis muscle can result in damage to

If a transverse incision is extended laterally beyond the edge of the rectus abdominis muscle and into
the substance of the external and internal oblique muscles, injury to the ilioinguinal and iliohypogastric
nerves can occur. Lateral sutures should only be placed in the external oblique aponeurosis.

Options for Questions 4-4

127
Is incised with the knife up to 1-2 cm lateral Is incised with scissors up to 1-2 cm lateral
A B
to the rectus muscle to the rectus muscle
Is incised with the knife up to 1-2 cm medial Is incised with scissors up to 1-2 cm medial
C D
to the lateral edge of the rectus muscle to the lateral edge of the rectus muscle
Is split by blunt dissection up to 1-2 cm
E
lateral to the rectus muscle

Explanation
QuestionWhen performing a Pfannestiel incision, the A(Correct answ er: B)
4 rectus sheath

Pfannestiel incision (Pfannenstiel, 1900)


• Classically located two fingers-breadth (2cm) above the pubic symphysis
• Results in good exposure to the central pelvis but limits exposure to the lateral pelvis and
upper abdomen.
• The low transverse skin incision curves gently upward laterally and is placed in a natural fold
of skin (the ’smile’ incsion)
• The subcutaneous tissue is incised with a scalpel down to the fascia, which is then nicked on
either side of the midline
• The exposed fascia is incised in a curvi-linear fashion with heavy curved Mayo scissors up to
1-2 cm lateral to the rectus muscle
• The superficial epigastric vessels are often near the lateral edges of the incision.

Options for Questions 5-5


The visceral peritoneum should not be
A B The parietal peritoneum should be closed
closed
The anterior and posterior rectus sheaths
C D Scarpa’s fascia should always be closed
should be closed
Single layer mass closure is recommended
E
in obese women

Explanation
Question A(Correct answ er: A)
During closure of the Pfannestiel incision
5

• During closure, the peritoneum does not need to be closed separately as re-epithelization
occurs within 48hours
• The rectus muscles should be carefully inspected and any bleeding areas cauterized or
ligated.

128
• Bleeding from small perforating vessels through the rectus muscle is a common source of
subfascial hematoma.
• The fascia is closed with a delayed absorbable suture ensuring all layers of the anterior
rectus sheath are incorporated.
• Unless a large area of dead space exists between the fascia and the skin (> 3cm), closure of
the Scarpa fascia is not needed
• The Kustner incision is a modification of the Pfannestiel incision. It is a slightly curved skin
incision beginning below the level of the anterior superior iliac spine and extending just below
the pubic hair line. The superficial branches of the inferior epigastric artery or vein may be
encountered.

Options for Questions 6-6


The fibres of the external oblique muscle The fibres of the transversus abdominis
A B
are split muscle are split horizontally
The fibres of the rectus abdominis muscle The aponeurosis of the transversus
C D
are split vertically abdominis muscle is divided
The aponeurosis of the internal oblique
E
muscle is divided

Explanation
Question A(Correct answ er: B)
When making the McBurney’s incision
6

McBurney’s / Gridiron incision (Charles McBurney, 1894)


• The external oblique aponeurisis is incised in the line of the fibres
• Internal oblique muscle is split transversely in the line of the fibres
• Transversus abdominis is split transversely in the line of the fibres
• Risk of scarring if the incision is not made in a skin crease
• Also risk of injury to the ilio-hypogastric and ilio-inguinal nerves and the deep circumflex
artery
• The appendix may also be approached through the Lanz incision which is a horizontal
incision located lower on the abdomen and closer to the anterior superior iliac spine than the
McBurney’s incision
• The Lanz incision offers better cosmetic results but has a greater tendency to injure the ilio-
hypogastric and ilio-inguinal nerves. This results in a increased risk of inguinal hernia
• The Gridiron incision allows extra-peritoneal drainage of abscesses, avoiding peritoneal
contamination. Can be performed in the left lower quadrant to drain abscesses on the left side
of the pelvis

Options for Questions 7-7


A The primary incision should be horizontal in B The operating table should be horizontal at

129
the skin just below the umbilicus the start of the operation
The operating table should have a 15
The skin of the lower abdominal wall should
C degrees head-down tilt at the start of the D
be picked up to insert the Verres needle
operation
The Verres needle should be inserted at 45
E
degrees to the skin

Explanation
Question A(Correct answ er: B)
During closed laparoscopy
7

Closed laparoscopy - technique


• Primary incision should be vertical from the base of the umbilicus (not in the skin below the
umbilicus). Care should be taken not to enter the peritoneal cavity.
• The Veress needle should be sharp, with a good and tested spring action. A disposable
needle is recommended, as it will fulfill these criteria.
• The operating table should be horizontal (not in the Trendelenburg tilt) at the start of the
procedure.
• The abdomen should be palpated to check for any masses and for the position of the aorta
before insertion of the Veress needle.
• The lower abdominal wall should be stabilised in such a way that the Veress needle can be
inserted at right angles to the skin and should be pushed in just sufficiently to penetrate the
fascia and the peritoneum.
• Two audible clicks are usually heard as these layers are penetrated.
• Excessive lateral movement of the needle should be avoided, as this may convert a small
needlepoint injury in the wall of the bowel or vessel into a more complex tear.

Options for Questions 8-8


A 1 in 1000 B 1 in 10,000
C 1 in 100,000 D 8 in 100,000
E 15 in 100,000

Explanation
The risk of venous thrombo-embolism in a
Questionwoman who is using the combined oral A(Correct answ er: E)
8 contraceptive pill with second generation
progestogens is

Oral Contraceptives: Risk of VTE


1) Healthy woman not taking COCP - 5 / 100,000

130
2) Second generation COCP user - 15 / 100,000
3) Third generation COCP user (desogestrel or gestodene) - 25 / 100,000
4) Pregnancy - 60 / 100,000

Options for Questions 9-9


She has metal plates in her leg following a
A B She is in cardiac failure
fracture 5 years ago
C Her BMI is over 40 D Her BMI is below 17
She has right lower limb spasticity following
E
a stroke

Explanation
A 75 year old woman has been admitted to the
Questiongynaecology ward with urinary retention and an A(Correct answ er: B)
9 abdominal mass. Use of anti-embolism stockings
is contra-indicated if

NICE VTE Guidelines


Do not offer anti-embolism stockings to patients who have:
· suspected or proven peripheral arterial disease
· peripheral arterial bypass grafting
· peripheral neuropathy or other causes of sensory impairment
· any local conditions in which stockings may cause damage, for example fragile 'tissue
paper' skin, dermatitis, gangrene or recent skin graft
· known allergy to material of manufacture
· cardiac failure
· severe leg oedema or pulmonary oedema from congestive heart failure
· unusual leg size or shape
· major limb deformity preventing correct fit.

Options for Questions 10-10


Anti-embolic stockings + unfractionated
A Anti-embolic stockings B
heparin
C Anti-embolic stockings + LMW heparin D Anti-embolic stockings + fondaparinux
Anti-embolic stockings + good hydration +
E
early mobilisation

Explanation

QuestionA 37 year old woman attends for abdominal A(Correct answ er: B)

131
10 hysterectomy for a large fibroid uterus. She is
known to have renal impairment secondary to
SLE. Her BMI is 35 kg/m2. Her platelet count is
normal. The most appropriate
thromboprophylactic regimen is

NICE VTE Guidelines


· Continue mechanical VTE prophylaxis until the patient no longer has significantly reduced
mobility.
· Add pharmacological VTE prophylaxis for patients who have a low risk of major bleeding,
taking into account individual patient factors and according to clinical judgement. Choose any
one of:
o fondaparinux
o LMWH
o UFH (for patients with renal failure).
· If the patient is expected to have significantly reduced mobility after discharge, continue
pharmacological VTE prophylaxis, generally for 5–7 days.

Options for Questions 11-11


A Fondaparinux B Anti-embolic stockings
Anti-embolic stockings + low molecular Anti-embolic stockings + good hydration
C D
weight heparin and early mobilization
Anti-embolic stockings + foot impulse
E
device

Explanation
A 47 year old woman is admitted for abdominal
hysterectomy because of a large fibroid uterus.
Question A(Correct answ er: C)
Her BMI is 26 kg/m2 and her mother suffered
11
from pulmonary embolism. The most
appropriate thrombo-prophylaxis is

NICE VTE Guidelines


· Continue mechanical VTE prophylaxis until the patient no longer has significantly reduced
mobility.
· Add pharmacological VTE prophylaxis for patients who have a low risk of major bleeding,
taking into account individual patient factors and according to clinical judgement. Choose any
one of:
o fondaparinux
o LMWH
o UFH (for patients with renal failure).

132
If the patient is expected to have significantly reduced mobility after discharge, continue
pharmacological VTE prophylaxis, generally for 5–7 days.

Options for Questions 12-12


A CO2 and Argon laser B Argon and Nd-YAG laser
C Argon laser D CO2 and Nd-YAG laser
E CO2 laser

Explanation
Question A(Correct answ er: E)
Which laser beam is readily absorbed by water?
12

• The CO2 laser is strongly absorbed by water and by all biological tissues with a high water
content.
• The extinction length is ~ 0.03 mm in water and in soft tissue with negligible reflection and
scattering
• Absorption of CO2 laser is independent of tissue colour and the thermal effects on adjacent
tissues are minimal. Tissue damage is relatively superficial, ranging from 50 to 100
micrometers. This makes the CO2 laser an extremely versatile surgical tool
• The extinction length of the Nd-YAG laser in water is about 40 mm: it can be transmitted
through clear liquids facilitating its use water filled cavities such as the urinary bladder.
• The extinction length of the Argon laser in pure water is about 80 mm. The laser is therefore
readily transmitted through clear aqueous tissues such as cornea, lens, and vitreous humor

Options for Questions 13-13


In pulsed wave mode, an external shutter is
In continuous wave mode, tissues are
A B used to temporarily cut off tissue exposure
continuously exposed to the laser beam
to the laser beam
Q-switching is used in both continuous
C Q-switching is used in pulsed mode D
wave and pulsed modes
The CO2 laser cannot be operated in
E
continuous wave mode

Explanation
Question A(Correct answ er: C)
With respect to lasers
13

Continuous wave / Pulsed mode laser?Continuous wave (cw) mode:

133
• The laser is always on. An external shutter controls the exposure time allowing the laser to
operate independent of the exposure time or the frequency of exposures. This gives the most
stable operation. A surgical CO2 or Nd:YAG laser will operate cw at intensities of a few watts to
more than 50 watts.
Pulsed mode:
• The laser out-put varies with time by switching between an ‘off’ and an ‘on’ mode. When the
laser is ‘off’, the laser energy builds up and is released in a short burst when the laser is ‘on’
allowing a large amount of laser energy to be delivered in a short time interval. One technique
to produce the short pulses is Q-switching.
• In a Q-switched laser, the laser energy is allowed to build up by making the cavity conditions
(the 'Q') unfavourable for lasing. When the energy stored in the laser medium is at the desired
level, the 'Q' is adjusted to favourable conditions, releasing the pulse. This results in high peak
powers as the average power of the laser (were it running in CW mode) is delivered in a shorter
time

Options for Questions 14-14


The power density is proportional to the The power density is inversely proportional
A B
diameter of the laser beam to the diameter of the laser beam
Doubling the diameter of the laser beam Doubling the diameter of the laser beam
C D
results in a doubling of the power density results in a halving of the power density
Halving diameter of the laser beam
E
increases the power density by a factor of 4

Explanation
QuestionWith respect to the operating parameters of a A(Correct answ er: E)
14 surgical laser

Control of lasers?
The surgeon can control:
• The power of the laser (watts)
• The spot size (millimeters)
• The exposure time (seconds). Of these three variables, power is the least useful as a
parameter and may be kept
• Power density = power per unit area of the beam (watts per square cm) is a more useful
measure than power alone as it takes into account the surface area of the focal spot.
• PD = (Power in the focal spot) / (Area of the focal spot).
• If the time of exposure is kept constant, the relationship between power density and depth
of injury is linear as the spot size is varied.
• Power density is the most important operating parameter of a surgical laser
• Power density varies inversely with the square of the diameter. Doubling the beam diameter
reduces the power density to one-fourth and halving the spot diameter increases power
density by a factor of four

134
Options for Questions 15-15
Higher tensile strength up to 20 days post-
A Higher tensile strength B
incision
Higher tensile strength up to 40 days post- Lower tensile strength up to 20 days post-
C D
incision incision
Lower tensile strength up to 40 days post-
E
incision

Explanation
QuestionCompared to a scalpel incision, a CO2 laser A(Correct answ er: D)
15 incision has

• The tensile strength in a CO2 laser incision is less than that of a scalpel incision up to day 20 after
injury but by day 40, it equalled that of the scalpel produced incision.
Options for Questions 16-16
A 500 mmol B 300 mmol
C 150 mmol D 70 mmol
E 30 mmol

Explanation
With respect to peri-operative fluid
Question A(Correct answ er: D)
management, the recommended daily sodium
16
intake for adults is

• The normal sodium intake in adults is 70 mmol/24 hours, which should be accompanied by
about 1.5 to 2.5 L (25 to 35 mL/kg/24h) of water
• The physiological response to changes in water intake or to a low sodium intake is both rapid
and efficient. However, the response to sodium excess is sluggish and even normal subjects are
slow to excrete an excess sodium load
• Chloride ions cause renal vasoconstriction and reduce glomerular filtration rate resulting in
sodium retention

Options for Questions 17-17


Organisms from the surgeon’s nose and
A B Organism from the patient’s body
throat
Organisms from the assistant’s nose and
C Organisms from surgical instruments D
throat

135
E Organisms from the surgeon’s skin

Explanation
QuestionThe majority of surgical site infections are A(Correct answ er: B)
17 caused by

Preventing surgical site infection


• Surgical site infections make up ~ 20% of all of healthcare-associated infections and affect up
to 5% of patients undergoing a surgical procedure.
• Most surgical site infections are caused by contamination with microorganisms from the
patient’s own body during surgery. The majority of surgical site infections are preventable.

Options for Questions 18-18


Be admitted to hospital on the day of Stop smoking at least 30 days before
A B
surgery surgery
Have a shower or bath with soap on the day
C D Shave pubic hair on the day of surgery
of surgery
Stop the oral contraceptive pill and use
E alternative contraception 4 weeks before
surgery

Explanation
A healthy 32 year old woman is awaiting total
Questionabdominal hysterectomy for a large fibroid A(Correct answ er: D)
18 uterus. Which one is not recommended pre-
operative advice?

Factors affecting risk of surgical site infection


Social factors: Associated with increased risk of surgical site infection
• Poor nutritional status
• Smoking cigarettes / other tobacco products. Advise to stop smoking at least 30 days before
surgery
• Length of preoperative hospital admission: prolonged admission exposes patients to hospital
flora. Underlying medical conditions should be managed as out-patient and patients should be
admitted on the day of surgery whenever possible.
• Pre-operative bacterial flora: Advise women to shower or have a bath using soap, either the
day before, or on the day of, surgery.
Preoperative shaving – associated with increased risk of surgical site sepsis. Do not use hair removal
routinely to reduce the risk of surgical site infection. If hair has to be removed, use electric clippers

136
with a single-use head on the day of surgery. Do not use razors, because they increase the risk of
surgical site infection.
Patients should be advised not to shave the surgical field within 48 hours of surgery.

Options for Questions 19-19


Use new razor to shave before admission Use her husband’s hair clippers to shave
A B
for surgery before admission for surgery
She will be provided with a razor to shave She should not shave within 48 hours of
C D
once admitted surgery
Hair would be shaved in theatre under
E
anaesthesia

Explanation
A 42 year old woman is due to attend for total
Questionabdominal hysterectomy because of a fibroid A(Correct answ er: D)
19 uterus. She phones 1 week before her operation
asking for advice about shaving her pubic hair

Preoperative shaving
Associated with increased risk of surgical site sepsis. Do not use hair removal routinely to reduce the
risk of surgical site infection. If hair has to be removed, use electric clippers with a single-use head on
the day of surgery. Do not use razors, because they increase the risk of surgical site infection.
Patients should be advised not to shave the surgical field within 48 hours of surgery.

Options for Questions 20-20


A surgical mask is only required if the
A fresh surgical mask should be used for
A patient is known or suspected to be B
every operation
infected with blood-borne viruses
A visor is unnecessary if the surgeon wears A surgical mask is unnecessary during
C D
prescription spectacles laparoscopic procedures
A surgical mask is unnecessary during
E
vaginal hysterectomy

Explanation
Question A(Correct answ er: B)
With respect to the surgical dress code
20
Personal hygiene & dress code
• A surgical mask should be worn to protect the patient from exhaled bacteria. The mask must
fit snugly to prevent passage of air around the sides. A fresh mask should be donned

137
immediately before beginning the scrub procedure and should be changed after each
procedure and more often if it becomes damp. Do not allow the mask to dangle around the
neck and it should only be handled by the ties after it is removed.
• Use protective eye wear covering front and side of the eyes, or full face visors should be
worn. Prescription spectacles are insufficient.

Management of labour

Options for Questions 1-1


She may use the birthing pool if she is not
A Use of the birthing pool is contra-indicated B
drowsy
She can use the pool after 1 hour of She can use the pool after 2 hours of
C D
diamorphine administration diamorphine administration
She can use the pool after 4 hours of
E
diamorphine administration

Explanation
A low risk 34 year old woman has been admitted
to the delivery suite in early spontaneous labour
Question A(Correct answ er: D)
at 38 weeks gestation. She has been given im
1
diamorphine for analgesia. She subsequently
requests to enter the birthing pool.

Intravenous and intramuscular opioids


• Pethidine, diamorphine or other opioids should be available in all birth settings. Women
should be informed that these will provide limited pain relief and may have significant side
effects for both the woman (drowsiness, nausea and vomiting) and her baby (short-term
respiratory depression and drowsiness which may last several days).
• Women should be informed that pethidine, diamorphine or other opioids may interfere with
breastfeeding.
• If an intravenous or intramuscular opioid is used, it should be administered with an
antiemetic.
• Women should not enter water (a birthing pool or bath) within 2 hours of opioid
administration or if they feel drowsy.

Options for Questions 2-2


A Local anaesthetic agents only B Opioid analgesic agents only

138
C 0.0625 – 0.1% bupivacaine D 1-2 mg/ml fentanyl
0.0625% bupivacaine + 1-2 micrograms / ml
E
fentanyl

Explanation
QuestionEpidural solutions used for labour analgesia A(Correct answ er: E)
2 should contain

Establishing and maintaining regional analgesia


• Either epidural or combined spinal–epidural analgesia is recommended. If rapid analgesia is
required, combined spinal–epidural analgesia is recommended.
• It is recommended that combined spinal–epidural analgesia is established with bupivacaine
and fentanyl.
• It is recommended that epidural analgesia is established with a low-concentration local
anaesthetic and opioid solution with, for example, 10–15 ml of 0.0625–0.1% bupivacaine with
1–2 micrograms per ml fentanyl. The initial dose of local anaesthetic plus opioid is essentially a
test dose and as such should be administered cautiously to ensure that inadvertent intrathecal
injection has not occurred.
• Low-concentration local anaesthetic and opioid solutions (0.0625–0.1% bupivacaine or
equivalent combined with 2.0 micrograms per ml fentanyl) are recommended for maintaining
epidural analgesia in labour.
• High concentrations of local anaesthetic solutions (0.25% or above of bupivacaine or
equivalent) should not be used routinely for either establishing or maintaining epidural
analgesia.
• Either patient-controlled epidural analgesia or intermittent bolus given by healthcare
professionals are the preferred modes of administration for maintenance of epidural analgesia?

Options for Questions 3-3


A 0.01 – 0.02% fentanyl B 0.04 – 0.06% fentanyl
C 0.0625 – 0.1% fentanyl D 0.125 – 0.25% fentanyl
E 0.225 – 0.5% fentanyl

Explanation
QuestionSolutions used to establish or maintain epidural A(Correct answ er: C)
3 analgesia during labour should contain

Establishing and maintaining regional analgesia


• Either epidural or combined spinal–epidural analgesia is recommended. If rapid analgesia is
required, combined spinal–epidural analgesia is recommended.

139
• It is recommended that combined spinal–epidural analgesia is established with bupivacaine
and fentanyl.
• It is recommended that epidural analgesia is established with a low-concentration local
anaesthetic and opioid solution with, for example, 10–15 ml of 0.0625–0.1% bupivacaine with
1–2 micrograms per ml fentanyl. The initial dose of local anaesthetic plus opioid is essentially a
test dose and as such should be administered cautiously to ensure that inadvertent intrathecal
injection has not occurred.
• Low-concentration local anaesthetic and opioid solutions (0.0625–0.1% bupivacaine or
equivalent combined with 2.0 micrograms per ml fentanyl) are recommended for maintaining
epidural analgesia in labour.
• High concentrations of local anaesthetic solutions (0.25% or above of bupivacaine or
equivalent) should not be used routinely for either establishing or maintaining epidural
analgesia.
Either patient-controlled epidural analgesia or intermittent bolus given by healthcare professionals are
the preferred modes of administration for maintenance of epidural analgesia

Options for Questions 4-4


Maternal pulse every hour and fetal heart
Maternal BP every 2 hours and frequency of
A B rate for 1 minute after a contraction every
contractions every 15 minutes
15 minutes
Maternal BP every hour and frequency of Maternal temperature every 2 hours and
C D
contractions every 30 minutes BP every hour
Maternal pulse every hour + frequency of
E
contractions every 15 minutes

Explanation
QuestionRecommendations for observations during the A(Correct answ er: C)
4 second stage of labour include

Observations during the second stage


• Hourly BP and Pulse
• 4-hourly temperature
• Half-hourly documentation of the frequency of contractions with subjective assessment of
strength of contractions
• Frequency of emptying the bladder
• Fetal heart rate by intermittent auscultation for 1 minute after a contraction at least every
5 minutes.
• Maternal pulse should be palpated if there is any fetal heart rate abnormality to differentiate the
two heart rates.
• Assessment of progress based on maternal behaviour, effectiveness of pushing and descent of the
fetus.
• There should be ongoing assessment of level of hydration, level of pain and need for pain relief.

140
Options for Questions 5-5
Compared to oxytocin + ergometrine, Compared to oxytocin + ergometrine,
A oxytocin is associated with an increased risk B oxytocin is associated with an increased risk
of post-partum haemorrhage over 500ml of blood loss over 1000 ml
Compared to oxytocin + ergometrine, Compared to oxytocin + ergometrine,
C oxytocin is associated with an increased risk D oxytocin is associated with an increased risk
of post-partum hypertension of retained placenta
Compared to oxytocin + ergometrine,
E oxytocin is associated with an increased risk
of vomiting

Explanation
QuestionWith respect to the drugs used for active A(Correct answ er: A)
5 management of the third stage of labour

• The combination of ergometrine + oxytocin takes advantage of the speed of oxytocin action and
the long-term effects of ergometrine. However, it is associated with the side-effects of ergometrine.
• Oxytocin is associated with a small but significant increase in the risk of post-partum haemorrhage
over 500 ml when compared to syntometrine® . Risk of blood loss over 1000ml is not altered.
• When 100 women are treated with oxytocin + ergometrine instead of oxytocin alone, 3 additional
episodes of blood loss >500 ml will be prevented but 1 additional case of hypertension and 10
additional cases of vomiting, will be observed.

Options for Questions 6-6


No significant difference in the risk of
A B A significant reduction in perinatal mortality
cerebral palsy
No significant difference in the caesarean No significant difference in instrumental
C D
section rate delivery rate
A significant increase in the risk of neonatal
E
seizures

Explanation
In low risk women in labour, use of continuous
Question A(Correct answ er: A)
electronic fetal heart rate monitoring is
6
associated with

CONTINUOUS ELECTRONIC FETAL MONITORING IN LABOUR

141
• Continuous electronic fetal monitoring by cardio-tocography (CTG) is now the accepted standard
for intra-partum fetal monitoring in women with additional risk factors.
Compared to intermittent auscultation, CTG is associated with:
• A halving of the risk of neonatal seizures
• A significant increase in the risk of caesarean section (relative risk 1.66; 1.30 – 2.13)
• A significant increase in the risk of instrumental delivery (relative risk 1.16; 1.01 – 1.32)
• No significant difference in overall perinatal death rate
• No significant difference in the risk of cerebral palsy
• ?These results remain true for low-risk, high-risk and preterm pregnancies and were unaffected by
access to fetal blood sampling.

Options for Questions 7-7


A Baseline of 150 bpm B Baseline of 106 bpm
C Baseline of 112 bpm D Baseline of 185 bpm
E Baseline of 96 bpm

Explanation
Question A(Correct answ er: B)
Which one is a non-reassuring feature on a CTG?
7

Classification of CTG features?

Feature Baseline (bpm) Variability (bpm) Accelerations


Decelerations

Present

110–160 ≥5
Reassuring None

100–109 < 5 for Typical variable The absence of


Non-
40– decelerations with over 50%accelerations with
reassuring
90 minutes of contractions, occurring otherwise normal
161–180 for over 90 minutes trace is of uncertain
significance

142
Single prolonged
deceleration for up to
3 minutes

Either atypical variable


decelerations with over 50%
of contractions or late
decelerations, both for over
< 100
30 minutes

< 5 for
> 180
Abnormal
90 minutes Single prolonged
deceleration for more than
Sinusoidal pattern
3 minutes
≥ 10 minutes

• If repeated accelerations are present with reduced variability, the FHR trace should be
regarded as reassuring.
• Most decelerations in labour are variable.
• Fetal tachycardia of 160–180 bpm, where accelerations are present and no other adverse
features appear, should not be regarded as suspicious.

Options for Questions 8-8


A After all deliveries B After all caesarean sections
C After all vaginal operative deliveries D After all vaginal breech deliveries
After all deliveries before 34 weeks
E
gestation

Explanation
QuestionUmbilical cord acid-base status should be A(Correct answ er: C)
8 assessed

143
Options for Questions 9-9
A CTG has a high sensitivity but low specificity B CTG has a low sensitivity but high specificity
CTG has a high sensitivity and a high
C CTG has a low sensitivity and low specificity D
specificity
CTG has a high sensitivity and high
E
specificity in high-risk labours

Explanation
QuestionWith respect to the detection of fetal acidosis A(Correct answ er: A)
9 during labour

Fetal heart rate monitoring - CTG - high sensitivity but low specificity??
· Fetal blood sampling - avoids intervention ob the basis of false positive CTGs??
· Contra-indications to FBS are: maternal HIV, fetal bleeding disorders and prematurity (<34 weeks
gestation)????
· Post-partum??RCOG recommends that umbilical artery acid-base status should be assessed (as a
minimum) after:??
1) Emergency C/S??
2) Instrumental delivery??
3) FBS has been performed in labour??
4) Delivery of a baby in a poor condition???
· Fetal heart rate monitoring - CTG - high sensitivity but low specificity??
· Fetal blood sampling - avoids intervention ob the basis of false positive CTGs??
· Contra-indications to FBS are: maternal HIV, fetal bleeding disorders and prematurity (<34 weeks
gestation)????
· Post-partum??RCOG recommends that umbilical artery acid-base status should be assessed (as a
minimum) after:??
1) Emergency C/S??
2) Instrumental delivery??
3) FBS has been performed in labour??
4) Delivery of a baby in a poor condition????
Options for Questions 10-10
Abandon the procedure and perform
A B Use Neville-Barnes forceps
caesarean section
Perform rotational forceps delivery with
C Use a vacuum extractor D
consultant supervision
Perform manual rotation to mento-
E
posterior position followed by Neville

144
Barnes forceps delivery

Explanation
You are undertaking a vaginal operative delivery
in the operating theatre on a 35 year old woman
with 3 previous vaginal deliveries. Spinal
Question A(Correct answ er: B)
analgesia has been established. There is a face
10
presentation with a mento-anterior position 2
cm below the ischial spines. The fetal heart rate
is normal.

Face presentation
• Will deliver vaginally only in the mento-anterior position?
• In the mento-posterior position, the neck is maximally extended and cannot extend further to
deliver beneath the simphysis pubis??
• 60-80% are in mento-anterior position, ~10% are mento-transverse and 20-30% are mento-
posterior??
• Labour should be managed as for vertex presentations and duration of labour is similar??
• Continuous fetal monitoring recommended??
• Electrode if needed may be applied to the mentum with care - facial oedema may obscure facial
landmarks??
• C/S for obstetric indications??
• Manual conversion to vertex presentation or rotation from mento-posterior to mento-anterior
seldom succeeds??
• Forceps applicable in the mento-anterior position with traction downwards to maintain extension
until the chin clears the simphysis pubis, then upwards to deliver by flexion. Hyperextension of the
neck should be avoided??
• Neonatologist should be present at delivery as oedema may involve the trachea and cause
respiratory difficulties??
• At diagnosis, parents should be advised that neonate will have significant facial oedema but this
would resolve spontaneously

Options for Questions 11-11


Initial direction of traction should be Initial direction of traction should be
A B
downwards to maintain extension downwards to flex the head
Initial direction of traction should be Initial direction of traction should be
C D
upwards to maintain extension upwards to flex the head
Initial direction of traction should be at
E
right angles to the axis of the birth canal

Explanation

145
You are undertaking a vaginal operative delivery
in the operating theatre on a 35 year old woman
with 3 previous vaginal deliveries. Spinal
Question A(Correct answ er: A)
analgesia has been established. There is a face
11
presentation with a mento-anterior position 2
cm below the ischial spines. The fetal heart rate
is normal.

Face presentation
• Will deliver vaginally only in the mento-anterior position?
• In the mento-posterior position, the neck is maximally extended and cannot extend further to
deliver beneath the simphysis pubis??
• 60-80% are in mento-anterior position, ~10% are mento-transverse and 20-30% are mento-
posterior??
• Labour should be managed as for vertex presentations and duration of labour is similar??
• Continuous fetal monitoring recommended??
• Electrode if needed may be applied to the mentum with care - facial oedema may obscure facial
landmarks??
• C/S for obstetric indications??
• Manual conversion to vertex presentation or rotation from mento-posterior to mento-anterior
seldom succeeds??
• Forceps applicable in the mento-anterior position with traction downwards to maintain extension
until the chin clears the simphysis pubis, then upwards to deliver by flexion. Hyperextension of the
neck should be avoided??
• Neonatologist should be present at delivery as oedema may involve the trachea and cause
respiratory difficulties??
• At diagnosis, parents should be advised that neonate will have significant facial oedema but this
would resolve spontaneously

Options for Questions 12-12


A 1 in 1000 deliveries B 5 in 1000 deliveries
C 10 in 1000 deliveries D 15 in 1000 deliveries
E 20 in 1000 deliveries

Explanation
Question A(Correct answ er: B)
The incidence of shoulder dystocia is about
12

SHOULDER DYSTOCIA ???


• Delivery that requires additional obstetric manoeuvres to release the shoulders after gentle
downwards traction has failed??

146
• Incidence ~ 6 per 1000 deliveries??
• Associated with increased maternal morbidity including PPH (11%) and 4th degree tears (3.8%)??
• Associated with increased perinatal mortality and morbidity: brachial plexus injury (4-16%), 10% of
which result in permanent injury. Maternal expulsive force may contribute to some injuries. Brachial
plexus injury may occur in the absence of shoulder dystocia and 4% have been reported to occur after
C/S??
• Erb’s palsy affecting the posterior shoulder is considered to be independent of the actions of the
accoucheur??

Options for Questions 13-13


Bring induction of labour forward to next
A B Offer caesarean section within 24-48 hours
available space
Offer caesarean section once
C D Offer caesarean section at 38 weeks
corticosteroids administered
E Keep planned induction of labour date

Explanation
A 34 year old woman with Type I diabetes
attends the antenatal clinic at 37 weeks
gestation. Her blood glucose control has been
Questionsatisfactory. Growth scan shows abdominal A(Correct answ er: D)
13 circumference and estimated fetal weight above
the 95th centile. The estimated fetal weight is
4520g. Induction of labour had been planned for
38+4 weeks gestation.

• Elective C/S for suspected macrosomia in n on-diabetics is not recommended. It has been
estimated that an additional 2345 C/S will need to be performed to prevent one permanent injury
from shoulder dystocia.??
• Elective C/S should be considered in women with diabetes mellitus and suspected fetal
macrosomia (EFW > 4.5kg).??

Options for Questions 14-14


Over 2000 additional caesarean sections Over 2000 additional caesarean sections
A need to be performed to prevent 1 case of B need to be performed to prevent 1 case of
shoulder dystocia brachial plexus injury
Over 2000 additional caesarean sections Over 200 additional caesarean sections
C need to be performed to prevent 1 case of D need to be performed to prevent one case
permanent injury from shoulder dystocia of shoulder dystocia

147
Over 2000 additional caesarean sections
E need to be performed to prevent 1
perinatal death due to shoulder dystocia

Explanation
With respect to the role of caesarean section in
Question A(Correct answ er: C)
preventing injury from shoulder dystocia in non-
14
diabetic women

• Elective C/S for suspected macrosomia in n on-diabetics is not recommended. It has been
estimated that an additional 2345 C/S will need to be performed to prevent one permanent injury
from shoulder dystocia.??
• Elective C/S should be considered in women with diabetes mellitus and suspected fetal
macrosomia (EFW > 4.5kg).??

Options for Questions 15-15


Delaying pushing for 1h after the diagnosis
Use of partograms – no effect on operative
A B of full dilatation – reduction in operative
vaginal delivery rate
vaginal delivery rate
Delaying pushing for 1-2h after diagnosis of
full dilatation – reduction in operative Use of acupuncture during labour –
C D
vaginal delivery rate in primigravidae using reduction in operative vaginal delivery rate
epidural analgesia
Use of action line on a partogram –
E
reduction in operative vaginal delivery rate

Explanation
QuestionWith respect to possible interventions to reduce A(Correct answ er: C)
15 the rate of operative vaginal deliveries

OPERATIVE VAGINAL DELIVERY


Reducing the incidence of operative vaginal deliveries
• Provide continuous support for women in labour
• Use of upright or lateral position in labour associated with a lower risk of assisted delivery
compared to supine or lithotomy position
• Use of partograms associated with fewer operative births and less use of oxytocin
• In primigravidae with an epidural, delaying pushing for 1-2h after full dilatation is diagnosed is
associated with a lower risk of rotational or mid-cavity operative delivery

148
• Discontinuing epidurals in the second stage does not reduce the need for operative delivery bus is
associated with increased pain

Options for Questions 16-16


The vacuum extractor is more likely to be
The vacuum extractor is associated with an
A B associated with maternal worries about the
increase in the need for phototherapy
baby
Non-rotational forceps are more likely to be Non-rotational forceps are more likely to be
C D
associated with shoulder dystocia associated long-term fecal incontinence
Non-rotational forceps are more likely to be
E
associated with fetal injury

Explanation
You have been asked to review a 32 year old
woman in spontaneous labour because of delay
in the second stage of labour. She has effective
epidural analgesia and the head is in a direct OA
Questionposition 2 cm below the ischial spines. There is A(Correct answ er: B)
16 1+ of caput, 1+ of moulding and the fetal heart
rate is normal. The woman is unsure about the
risks and benefits of the vacuum extractor and
non rotational forceps. She should be informed
that

Choice of instruments for operative vaginal delivery


• Soft vacuum extractor cups are associated with a significantly higher risk of failure compared to
the metal cup but a significantly lower risk of scalp trauma
• Compared to the forceps, the vacuum extractor is:
1) More likely to fail??
2) More likely to be associated with cephalohaematoma
3) More likely to be associated with retinal haemorrhage
4) More likely to be associated with maternal worries about the baby
5) Less likely to be associated with significant perineal trauma
6) No more likely to be associated with delivery by C/S
7) No more likely to be associated with low Apgar scores
8) No more likely to be associated with the need for phototherapy
9) No more likely to be associated with long-term adverse maternal / infant outcome (5 years follow-
up) including risk of altered fecal continence.

Options for Questions 17-17


A Expectant management is associated with B The value of post-mortem is reduced by

149
less maternal anxiety expectant management
The risk of DIC is not increased as long as The risk of failed induction of labour is
C D
delivery occurs within 2 weeks of diagnosis reduced by expectant management
Expectant management is associated with
E
an increased risk of infection

Explanation
A healthy 37 year old woman is confirmed to
Questionhave intra-uterine fetal death at 32 weeks A(Correct answ er: B)
17 gestation. She wishes to delay induction of
labour. She should be informed that

• In otherwise healthy women, expectant management according to maternal wishes. The


following should be explained to women wishing expectant management:
• Higher risk of complications such as DIC
• Greater maternal anxiety
• Appearance of baby would deteriorate
• Value of post-mortem is reduced
• If IOL delayed for longer than 48h, screening for DIC should be performed twice a week
• IOL with mifepristone and vaginal prostaglandin. Mifepristone reduces induction-to-delivery
interval
• If women has one previous CS, a consultant obstetrician should discuss options with the
woman including
• Mifepristone alone
• Mifepristone plus prostaglandins
• Caesarean section
• The risk of uterine rupture is higher with misoprostol compared to prostaglandin E2 and if
used, the dose of misoprostol should be reduced to 25-50 micrograms

Options for Questions 18-18


A Reassure and discharge home B Offer fetal fibronectin test
C Offer ultrasound scan for liquor volume D Admit for 24 h to observe pads
Offer vaginal examination to confirm fore-
E
waters intact

Explanation
A 33 year old woman with 3 previous vaginal
Questionbirths presents with a history of watery vaginal A(Correct answ er: A)
18 discharge at 39 weeks gestation. She is feeling
good fetal movements and there are no uterine

150
contractions. On speculum examination, there
is a white vaginal discharge but no evidence of
liquor. The fetal heart rate is normal.
NICE Intra-partum care guidelines
Suspected PROM
 Offer speculum exam
 Avoid digital vaginal exam in absence of contractions
 If membranes intact advise woman to return home

Options for Questions 19-19


Speculum examination to confirm ruptured Discuss options for induction of labour or
A B
membranes then discuss options expectant management
Speculum examination to confirm ruptured Speculum examination to confirm ruptured
C D
membranes and obtain HVS membranes and obtain LVS and HVS
FBC, CRP and speculum examination to
E
obtain LVS and HVS

Explanation
A 34 year old woman presents at 39 weeks
gestation with a history of sudden gush of fluid
Questionfrom the vagina. She has been wearing a pad A(Correct answ er: B)
19 and it is soaked with continuous leaking clear
fluid. She is feeling good fetal movements and
the fetal heart rate is normal.

NICE Guidelines
PROM certain history - No speculum exam

Options for Questions 20-20


Should originate 2-3 cm lateral to the Should be routinely offered to women with
A B
fourchette a previous third degree tear
Should originate at the fourchette, 45 to 60 Should originate 1-2 cm lateral to the
C D
degrees to the right fourchette, 45 to 60 degrees to the right
Should originate at the fourchette 30 to 45
E
degrees to the right

Explanation

QuestionAn episiotomy A(Correct answ er: C)

151
20

NICE Guidelines: Episiotomy


Carry out episiotomy only when there is:
? clinical need such as instrumental birth
? suspected fetal compromise
Do not offer routinely following previous third- or fourth degree trauma
Use mediolateral technique (between 45° and 60° to right side, originating at vaginal fourchette)

Family planning and pelvic infection


Options for Questions 1-1
Can be started up to day 5 of the menstrual
Should ideally be started at the end of
A cycle without need for additional B
menstruation
contraception
Should be started within 7 days of delivery
Should not be used beyond the age of 50
C D otherwise additional contraception will be
years
needed for 48 hours
Must be started on the day of termination
E of pregnancy otherwise additional
contraception will be required for 7 days

Explanation
Question A(Correct answ er: A)
The progestogen-only pill
1

When to start POP


· Start on day 1 of the cycle and take 1 pill everyday.
· Can be started up to and including day 5 of menstrual cycle without need for additional
contraception. If started after day 5 and woman is not pregnant, needs additional
contraception for 48 hours.
· Can be started up to and including day 21 post partum without need for additional
contraceptive cover
· Can be started within 5 days of termination of pregnancy or miscarriage (< 24wks
gestation)
· Can be continued up to age 55 yrs if no contraindications

Options for Questions 2-2


A Reassure and take the next pill B Pregnancy test

152
Take 2 pills at the same time then use
C Offer emergency contraception D
condoms for the next 7 days
Offer emergency contraception plus
E
pregnancy test in 7 days

Explanation
A 23 year old woman requests emergency
contraception. She is taking a low dose (20 mcg)
Questioncombined oral contraceptive pill. Her LMP was 5 A(Correct answ er: C)
2 days ago and she has missed the 4th pill. She has
had unprotected intercourse about every other
day in the last 2 weeks.

COCP
• If 1 or 2 missed pills (30 µg) or 1 missed pill (20 µg): take pill as soon as possible. NO need for
emergency contraception
• If 3 or more missed pill (30µg) or 2 or more missed pills (20µg): take pill as soon as possible continue
taking pill as usual and avoid sex or use condoms for 7 days
• If missed pills are in 1st week of pack (day 0-7): Emergency contraception if unprotected intercourse
in pill free interval or 1st 7 days of pack
• If pills missed in 2nd week of pack (day 8-14): NO need for emergency contraception if she has taken
at least 7 consecutive days of the pill.
• If missed pills in the 3rd week (day 15-21): Omit pill free interval of current park and continue straight
to new pack

Options for Questions 3-3


A Offer emergency contraception B Reassure
Take two pills at once and use condoms for
C D Use condoms for the next 7 days
the next 7 days
E Await pregnancy test on day 28 of her cycle

Explanation
A 20 year old woman phones for advice because
she has missed her contraceptive pills. Her LMP
was 15 days ago and she has a regular 28 day
Question A(Correct answ er: B)
cycle. She is using a low dose (20 mcg ethinyl-
3
oestradiol) combined pill and missed one pill on
day 13. she has had unprotected intercourse
several times in the last week.

153
COCP
• If 1 or 2 missed pills (30 µg) or 1 missed pill (20 µg): take pill as soon as possible. NO need for
emergency contraception
• If 3 or more missed pill (30µg) or 2 or more missed pills (20µg): take pill as soon as possible continue
taking pill as usual and avoid sex or use condoms for 7 days
• If missed pills are in 1st week of pack (day 0-7): Emergency contraception if unprotected intercourse
in pill free interval or 1st 7 days of pack
• If pills missed in 2nd week of pack (day 8-14): NO need for emergency contraception if she has taken
at least 7 consecutive days of the pill.
• If missed pills in the 3rd week (day 15-21): Omit pill free interval of current park and continue straight
to new pack

Options for Questions 4-4


A Is used to access the peritoneal cavity B Is used to access the uterine cavity
C Is used for tubal ligation D Is used during vaginal hysterectomy
Is used during radical abdominal
E
hysterectomy

Explanation
Question A(Correct answ er: C)
The Pomeroy technique
4

• Pomeroy technique - use absorbable suture to tie the base of a loop of tube near the mid-portion
and cutting off the top of the loop. Destroys 3-4cm of tube, making reversal more difficult
• Modified Pomeroy procedure (separating a small segment of tube from the mesosalpinx, tying each
end of the segment and removing the portion between the sutures) rather than Filshie clip may be
preferable for post-partum sterilisation performed at C/S or by mini-laparotomy - lower failure rates

Options for Questions 5-5


A Inhibition of the hypothalamus B Weight gain
C Drug deposits in muscle tissue D Drug deposits in adipose tissue
E Increased insulin resistance

Explanation
The delay in resumption of ovulation after
Question A(Correct answ er: C)
discontinuing depo-medroxyprogesterone
5
acetate is thought to be due to

154
Thought to be due to delayed metabolism of the drug from micro-crystalline deposits in muscle tissue

Options for Questions 6-6


A Is an intracellular gram positive bacteria B Is an extracellular gram negative bacteria
C Is an intracellular gram negative bacteria D Is an extracellular gram positive bacteria
E Typically grows on blood agar

Explanation
Question A(Correct answ er: C)
Chlamydia trachomatis
6

Chlamydia Trachomatis

 Commonest sexually transmitted infection in the UK

 Intra-cellular gram negative bacteria, can only be grown in cell culture systems

Options for Questions 7-7


DNA based testing of early morning urine
A DNA based testing of buccal swab B
sample
C ELISA based testing of low vaginal swab D ELISA based testing of endocervical swab
E DNA based testing of low vaginal swab

Explanation
QuestionWhich is the most sensitive approach to screen A(Correct answ er: B)
7 for Chlamydia trachomatis infection?

10-30% of women with PID who lack culture evidence of chlamydia infection have evidence of acute
chlamydial infection on serial antibody testing
 20-40% of sexually active women have antibodies to chlamydia trachomatis
 DNA based testing (PCR) of early morning urine specimen more sensitive than ELISA (detects chlamydia
antigens) based tests
 Produces a milder form of salpingitis with insidious onset
 May remain in the fallopian tubes for several months in untreated patients (N. Gonorrhoea persists for
a few days only)
 Chlamydia salpingitis is confined to the tubal mucosa only

155
Options for Questions 8-8
A Thrombophilia B Von Willebrand’s disease
C Severe asthma D Prolactinoma
E A strong family history of VTE

Explanation
Question A(Correct answ er: B)
Mifepristone is contraindicated in women with
8

CONTRA-INDICATIONS
• Suspected ectopic pregnancy
• Chronic adrenel insufficiency
• Long-term corticosteroid therapy
• Haemorrhagic disorders
• Anti-coagulant therapy
• Smokers over the age of 35 (avoid smoking / alcohol 2 days before and on the day of
prostaglandin administration)
• Hepatic / renal impairement
Avoid aspirin / NSAIDS for at least 8-12 days after mifepristone.

Options for Questions 9-9


Explain that it is too late for emergency
A Reassure that pregnancy is unlikely B
contraception
Offer levonorgestrel 750 mcg x 2 doses 12
C D Offer Cu IUCD
hours apart
Offer ethinyl-oestradiol 100 mcg +
E
levonorgestrel 500 mcg

Explanation
A 22 year old woman attends the emergency
gynaecology clinic requesting emergency
contraception. Her LMP was 22 days ago and she
Question A(Correct answ er: B)
has a regular 28 day cycle. She had unprotected
9
intercourse 2 weeks ago and is getting
increasingly anxious as her next period
approaches

Copper IUCD

156
• Effective post-coital contraceptive - may be used up to 5 days after unprotected intercourse
or after the most probable day of ovulation
• Hormonal methods recommended up to 72h after intercourse
• Failure rate < 0.1%, main primary complications: uterine cramps / bleeding; risk of infection
therefore avoid in women with recent / recurrent PID / multiple sexual partners.

Options for Questions 10-10


A Is licensed in the UK for 3 years B Is licensed in the UK for 5 years
Has a failure rate of about 0.1 per 100
C Is licensed in the UK for 8 years D
women over 5 years
Has an expulsion rate of about 0.5 per 100
E
women over 5 years

Explanation
Question A(Correct answ er: C)
The Cu T-380 IUCD
10

Cu T 380 or T Safe 380A


• Licensed for 8 years in theUK
• First choice IUCD - low failure rate (1.4 - 2.2 / 100 women over 5 years) and low expulsion
rate (~8 / 100 women over 5 years)

Options for Questions 11-11


A Is licensed in the UK for 3 years B Is licensed in the UK for 4 years
C Is licensed in the UK for 6 years D Is less likely to fail than the Cu T 380 IUCD
Has a similar expulsion rate to the Cu T 380
E
IUCD

Explanation
Question A(Correct answ er: E)
The multiload Cu 375 IUCD
11

Multiload Cu 375
• Licensed for 5 years
• Twice as likely to result in pregnancy compared to Cu T 380 with similar expulsion rates and
removal rates for abdominal pain and bleeding.

Options for Questions 12-12


A The size of the device B The volume of the device

157
C The amount of copper in the device D The surface area of copper in the device
The length of the device in relation to the
E
length of the uterine cavity

Explanation
Question A(Correct answ er: D)
The efficacy of Cu IUCD is dependent on
12

• Efficacy of IUCDs is dependent on the surface area of copper and the Nova-T200, and Multiload Cu
250 (devices containing 200-250 square mm of Cu) are less effective and unsuitable for long-term use.

Options for Questions 13-13


Screen for other STIs and treat Chlamydia Screen for other STIs, treat Chlamydia
A B
infection infection and refer for contact tracing
Treat Chlamydia infection, remove IUCD,
Treat Chlamydia infection, remove IUCD
C D offer emergency contraception and refer
and refer for contact tracing
for contact tracing
Screen for other STIs, treat Chlamydia
E infection, remove IUCD, offer emergency
contraception and refer for contact tracing

Explanation
A 24 year old woman attends the family
planning clinic because her screening test for
Chlamydia trachomatis has been reported as
Questionpositive. Her LMP was 3 weeks ago and she has a A(Correct answ er: B)
13 regular 28 day cycle. She had a Cu IUCD fitted 3
years ago after the birth of her second child. She
has had ‘unprotected’ intercourse several times
in the last week.

STI & PID


• Current PID (UKMEC 4 for initiation and UKMEC 2 for continuation)
• Chlamydia infection (symptomatic or asymptomatic), current purulent cervicitis or
gonorrhoea (UKMEC 4 for initiation and UKMEC 2 for continuation). Treat the STI using
appropriate antibiotics. There is usually no need for removal of the IUD if the woman wishes to
continue use. Woman should be screened for other STIs and contact tracing is essential and
best done at a GUM clinic.

Options for Questions 14-14

158
Is licensed in the UK for contraception for 8
A Contains 520 mcg levonorgestrel B
years
Releases levonorgestrel at a rate of 20 mcg Releases levonorgestrel at a rate of 52 mcg
C D
per day per day
Is licensed in the UK for the treatment of
E
heavy menstrual bleeding for 3 years

Explanation
Question A(Correct answ er: C)
The levonorgestrel intra-uterine system
14

THE MIRENA INTRA-UTERINE SYSTEM


• Contains 52mg Levonorgestrel released at the rate of 20mcg / day.
• The frame is rendered radio-opaque by impregnation with barium sulphate.
• Licensed for contraception for 5 years. Levonorgestrel - induced endometrial changes are
established within three cycles with atrophy of endometrial glands, decidualisation of the
stroma, inactivation of the epithelium, supression of spiral arterioles and an inflammatory
response. The endometrium becomes unresponsive to oestrogen.
• After removal, endometrial morphology returns to normal with menstruation within 30
days

Options for Questions 15-15


Has similar clinical effects to cyproterone
A B Has similar clinical effects to mifepristone
acetate
C Has similar clinical effects to dinoprostone D Has similar clinical effects to misoprostol
E Has similar clinical effects to carbetocin

Explanation
Question A(Correct answ er: B)
Ulipristal acetate
15

Ulipristal acetate (30 mg)


 Synthetic steroid derived from 19-norprogesterone
 Selective progesterone receptor modulator (second generation SPRM; mifepristone is first generation)
with antagonistic and partial agonistic effects
 Licensed for emergencycontraception up to 120 hours (5 days) after unprotected sexual intercourse or
contraceptive failure.
 Weaker antiglucocorticoid activity than mifepristone as a result of differences in their active
metabolites.

159
Options for Questions 16-16
A Accelerated follicular development B Ovulation
Suppression of the growth of the lead
C D Leutenisation
follicle
E Multi-follicular ovulation

Explanation
Administration of a single 30 mg dose of
Question A(Correct answ er: C)
ulipristal acetate during the mid-folicular phase
16
results in

Ulipristal: Mechanism of Action


 Inhibition or delay of ovulation. A single mid-follicular dose has been shown to suppress growth of lead
follicles.
 Administration just before, or in some cases just after, the luteinising hormone surge can inhibit
follicular rupture.
 Endometrial changes may also play a role. Early luteal administration results in delayed endometrial
maturation and alterations in progesterone-dependent markers of implantation.
 A mid-luteal dose has been shown to induce early endometrial bleeding in a dose-dependent manner.
 Alterations to the endometrium may inhibit implantation by rendering the uterus less receptive to the
trophoblast.

Options for Questions 17-17


Plasma concentrations are unaffected by
A Is metabolised by plasma esterases B
sodium valproate co-administration
Plasma concentrations may be increased by Plasma concentrations may be reduced by
C D
phenytoin co-administration ketoconazole co-administration
Plasma concentrations may be reduced by
E
St John’s wort co-administration

Explanation
Question A(Correct answ er: B)
Ulipristal acetate
17

Ulipristal acetate: Interactions


 Metabolised via cytochrome P450, in particular CYP3A4.
Liver enzyme inducers

160
 CYP3A4 inducers (e.g. rifampicin, phenytoin, carbamazepine, ritonavir, St John’s wort) may reduce
plasma concentrations of ulipristal and may reduce efficacy.
Liver enzyme inhibitors
 The effect of CYP3A4 inhibitors (e.g. ketoconazole, itraconazole, clarithromycin) may increase exposure
to ulipristal, but the significance is uncertain.

Options for Questions 18-18


A 48 hours B 7 days
C 2 weeks D 4 weeks
E 3 months

Explanation
QuestionPrior to use, the Persona contraceptive device A(Correct answ er: E)
18 needs to be programmed for

PERSONA
• Natural family planning - measures levels of LH and oestron-3-glucuronide in early morning
urine.
• Needs to be programmed for three months (test urine for 16 days in the first month and 8
days in subsequent months) before device can be relied upon.
• Not suitable for the following groups of women:
1) Cycle length <23 days or > 35 days
2) PCOS
3) Breastfeeding
4) Menopausal symptoms
5) Women taking hormonal medication
Needs to be re-programmed after post-coital contraception??Failure rate ~ 6/100 woman years with
perfect use. Much higher for ‘typical’ user.

Options for Questions 19-19


A Increases in women aged over 40 years B Increases in women aged below 22 years
C Decreases in women weighing over 70kg D Decreases in women with BMI over 25
E Is reduced in vegans

Explanation
QuestionThe efficacy of progestogen-only contraceptive A(Correct answ er: A)
19 pills

Efficacy ??

161
· Daily pill taking around same time of day (within 3 hrs) or (within 12 hrs for cerazette)?
· Failure rate for traditional POPs vary from 0.3 to 0.8 / 100 women years?
· Cerazette – pearl index 0.41 / 100 woman years?
· Efficacy increases with age particularly after 40 yrs?
· No evidence to suggest reduced efficacy in women > 70 kg therefore licensed use of 1 pill / day is
recommended?
· No evidence that one POP is more effective than another??

Options for Questions 20-20


A Can cause depression B Can cause pre-menstrual tension
Can result in improvement in women with
C Can result in a flare-up of endometriosis D
primary dysmenorrhea
Are a recognized cause of weight gain in
E
women

Explanation
Question A(Correct answ er: D)
Progestogen-only contraceptive pills
20
Advantages
??Contraceptive benefits??
· Effective - see notes on efficacy above?
· Reversible?
· No delay in return of fertility?
· Everyday regime may help compliance with some patients
??Non contraceptive benefits (including Cerazette)??
· Endometriosis – may improve symptoms?
· Primary dysmenorrhoea – inhibits ovulation therefore can relieve primary dysmenorrhoea?
· Premenstrual tension - inhibits ovulation therefore cyclical changes?
· Can be used when oestrogens pose a significant health risk e.g. smokers over 35 yrs, diabetics
with vascular complications,??
Weight gain – though weight gain has been documented, there is NO evidence of a causal association
between POP use and weight gain (FSRH Nov 2008 guidance)?

Antenatal and post-natal care

4 random questions for Best of 5 Test


Options for Questions 1-1
Multiparous women typically perceive fetal
Fetal movements are usually first perceived
A B movements at earlier gestations than
at 16-18 weeks gestation
primiparous women

162
The frequency of fetal movements
At term, normal fetuses make an average of
C increases between 20 and 32 weeks D
10 movements per hour
gestation
Women perceive more fetal movements
E
when standing compared to lying down

Explanation
Question A(Correct answ er: C)
With respect to fetal movements
1
REDUCED FETAL MOVEMENTS

 Fetal movements first perceived by the mother at 18 to 20 weeks gestation


 Multiparous women may perceive fetal movements earlier while primips may perceive movement
much later than 20 weeks
 The frequency of movement increases until the 32 weeks then plateaus until the onset of labour.
However, there is no reduction in the frequency of fetal movements in the late third trimester.
 The type of fetal movement may change as pregnancy advances in the third trimester with ‘rolling’
movements being more common in the late third trimester
 At term, fetuses make on average 31 movements per hour (range 16–45), with the longest period
between movements ranging from 50 to 75 minutes.
 Fetal movements show diurnal variation with the afternoon and evening being periods of peak activity.
Movements are usually absent during fetal ‘sleep’cycles, which usually last for 20–40 minutes and
rarely exceed 90 minutes
 Women perceive most fetal movements when lying down, fewer when sitting and fewest while
standing
 A reduction or sudden alteration in fetal movement may be a warning sign of fetal compromise or
impending fetal death. Reduced fetal movement is associated with poor perinatal outcome
 Women should be advised of the need to be aware of fetal movements up to and including the onset
oflabour and should report any decrease or cessation of fetal movements to their maternity unit.

Options for Questions 2-2


An anterior placenta may result in women Corticosteroids used to enhance fetal lung
A perceiving fewer fetal movements at 36 B maturity are associated with increased fetal
weeks gestation movements
Women perceive fewer fetal movements
Cigarette smoking is associated with an
C D with a breech presentation compared to a
increase in fetal movements
cephalic presentation
The fetal spine is likely to be anterior in
E women who do not perceive fetal
movements despite movements being

163
visualized on ultrasound scan

Explanation
Question A(Correct answ er: E)
With respect to fetal movements
2
Factors affecting fetal movements

 Anterior placenta: may decrease a woman’s perception of fetal movements before 28 weeks
 Corticosteroids to enhance fetal lung maturation: may decrease fetal movements and fetal heart rate
variability on CTG over the 2 days following administration
 Cigarette smoking: associated with a decrease in fetal movements
 Congenital anomalies: major malformations associated with reduced fetal activity
 Fetal position: 80% of fetal spines lay anteriorly in women who were unable to perceive fetal
movements despite being able to visualise them by ultrasound scanning. Fetal presentation has no
effect on perception of movement
 There is a diurnal variation with more movements being perceived in the evening

Options for Questions 3-3


The maternity assessment unit should
The woman should be advised to contact contact the woman’s community midwife
A B
her community midwife immediately and ask her to visit the woman as soon as
possible
The woman should be advised to lie on her
The woman should be asked to attend the
left side for 2 hours and attend if she does
C maternity assessment unit at 08:00 the next D
not feel 10 or more movements during this
morning if she has not felt fetal movements
period
The woman should be advised to lie on her
E left side for 2 hours and attend if she does
not feel any movements during this period

Explanation
A healthy 32 year old woman telephones the
maternity assessment unit at 19:00. She is 35
Question A(Correct answ er: D)
weeks pregnant and has just returned from
3
work. She is unsure whether she has felt fetal
movements during the day
Assessment of fetal movements

 Fetal movements should be assessed by subjective maternal perception


 Formal fetal movement counting is not recommended

164
 Women should be advised to be aware of their baby’s individual pattern of movements. If they are
concerned about a reduction of fetal movements after 28 weeks, they should contact their maternity
unit and should not wait until the next day for assessment
 If women are unsure whether movements are reduced after 28 weeks, they should beadvised to lie on
their left side and focus on fetal movements for 2 hours. If they do not feel 10 or more discrete
movements in 2 hours, they should contact their maternity unit immediately.

Options for Questions 4-4


She should be reassured and asked to re-
A CTG is recommended B attend if fetal movements are reduced
again
Induction of labour should be
Ultrasound scan is recommended to assess
C recommended at 40 weeks if her pregnancy D
liquor volume
remains uncomplicated
The woman should be given a Count to Ten
E
kick chart

Explanation
A healthy 32 year old primigravida attends the
maternity assessment unit at 36 weeks gestation
because of reduced fetal movements for 24
Questionhours. While on the unit, she perceives normal A(Correct answ er: B)
4 fetal movements. She has no other risk factors,
BP = 110/80 mmHg and fundal height is
appropriate for gestation age. Fetal heart rate is
138 / minute using a hand-held Doppler device.
Initial management & counseling

 If woman does not have reduced fetal movements from history, there are no other risk factors for
stillbirth and fetal heart rate is normal on auscultation, the woman should be reassured. She should be
advised to re-attend if she has further concerns about reduced fetal movements. Care should be taken
during clinical examination to differentiate the fetal from the maternal heart beat
 If the presence of a fetal heart beat is not confirmed,ultrasound scan should be performed
immediately

165

You might also like